Barbri - Missed Qs Flashcards

(201 cards)

1
Q

A citizen of State A asserted a state law claim of $80,000 against a citizen of State B in the federal district court. The State B citizen has a state law claim against another citizen of State B for $90,000 that arose out of the same transaction or occurrence as the original complaint. As a result, the State B citizen brought a third-party action against that person.

Does the court have subject matter jurisdiction over the State B citizen’s claim in the third-party action?

A) No, because the amount in controversy is insufficient.

B) No, because there is no diversity of citizenship.

C) Yes, because the requirements of diversity jurisdiction have been met.

D) Yes, because the court has supplemental jurisdiction.

A

D) Yes, because the court has supplemental jurisdiction.

The court has subject matter jurisdiction over this claim under its supplemental jurisdiction powers. The prerequisites for diversity of citizenship do not exist, because both are residents of State B. However, the court would have supplemental jurisdiction over a third-party claim that arose from the same transaction or occurrence as the underlying claim. Although (B) is a true statement, it is incorrect because there is supplemental jurisdiction. (A) is factually incorrect. (C) is incorrect because, as stated, diversity of citizenship is lacking.

How well did you know this?
1
Not at all
2
3
4
5
Perfectly
2
Q

Four investors, all of whom are American citizens, own as a partnership a chain of 15 car dealerships in a number of states. Two of the investors live in State A, one lives in State B, and one lives in State C. The investors leave the day-to-day operation of each dealership to a manager that the partnership employs. The investors leave the management of the entire chain of dealerships and the day-to-day operation of the partnership to several key officers that it employs. The officers operate out of the partnership’s largest dealership, which is in State D. A customer of the State D dealership sued the partnership in federal district court in State D, alleging fraud and breach of contract arising from her purchase of a car, and claiming, in good faith, damages exceeding $75,000. The customer is a citizen of State D.

Does the federal district court in State D have subject matter jurisdiction over the customer’s action against the partnership?

A) Yes, because the federal district court is located in State D and not another state.

B) Yes, because the plaintiff customer is a citizen of State D while the defendant partnership is a citizen of State A, State B, and State

C) No, because the plaintiff customer is a citizen of State D and the defendant partnership is also a citizen of State D, the state where its principal place of business is located.

D) No, because federal courts do not have subject matter jurisdiction over local transactions that take place entirely in one state.

A

B) Yes, because the plaintiff customer is a citizen of State D while the defendant partnership is a citizen of State A, State B, and State C.

The court has subject matter jurisdiction. Diversity of citizenship jurisdiction is available when (i) there is complete diversity of citizenship, meaning that each plaintiff is a citizen of a different state from every defendant; and (ii) the amount in controversy exceeds $75,000. A natural person’s citizenship is the state that is the person’s permanent home. A partnership is a citizen of each state of which one of its partners is a citizen, both limited and general. Here, the plaintiff is a citizen of State D, and the partners are citizens of State A, State B, and State C. Thus, complete diversity exists, and the amount in controversy is stated to exceed $75,000. As a result, diversity of citizenship jurisdiction exists. (A) is incorrect because diversity would be present in any federal court. Subject matter jurisdiction would thus exist in any federal court. (C) is also incorrect. A corporation’s principal place of business is one of its possible citizenships-every state in which the corporation was incorporated being the other possible citizenships. The same rule does not apply to partnerships; a partnership takes on the citizenships of its partners. (D) is an incorrect statement of the law. The fact that the transaction was local does not preclude subject matter jurisdiction founded on diversity of citizenship jurisdiction.

How well did you know this?
1
Not at all
2
3
4
5
Perfectly
3
Q

While working on a site in State A, a State B construction worker was standing near a steel crane when the crane’s boom swung near a high tension power line. The worker was electrocuted and severely injured. The worker filed an action in federal district court against the power company that owns the power lines. The action seeks $500,000 and alleges that the power company’s negligent construction, maintenance, and operation of the power lines caused the injury. The power company is a State A corporation and all its operations are in State A. The power company filed a third-party complaint against the owner-operator of the crane, a State B citizen. The third-party claim is based on state law and alleges that the crane’s owner-operator is liable to the power company for any liability the power company has to the injured worker. The worker amended his complaint to add a state law negligence claim for $500,000 against the crane’s owner-operator.

Does the federal court have subject matter jurisdiction over the worker’s claim against the owner-operator of the crane?

A) No, because the court does not have supplemental jurisdiction over the worker’s claim against the owner-operator of the crane.

B) Yes, because the claim arose from the same transaction or occurrence as the worker’s claim against the power company.

C) Yes, because all claims asserted arose from a common nucleus of operative fact.

D) Yes, because the State B worker has sufficient contacts with State A.

A

A) No, because the court does not have supplemental jurisdiction over the worker’s claim against the owner-operator of the crane.

The court does not have subject matter jurisdiction over the worker’s claim against the owner-operator of the crane. Diversity of citizenship jurisdiction is available when (i) there is complete diversity of citizenship, meaning that each plaintiff must be a citizen of a different state from every defendant; and (ii) the amount in controversy exceeds $75,000. The citizenship of an individual is his domicile, and a corporation is a citizen of every state in which it is incorporated and the one state in which it has its principal place of business. Here, the worker is from State B and the power company is a State A corporation with all its operations (and therefore its principal place of business) in State A. The plaintiff’s claim is for $500,000, satisfying the amount in controversy requirement. Accordingly, there is diversity of citizenship jurisdiction over this claim. Once one claim satisfies the requirements for original federal subject matter jurisdiction, the court has discretion to exercise supplemental jurisdiction over related claims that derive from the same common nucleus of fact and are such that a plaintiff would ordinarily be expected to try them in a single judicial proceeding. However, for cases based solely on diversity, supplemental jurisdiction is not available for claims by the plaintiff against persons made parties under the impleader rules when use of supplemental jurisdiction would be contrary to the requirements of diversity jurisdiction. In the instant case, the owner-operator shares state citizenship with the worker and was made a party when the power company impleaded him on a claim for indemnity. Because a claim by the worker against the owner-operator would circumvent the complete diversity requirement, supplemental jurisdiction is not available for that claim. Hence, (A) is correct. (B) and (C) are incorrect because supplemental jurisdiction cannot be used to override the requirements of diversity jurisdiction for claims by a plaintiff against an impleaded party (as is the case in this question). (D) is an incorrect statement of law, and irrelevant. First, subject matter jurisdiction (the power to hear a case) must be distinguished from personal jurisdiction (the power over a particular defendant). A court technically may have subject matter jurisdiction (for example, when diversity jurisdiction is satisfied) without having personal jurisdiction over one of the defendants because the defendant has insufficient contacts with the jurisdiction. Furthermore, a plaintiff consents to the personal jurisdiction of the court by filing suit. Here, that means the State B worker submitted to personal jurisdiction in State A by filing suit.

How well did you know this?
1
Not at all
2
3
4
5
Perfectly
4
Q

An author from State A filed a claim in federal court sitting in State B against a publisher headquartered in State B. The complaint alleged that the publisher plagiarized a portion of the author’s book and asserted both a copyright infringement claim under federal law and an unfair business practices claim under an applicable State B statute. At trial, the publisher presented evidence that the author never filed the copyright infringement claim with the appropriate federal agency, as required by the federal statute, thereby invalidating the copyright infringement claim. The claim for unfair business practices, however, was still capable of obtaining a favorable verdict. The publisher moved for dismissal of the state law claim as well.

How should the federal court rule?

A) The court must remand the state law claim to state court because there is no federal subject matter jurisdiction without the copyright infringement claim.

B) The court must dismiss the state law claim, because there is no federal subject matter jurisdiction without the copyright infringement claim.

C) The court should, in its discretion, retain jurisdiction over the state law claim because the trial has begun.

D) The court should, in its discretion, dismiss the state law claim because the jury has not yet begun to deliberate.

A

C) The court should, in its discretion, retain jurisdiction over the state law claim because the trial has begun.

The court may dismiss or hear the state law claim in its discretion, but will likely retain jurisdiction over it. When a claim is in federal court under federal question jurisdiction, and the plaintiff has a state law claim against the defendant that cannot invoke diversity jurisdiction, the federal court has discretion to exercise supplemental (pendent) jurisdiction over the state law claim if the federal and state claims derive from a common nucleus of operative fact and are such that a plaintiff would ordinarily be expected to try them all in one judicial proceeding. The court may continue to exercise supplemental (pendent) jurisdiction over the state claim even though the federal claim is dismissed on the merits. However, the state claim should probably also be dismissed (without prejudice) if the federal claim is dismissed before trial. Here, although the federal copyright claim was invalid, it was deemed to be so during trial, before a verdict was rendered. Since the case is currently being tried, the court will likely hear the state claim for the sake of judicial economy. (B) is wrong, because, as stated above, the federal court has the discretion under supplemental jurisdiction to hear the state claim. (A) is wrong for the same reason, and also because remand can only occur when the action was commenced in state court and then removed to federal court. (D) is wrong because, although the decision is within the court’s discretion, the start of the trial is the point at which a federal court will usually retain jurisdiction over supplemental state law claims for the sake of judicial economy.

How well did you know this?
1
Not at all
2
3
4
5
Perfectly
5
Q

A citizen of State A filed a complaint alleging negligence by two parties, both of whom reside in State B. The complaint was filed in the United States District Court of State B. However, the accident took place in State D, the plaintiff was treated by an emergency room physician in State D, and all of the witnesses reside in State D. Two months after they filed their respective answers, which did not address any problems with personal jurisdiction or venue, the defendants filed a joint motion to transfer the case to the District of State D.

May the court grant the motion?

A) No, because the defendants have waived any issue as to venue by not raising the issue of venue in the answer.

B) No, because a plaintiff is entitled to choose venue.No, because a plaintiff is entitled to choose venue. - no response given

C) Yes, because the defendants raised the issue in their first motion before the court.Yes, because the defendants raised the issue in their first motion before the court. - no response given

D) Yes, because transferring the case to the District of State D could be “in the interests of justice.”

A

D) Yes, because transferring the case to the District of State D could be “in the interests of justice.”

The court could grant the motion. Pursuant to Rule 12(b), improper venue must be raised in a defendant’s first response-either in its timely motion to dismiss before the answer or in the answer, whichever is first. Here, though, venue was initially proper because the defendants resided in State B. Rather, the issue is whether the court may transfer the case from one proper venue to another. Such a transfer has no strict time limit. Rather, transfer is left to the discretion of the trial judge, and the judge may refuse transfer where the case has been pending for some time and would work a prejudice to one of the parties. Two months is likely not a sufficient period to prevent transfer of the case. Thus, (D) is correct. (C) is incorrect. First, it misstates Rule 12. Rule 12 defenses must be raised in the first motion or answer, whichever comes first. Second, transfer where venue is initially proper does not fall under the Rule 12 timing requirements. (B) is incorrect because, although a plaintiff is entitled to choose the initial venue, venue may be transferred, in the interests of justice, for the convenience of the parties and witnesses despite the plaintiff’s initial choice of the forum. (A) is incorrect because, as explained, transfer of venue in the interests of justice does not fall under the “first pleading or motion” rule.

How well did you know this?
1
Not at all
2
3
4
5
Perfectly
6
Q

Select the statement that best describes the relationship between removal and venue:

Responses

A) In a properly removed case, venue is proper in the federal court of the state where the case was pending, even if venue would have been improper had the plaintiff originally filed the action in the federal district court of that state

B) In a properly removed case, venue is proper in the federal court of the state where the case was pending, but only if venue would have been proper had the plaintiff originally filed the action in the federal district court of that state

C) Venue and removal have no correlation

A

A) In a properly removed case, venue is proper in the federal court of the state where the case was pending, even if venue would have been improper had the plaintiff originally filed the action in the federal district court of that state

In a properly removed case, venue is proper in the federal court of the state where the case was pending, even if venue would have been improper had the plaintiff originally filed the action in the federal district court of that state. This is because venue for an action removed under section 1441(a) lies in the federal district court “embracing the place where such [state] action is pending.” Hence, it is not correct to state that venue and removal have no correlation.

How well did you know this?
1
Not at all
2
3
4
5
Perfectly
7
Q

When a defendant attempts to remove a case from state court to federal court, the state court _______ have had subject matter jurisdiction over the case.

A) Need not

B) Must

C) Must not

A

A) Need not

By statute, the state court need not have had subject matter jurisdiction over the case. A federal court may decide a claim in a removed civil action even if the state court had no subject matter jurisdiction.

Hence, it is not correct that the state court must or must not have had subject matter jurisdiction over the case.

How well did you know this?
1
Not at all
2
3
4
5
Perfectly
8
Q

A patient sued a surgeon for medical malpractice, alleging that the surgeon used an improper procedure during cardiac surgery, thereby permanently injuring the patient. The surgeon then wrote to a colleague, who was a renowned cardiologist, asking whether in her opinion the procedure was proper. In response, the colleague stated that the procedure was likely improper, but that she was unsure whether the patient’s injuries resulted from it. After receiving the colleague’s letter, the surgeon did not have any further communication with her about the matter.

Is the letter discoverable?

A No, because the colleague did not have first-hand knowledge regarding the surgery that she was evaluating.

B No, because the colleague will not testify at trial on behalf of the doctor.

C Yes, because the colleague was not retained or specially employed by the doctor.

D Yes, because the colleague is not an expert

A

B No, because the colleague will not testify at trial on behalf of the doctor.

The letter is not discoverable because the colleague is a nontestifying expert. An expert is a person with knowledge, skill, experience, training, or education in a particular field. The opinion of an expert consulted in anticipation of litigation is discoverable if a party intends to call the expert as a trial witness. Conversely, the opinion of an expert who is retained in anticipation of litigation but who is not expected to testify at trial (i.e., a consulting expert) is discoverable only upon a showing of exceptional circumstances under which it is impracticable to obtain facts or opinions by other means. Absent such circumstances, these opinions are not discoverable, regardless of whether the nontestifying expert was informally consulted or whether she was retained or specially employed. Thus, the doctor need not produce the letter from his colleague, an expert in cardiology, because the doctor does not expect to call her as a trial witness. (A) is incorrect because an expert need not have first-hand knowledge of facts to give an opinion based on those facts. In any event, although a fact witness’s lack of knowledge may in some circumstances provide a reason for excluding testimony at trial, it is not a valid ground for resisting discovery of relevant information. (C) is incorrect because the opinion of a nontestifying expert is nondiscoverable regardless of whether she is formally retained or not. (D) is incorrect because the cardiologist is an expert. The question states that she is a renowned cardiologist and that the doctor is consulting her about a cardiac surgery.

How well did you know this?
1
Not at all
2
3
4
5
Perfectly
9
Q

A car collector bought a car with gold leaf paint from a manufacturer. During the first month, all the gold leaf paint peeled off. The collector sued the manufacturer, and during discovery served an interrogatory asking the manufacturer to identify all other purchasers of the gold leaf paint over the previous 10 years. The manufacturer was aware that only about 25 of the 2 million buyers of its cars have ordered the gold leaf option. The manufacturer has retained copies of all sales forms, but has not maintained separate files of the buyers of each particular option.

In a court using the federal rules, what are the manufacturer’s obligations with respect to the collector’s interrogatory?

Responses

A It must search its files and then disclose the information.

B It may allow the collector to search the records himself.

C It may respond by stating that only about 25 of the 2 million buyers ordered the option.

D It may state that searching the records is too great a burden and so it is excused from answering the interrogatory.

A

B It may allow the collector to search the records himself.

Assuming the interrogatory is otherwise proper, the manufacturer may search the 2 million order forms itself or it may allow the collector access to the files. In a situation where desired information may be ascertained from the business records of the party on whom the interrogatory was served, and where the burden of finding the information is substantially the same for the party serving the interrogatory as for the party served, it is a sufficient answer to provide the serving party reasonable opportunity to examine the records. [Fed. R. Civ. P. 33] (A) is incorrect because instead of searching the files itself, the manufacturer can allow the collector access to do it. (C) is incorrect because it is not responsive to the interrogatory. (D) is incorrect because a party is not relieved of its duty to answer just because the search is burdensome.

How well did you know this?
1
Not at all
2
3
4
5
Perfectly
10
Q

In a federal question case, the action is always deemed commenced for statute of limitations purposes when:

A Either the complaint is filed or process is served on the opposing party, depending on the law of the state in which the federal court sits

B Process is served on the opposing party

C The complaint is filed with the court

D The complaint is filed, if service of process occurs within a specified timeframe

A

C The complaint is filed with the court

In a federal question case, the action is deemed commenced for statute of limitations purposes when the complaint is filed with the court. In diversity cases (not federal question cases), the Supreme Court has held that the state rule for determining when the action is commenced applies. Thus, depending on the state rule, an action may be commenced for statute of limitations purposes when process is served on the opposing party. However, the federal rule (as described above) applies in federal question cases. The choice providing commencement when either the complaint is filed or process is served on the opposing party, depending on the law of the state in which the federal court sits is incorrect; this essentially states the rule when subject matter jurisdiction is based on diversity of citizenship. The choice providing for commencement when the complaint is filed, if service of process occurs within a specified timeframe is incorrect. Some states have such dual provisions, in that the date that the complaint is filed is used when service occurs within a certain timeframe after filing, but if that timeframe for service is exceeded, the date of service is used. But that is not the rule in federal question cases.

How well did you know this?
1
Not at all
2
3
4
5
Perfectly
11
Q

A manufacturer sued a department store for breach of contract, alleging that the department store failed to pay for a shipment of watches delivered by the manufacturer to the department store in July of that year.

At trial, the manufacturer introduced evidence regarding the department store’s failure to pay for the watches. In addition, it introduced evidence that the department store failed to pay for a shipment of necklaces, also delivered that July pursuant to a separate contract. At the close of the manufacturer’s case, the department store introduced evidence showing that both the watches and the jewelry were defective. The jury returned a verdict for the manufacturer, awarding the manufacturer damages for breach of both contracts. The manufacturer then immediately moved to amend its complaint to conform to the evidence introduced at trial.

Should the court allow the amendment?

A Yes, because the department store impliedly consented to the introduction of the evidence.

B Yes, because the parties entered into the contracts at approximately the same time.

C No, because the motion should have been made before the verdict was rendered.

D No, because the defendant must have had knowledge of the claim when the complaint was served.

A

A Yes, because the department store impliedly consented to the introduction of the evidence.

The court should allow the amendment. An issue not raised by the pleadings is treated as if raised in the pleadings if it is tried by the parties’ express or implied consent. In such a case, a party may move-at any time, even after judgment-to amend the pleadings to conform them to the evidence and to raise the unpleaded issue. Here, the department store impliedly consented to the introduction of evidence relating to the jewelry contract. Not only did it fail to object to the evidence, it introduced evidence to show that the jewelry was defective. Thus, the manufacturer was entitled to move to amend its complaint to conform to the evidence introduced at trial. (B) is incorrect because the relevant determination is whether the party impliedly consented to the trial of the issue, not whether the issue was related to an issue raised by the pleadings. (C) is incorrect because a motion to amend a pleading to conform to the evidence introduced at trial may be made at any time, even after judgment. (D) is incorrect because the defendant need not know about the claim when the complaint was served.

How well did you know this?
1
Not at all
2
3
4
5
Perfectly
12
Q

A woman sued her employer for sexual harassment. At the close of the trial, the employer made a motion for judgment as a matter of law, arguing that the woman’s evidence was insufficient to establish the elements of her claim. The court denied the motion.

When the jury returned a verdict in favor of the woman, the employer renewed its motion for judgment as a matter of law. In addition to the renewed motion for judgment as a matter of law, the employer also moved for a new trial, asserting that the verdict was against the weight of the evidence. The court denied both motions.

If the employer appeals the denials of both the renewed motion for a judgment as a matter of law and the motion for a new trial, what is the appropriate standard of review?

A De novo for the renewed motion for judgment as a matter of law and abuse of discretion for the new trial motion.

B Clearly erroneous for the renewed motion for judgment as a matter of law and de novo for the new trial motion.

C Abuse of discretion for both.

D De novo for both.

A

A De novo for the renewed motion for judgment as a matter of law and abuse of discretion for the new trial motion.

(A) is correct. When an appellate court reviews a trial court’s ruling on a post-trial motion for judgment as a matter of law (including a renewed one), it employs a de novo standard. It does so because the issue is one of law. In contrast, when an appellate court reviews a trial court’s denial of a motion for new trial, it employs a more deferential standard, reversing the trial court’s denial only when there is a clear showing of an abuse of discretion. Note that when a renewed motion for judgment as a matter of law and a motion for a new trial are made in the alternative and the renewed motion is granted, the court rules hypothetically on the new trial motion so that no remand is required if the ruling on the judgment as a matter of law is subsequently reversed on appeal.

How well did you know this?
1
Not at all
2
3
4
5
Perfectly
13
Q

Once a final judgment on the merits has been rendered on a particular cause of action, the claimant is prevented from asserting the same cause of action in a later lawsuit by the doctrine of claim preclusion (res judicata).

When the claimant won the earlier lawsuit, the claim is said to be:

A
Barred by the prior judgment

B
Either merged into or barred by the prior judgment

C
Merged into the prior judgment

A

C
Merged into the prior judgment

Although both merger and bar are used to indicate that claim preclusion (res judicata) is in effect, where the claimant won the earlier lawsuit, the claim is said to be merged into the prior judgment.

Where the defendant won the earlier lawsuit, the claim is said to be barred by the prior judgment. Because the question specifically states that the claimant won the earlier suit, to say that the claim either merged into or is barred by the prior judgment is incorrect.

How well did you know this?
1
Not at all
2
3
4
5
Perfectly
14
Q

A consumer filed a breach of contract action against a seller in a state court in State A, seeking $100,000 in damages. The consumer was a citizen of State A. The seller was a State B corporation whose principal place of business was in State A. Five days after being served with the complaint and summons, the seller removed the action to federal district court. Seven months later, the consumer filed a motion to remand the action back to state court.

How should the federal court rule on the motion to remand the action to state court?

Responses

A Deny the motion, because it was untimely.

B Deny the motion, because the federal court has subject matter jurisdiction over the action based on its diversity of citizenship jurisdiction.

C Grant the motion, because the federal court lacks subject matter jurisdiction.

D The federal court has discretion to either hear the action or remand it to state court.

A

C Grant the motion, because the federal court lacks subject matter jurisdiction.

The court should grant the motion. A defendant may remove an action that could have originally been brought in the federal courts. Diversity jurisdiction requires complete diversity-meaning that each plaintiff must be a citizen of a different state from every defendant-and the amount in controversy must exceed $75,000. The citizenship of an individual is his permanent home, and a corporation is a citizen of every state in which it was incorporated and the one state in which it has its principal place of business. Here, the consumer was a citizen of State A, and the corporation was a citizen of State B (its state of incorporation) and of State A (in which it had its principal place of business). Thus, the case is not removable because complete diversity does not exist, and, as a result, there is no subject matter jurisdiction based on diversity of citizenship. Nonetheless, the case here was removed to federal court. If the plaintiff bases the motion to remand on a defect other than subject matter jurisdiction (i.e., a defect in the removal procedure), the motion to remand must be brought within 30 days of removal. There is no such time limit for a lack of subject matter jurisdiction. The court must remand whenever it is shown that there is no subject matter jurisdiction. Here, since the motion to remand is based on a lack of subject matter jurisdiction, as explained above, the fact that seven months has passed since removal is largely irrelevant. As a result, (C) is correct and (A) is incorrect. (B) is incorrect because diversity of citizenship jurisdiction does not exist, as explained above. (D) is incorrect because the court is without discretion to keep the case once it is shown that there is no subject matter jurisdiction. (This is to be distinguished from the fact that the federal court has discretion to remand a case to state court once all federal claims have been resolved over which there is no diversity jurisdiction (and over which the court had supplemental jurisdiction). Here, there was no claim within the court’s subject matter jurisdiction.)

How well did you know this?
1
Not at all
2
3
4
5
Perfectly
15
Q

A restaurant owner in State A bought two large freezers from a manufacturer of commercial refrigeration equipment with its principal place of business in State B. Within one week and after being fully stocked with meat, one of the freezers broke down. The restaurant owner filed a state-based products liability action against the manufacturer in federal court in State A, and included a demand for a jury trial. Under the law in State A, jury verdicts do not need to be unanimous, but the Federal Rules of Civil Procedure require jury verdicts to be unanimous.

At trial, the restaurant owner makes a motion asking the court to apply the State A law.

How should the court rule on the motion?

A Grant the motion, because applying the federal rule may change the outcome of the case.

B Grant the motion, because, when a federal court has diversity jurisdiction, it is required to apply the substantive law of the state in which it is sitting.

C Deny the motion, because the Federal Rules of Civil Procedure apply in federal court as long as they are consistent with the Rules Enabling Act and not unconstitutional.

D Deny the motion, because the Supreme Court’s balancing factors indicate that federal law should apply.

A

C Deny the motion, because the Federal Rules of Civil Procedure apply in federal court as long as they are consistent with the Rules Enabling Act and not unconstitutional.

The court should deny the motion. Under the Erie doctrine, when a state law-based claim is brought in federal court based on diversity of citizenship, the federal court generally applies the substantive law of the state in which it is sitting. However, where a specific federal statute or the Federal Rules of Civil Procedure are on point, the federal court must apply federal procedural law as long as the federal rule is valid. Under the Rules Enabling Act, a Federal Rule is valid if it deals with “practice or procedure” and does not “abridge, enlarge, or modify” a substantive right. Here, there is a specific federal procedural rule that is on point [Fed. R. Civ. P. 48], which requires jury verdicts to be unanimous, unless the parties agree otherwise. Since there is no evidence of agreement, the federal procedural rule will apply, and the motion should be denied. (A) is wrong because it states the wrong conclusion, and it incorrectly cites the “outcome determination” test. This is the test that may be applied when there is no federal procedure law on point, and it is unclear whether the state matter is substantive or procedural. As stated above, there is a federal procedural rule on point. Therefore, this test does not apply. Similarly, (D) is wrong because it is referring to the balance of interests test, which is another test that may be applied when there is no federal procedural law on point, and it is unclear whether the state matter is substantive or procedural. This test is not applicable here, however, because there is a federal procedural rule on point. (B) is wrong because, although it is a true statement of law that a federal court with diversity jurisdiction is required to apply the substantive law of the state in which it is sitting, the requirements for a jury verdict is a procedural rule, not a substantive rule. Therefore, as set forth above, the federal procedural rule governs.

How well did you know this?
1
Not at all
2
3
4
5
Perfectly
16
Q

A pedestrian sued a driver for personal injuries in federal court, properly invoking diversity of citizenship jurisdiction. In the complaint, the pedestrian alleged that the driver ran a red light and struck the pedestrian while the pedestrian was in the crosswalk. Concurrent with the accident, a police report was prepared on which the name and address of a witness to the accident was listed, but neither party requested a copy of the report from the police department. Thus, when the pedestrian submitted an interrogatory to the driver for the names and addresses of persons with knowledge of the accident known to the driver, the driver truthfully omitted the name of the witness. When asked during his deposition whether he knew of any witnesses, the driver again truthfully answered “I don’t know of any.”

At trial, the jury found for the driver. In one of the special interrogatories answered by the jury, the jury found that the driver had the green light and that the pedestrian was crossing against the light. Six months and a day after a final judgment was rendered in favor of the driver, the pedestrian’s attorney was contacted by the witness, who stated that the driver ran a red light and that the pedestrian had the “walk” sign when he attempted to cross the road.

The pedestrian’s attorney immediately moved for relief from judgment based on newly discovered evidence, and the trial judge granted the motion. On appeal, what should the court do?

A Affirm the trial judge’s decision, because whether to deny or grant a motion for relief from judgment is strictly a matter within the trial judge’s discretion.

B Affirm the trial judge’s decision, because the driver should have discovered the identity of the witness.

C Reverse the trial judge’s decision, because more than six months had passed since the judgment was rendered when the trial judge granted the motion.

D Reverse the trial judge’s decision, because the pedestrian could have discovered the witness’s identity with reasonable diligence.

A

D Reverse the trial judge’s decision, because the pedestrian could have discovered the witness’s identity with reasonable diligence.

Civil Procedure Capstone
17 of 2517 of 25 Items
17:23

Question
A pedestrian sued a driver for personal injuries in federal court, properly invoking diversity of citizenship jurisdiction. In the complaint, the pedestrian alleged that the driver ran a red light and struck the pedestrian while the pedestrian was in the crosswalk. Concurrent with the accident, a police report was prepared on which the name and address of a witness to the accident was listed, but neither party requested a copy of the report from the police department. Thus, when the pedestrian submitted an interrogatory to the driver for the names and addresses of persons with knowledge of the accident known to the driver, the driver truthfully omitted the name of the witness. When asked during his deposition whether he knew of any witnesses, the driver again truthfully answered “I don’t know of any.”

At trial, the jury found for the driver. In one of the special interrogatories answered by the jury, the jury found that the driver had the green light and that the pedestrian was crossing against the light. Six months and a day after a final judgment was rendered in favor of the driver, the pedestrian’s attorney was contacted by the witness, who stated that the driver ran a red light and that the pedestrian had the “walk” sign when he attempted to cross the road.

The pedestrian’s attorney immediately moved for relief from judgment based on newly discovered evidence, and the trial judge granted the motion. On appeal, what should the court do?

Responses

A Affirm the trial judge’s decision, because whether to deny or grant a motion for relief from judgment is strictly a matter within the trial judge’s discretion.Affirm the trial judge’s decision, because whether to deny or grant a motion for relief from judgment is strictly a matter within the trial judge’s discretion. - incorrect

B Affirm the trial judge’s decision, because the driver should have discovered the identity of the witness.Affirm the trial judge’s decision, because the driver should have discovered the identity of the witness. - no response given

C Reverse the trial judge’s decision, because more than six months had passed since the judgment was rendered when the trial judge granted the motion.Reverse the trial judge’s decision, because more than six months had passed since the judgment was rendered when the trial judge granted the motion. - no response given

D Reverse the trial judge’s decision, because the pedestrian could have discovered the witness’s identity with reasonable diligence.Reverse the trial judge’s decision, because the pedestrian could have discovered the witness’s identity with reasonable diligence. - not selected, this is the correct answer
Answer Discussion - Incorrect

The court should reverse the trial judge’s decision. On motion and just terms, the court may relieve a party from a final judgment or order on the following grounds: (i) mistake, inadvertence, surprise, or excusable neglect; (ii) newly discovered evidence that by due diligence could not have been discovered in time to move for a new trial; (iii) fraud, misrepresentation, or other misconduct of an adverse party; (iv) the judgment is void; (v) the judgment has been satisfied, released, or discharged; a prior judgment on which it is based has been reversed or otherwise vacated; or it is no longer equitable that the judgment should have prospective application; or (vi) any other reason justifying relief from the operation of the judgment. For grounds (i), (ii), and (iii), the motion must be made within a reasonable time not to exceed one year from the judgment; for the other grounds, the motion must be made within a reasonable time. (But remember that a lack of subject matter jurisdiction may be raised at any time.) Such a motion is left to the trial judge’s discretion, and, on appeal, her decision will be reviewed on an “abuse of discretion” standard. Here, the motion would be based on newly discovered evidence. When deciding whether to grant or deny such a motion, the trial judge must determine whether the evidence could have been discovered with any amount of due diligence. Here, the name and address of the witness could have been easily discovered had the pedestrian’s attorney simply requested a copy of the police report from the police department. This is such an obvious case of lack of due diligence on the part of a represented party that it would be an abuse of discretion to grant relief based on newly discovered evidence. (C) is incorrect because the time period for bringing a motion for relief from judgment based on newly discovered evidence is within a reasonable time, not to exceed one year from the judgment, not six months. (A) is incorrect because, although the decision is left to the trial judge’s discretion, the exercise of discretion would be reviewed on appeal on an abuse of discretion standard, and here, the newly discovered evidence could have easily been discovered in time for trial with minimal preparation. Thus, the judge very likely abused discretion by granting the motion. (B) is incorrect. Although a party is required to disclose information within his possession upon a proper request (and after a proper search of his records), and to truthfully answer questions at a deposition, those requirements do not relieve the opposing party from conducting an investigation of her own case. In other words, the fact that the driver’s attorney may have been grossly negligent in preparing his case does not relieve the failure of the pedestrian’s attorney to discover the identity of the witness.

How well did you know this?
1
Not at all
2
3
4
5
Perfectly
17
Q

Your client, a college student from State A, hit another car driven by a resident from State B when the client was traveling through State B. The State B driver brought an action in State B state court against the client, who has limited financial resources. You are considering filing a motion to dismiss on behalf of the client on the grounds that the State B court lacks personal jurisdiction.

What is the best argument to support the motion to dismiss?

A The client did not take actions to purposely avail herself of State B simply by driving through State B.

B It is unfair, inconvenient, and highly unreasonable to require a college student with limited financial resources to defend the action in State B for financial reasons.

C The interstate judicial system’s interest in obtaining the most efficient resolution of the controversy is violated by requiring the client to defend the action in the forum state.

D State B is not the proper venue to file this claim.

A

B It is unfair, inconvenient, and highly unreasonable to require a college student with limited financial resources to defend the action in State B for financial reasons.

The best argument the client has to defend a motion to dismiss for lack of personal jurisdiction is that the fairness prong of the constitutional minimum contacts test is not met. In addition to sufficient minimum contacts with the forum state, personal jurisdiction must not offend “traditional notions of fair play and substantial justice.” The Supreme Court in International Shoe listed several factors relevant to assess the fairness factor, including the burden on the defendant in terms of convenience in defending the action. Here, the best argument out of the four choices listed is that it would be unfair, inconvenient, and highly unreasonable for a college student from State A with limited financial resources to defend the action in State B. Note that this still may be a losing argument, as this argument will not prevail unless the burden to the defendant is “so gravely difficult and inconvenient that a party is unfairly put at a severe disadvantage in comparison to his opponent.” However, this is still the best argument to support the motion to dismiss for lack of personal jurisdiction. (A) is wrong because, to establish the contacts prong of the minimum contacts test, the defendant’s contact with the forum state must result in purposeful availment of the state, meaning the defendant took actions that were purposely directed toward the forum state and from which the defendant derived the benefits and protections of the state’s laws, such as using the roads in the state. Here, driving through State B was sufficient because it was a purposeful activity and she was protected by State B’s police and traffic laws while doing so. (C) is wrong because, although considering the interest in obtaining the most efficient resolution of the controversy is one of the factors the Supreme Court listed in assessing the fairness prong, it is inapplicable to the facts. Here, litigation in either State B or State A would likely be equally efficient. (D) is wrong because State B clearly is a proper venue, as it is the judicial district where the claim arose. Moreover, the question stated that the motion to dismiss was based on lack of personal jurisdiction, not lack of venue.

How well did you know this?
1
Not at all
2
3
4
5
Perfectly
18
Q

While traveling on a commercial bus line, a passenger was injured when some luggage fell on him. As required by applicable state law, the bus company’s in-house attorney conducted an investigation and filed the required report with the state transportation department. The passenger subsequently filed a civil action against the bus company in federal district court, seeking compensatory damages for the injuries he suffered. During discovery, the passenger’s lawyer served on the bus company a request for production of documents, including a request for the report that the bus company filed with the state. The bus company objected to the request for the report and refused to produce it on the grounds that the report was privileged and protected from discovery under the work product doctrine. It did, however, produce other documents that were requested. The passenger then filed a motion to compel production of the report.

If the court finds that the bus company’s claims of privilege and work product were not substantially justified, what orders must the court make relating to the passenger’s request for production of the report?

A The court must sanction the bus company for improperly obstructing discovery by entering a default judgment against the bus company.

B If the passenger’s attorney has attempted in good faith to resolve the dispute, the court must order the bus company to produce the report and to pay the passenger’s reasonable costs in making the motion.

C Regardless of whether the passenger’s attorney has attempted in good faith to resolve the dispute, the court must order the bus company to produce the report and to pay the passenger’s reasonable costs in making the motion.

D If the passenger’s attorney has attempted in good faith to resolve the dispute, the court must order the bus company to produce the report, but need not order the company to pay the passenger’s reasonable costs in making the motion.

A

B If the passenger’s attorney has attempted in good faith to resolve the dispute, the court must order the bus company to produce the report and to pay the passenger’s reasonable costs in making the motion.

If the passenger’s attorney has attempted in good faith to resolve the dispute, the court must order the bus company to produce the report and to pay the passenger’s reasonable costs in making the motion, including attorney’s fees. Costs will be awarded unless: (i) the movant filed the motion before attempting in good faith to obtain disclosure or discovery without court action; (ii) the opposing party’s nondisclosure, response, or objection was substantially justified; or (iii) other circumstances exist that make an award of expenses unjustified. [Fed R. Civ. P. 37(a)] None of these exceptions apply. (A) is incorrect because default judgments are possible sanctions for a party who fails to comply with an order to provide discovery. (C) is incorrect because an attempt to resolve the discovery dispute without court intervention is required before an award of reasonable costs may be made. (D) is incorrect because reasonable costs must be awarded absent the exceptions noted above, which do not appear applicable here. [Fed. R. Civ. P. 37(a)(5)(A)]

How well did you know this?
1
Not at all
2
3
4
5
Perfectly
19
Q

A witness who was unable to speak was an eyewitness to a robbery in the park. He went to the police station shortly after the crime was committed, and when asked who committed the robbery, pointed to the defendant in a police lineup in which there were several men who were dressed like the defendant and were the defendant’s size. The defendant was then indicted for robbery. The witness died before trial, and the sergeant in charge of the lineup desires to testify at the defendant’s trial that the witness pointed to the defendant at the lineup.

Is his testimony admissible?

A) Yes, as evidence of prior identification.

B) Yes, to prove the defendant committed the crime.

C) No, because the sergeant’s testimony is hearsay.

D) No, because the defendant’s counsel was not present at the lineup.

A

C) No, because the sergeant’s testimony is hearsay.

The testimony is inadmissible. For purposes of the hearsay rule, a statement is defined by Federal Rule 801(a) as an oral or written assertion, or nonverbal conduct which is intended as an assertion. In this case, the witness’s pointing at the defendant is a statement because it was intended by the witness to assert that the defendant was the criminal. The sergeant’s testimony concerning the witness’s identification is testimony concerning an out-of-court statement and is therefore hearsay. Because the statement comes within no exception to the hearsay rule, it is inadmissible. The evidence is not evidence of a prior identification because the witness is not present on the witness stand. (A) is incorrect. Evidence of a prior identification is nonhearsay under Federal Rule 801(d)(1)(c) in only a very narrow range of cases. For a prior identification to be nonhearsay, a witness on the stand must have made a prior statement identifying the defendant as someone he perceived earlier. The offered evidence does not come within that definition of nonhearsay because the witness is not present on the witness stand. The sergeant’s testimony of the witness’s assertive conduct is inadmissible hearsay. (B) is incorrect. The evidence is inadmissible because it is hearsay and comes within no hearsay exception. The evidence is not evidence of a prior identification because the witness is not present on the witness stand to testify to the prior identification. (D) is incorrect. The choice correctly states that the evidence is inadmissible. But it is inadmissible because it is hearsay, not because the defendant’s counsel was not present at the lineup. Under constitutional principles, the defendant’s counsel would not need to be present at a preindictment lineup such as this in order to have the prior identification be admitted at trial.

How well did you know this?
1
Not at all
2
3
4
5
Perfectly
20
Q

The federal government has complete jurisdiction over certain parkland located within a state. To conserve the wildlife inhabiting that land, the federal government enacted a statute forbidding all hunting of animals in the federal park. That statute also forbids the hunting of animals that have left the federal park and have entered the state. A hunter has a state hunting license, authorizing him to hunt deer anywhere in the state. On land within the state located adjacent to the federal park, the hunter shot a deer he knew had recently left the federal land.

If the hunter is prosecuted for violating the federal hunting law, what is the strongest ground supporting the constitutionality of the federal law forbidding the hunting of wild animals that wander off of federal property?

A) This law is a necessary and proper means of protecting United States property.

B) The animals are moving in the stream of interstate commerce.

C) The police powers of the federal government encompass protection of wild animals.

D) Shooting wild animals is a privilege, not a right.

A

A) This law is a necessary and proper means of protecting United States property.

The strongest ground for supporting this provision is the Necessary and Proper Clause. Congress has the authority to legislate to protect federal parklands. The federal law safeguards wild animals that wander off federal parklands. The Necessary and Proper Clause allows Congress to choose any means to carry out its constitutional powers. Thus, it may regulate hunting to protect the federal interest in safeguarding the wild animals. (B) is wrong because it rests on an unjustified factual statement, that the “animals are moving in the stream of interstate commerce.” There are no facts describing the movement patterns of the animals. (C) is wrong because there is no federal police power to protect wild animals. Generally, the federal government lacks the police power; the police power belongs to state and local governments. Although Congress possesses the police power in legislating for federal lands, this choice is not limited to those circumstances. It speaks broadly of a federal police power to protect wild animals, and none exists. (D) is wrong because the Supreme Court has discarded the distinction between rights and privileges as a constitutional principle. Moreover, it is a distinction used to determine when the government must provide procedural due process before deprivations of property or liberty. This question does not involve the Due Process Clause.

How well did you know this?
1
Not at all
2
3
4
5
Perfectly
21
Q

A writer registered under federal copyright law his copyright in certain song lyrics he wrote. The writer later entered into a contract with an advertiser in which the writer granted the advertiser a license to use the lyrics in radio advertisements. When the writer heard the advertisement using the lyrics, the writer was incensed at how the lyrics had been used. Believing that the advertiser had lied to him about how the lyrics would be used, the writer filed an action in federal district court claiming that the advertiser had made false representations that fraudulently induced the writer into entering the contract to license the lyrics. The writer is a citizen of State A. The advertiser is a partnership comprised of partners who are citizens of State A, State B, and State C. The partnership’s headquarters and most of its operations are in State B.

Does the federal court have subject matter jurisdiction over the action?

A) No, because the action does not arise under federal law and the parties are citizens of the same state.

B) Yes, because the plaintiff and defendant are citizens of different states.

C) Yes, because the action arises under federal law.

D) Yes, because the transaction involves interstate commerce.

A

A) No, because the action does not arise under federal law and the parties are citizens of the same state.

The federal court does not have jurisdiction because neither diversity of citizenship jurisdiction nor federal question jurisdiction exists. Diversity of citizenship jurisdiction is available when (i) there is complete diversity of citizenship, meaning that each plaintiff is a citizen of a different state from every defendant; and (ii) the amount in controversy exceeds $75,000. A natural person’s citizenship is the state that is the person’s domicile. A partnership is a citizen of each state of which its partners, both limited and general, are citizens. Here, the writer is a citizen of State A, and the advertiser’s partners are citizens of State A, State B, and State C. Given the State A-State A connection, complete diversity does not exist. Hence, (B) is an incorrect answer choice as to diversity jurisdiction. Federal question jurisdiction is available when the plaintiff, in his well-pleaded complaint, alleges a claim that arises under federal law. Anticipation of a federal defense or the fact that federal law is implicated by the plaintiff’s claim do not give rise to federal question jurisdiction; the plaintiff’s claim must arise under federal law. Here, although federal copyright law is peripherally involved, the writer’s cause of action is actually based on state contract law. As a result, no federal question has been presented by the writer’s complaint, making (C) incorrect. Note too that federal question jurisdiction does not have a complete diversity requirement, making (B) incorrect as to federal question jurisdiction. (D) is incorrect because federal question jurisdiction does not arise merely because interstate commerce is affected.

How well did you know this?
1
Not at all
2
3
4
5
Perfectly
22
Q

Under a state law, individuals who have been residents of the state for more than three years receive a substantial discount on tuition at state-owned colleges and universities. A woman who has lived in the state for two years applied to and was accepted by the most prestigious of the state-owned universities. When she learned that her tuition would be nearly double that of some classmates, she was furious and filed suit in federal court seeking a declaratory judgment requiring the university to allow her to pay the reduced rate. As a result of several unexpected and unusual delays, 15 months passed before the case came to trial. During that time, the woman remained in the same residence, and the school allowed her to defer enrollment. At the start of the trial, the state filed a proper motion to dismiss the case.

How should the court rule on the motion to dismiss?

A) The court should grant the motion because the case is not ripe since she never enrolled.

B) The court should grant the motion because the case is now moot.

C) The court should deny the motion because the claims of others remain viable.

D) The court should deny the motion because the controversy is capable of repetition but evading review.

A

B) The court should grant the motion because the case is now moot.

A federal court will not hear a case that has become moot; a real, live controversy must exist at all stages of review, not merely when the complaint is filed. Since the woman has now lived in the state for more than three years, she qualifies for the discounted tuition and her claim for relief is now moot. (A) is incorrect. Ripeness and mootness are related concepts in that the court will not hear a case unless there is a live controversy. Ripeness bars consideration of claims before they have been developed; mootness bars their consideration after they have been resolved. When she brought the claim, the issues were fit for a judicial decision, and since the tuition law seems to have been enforced, the plaintiff would have suffered substantial hardship (that is, the higher tuition rate) in the absence of review. She did not have to actually incur the cost of tuition to bring the claim for relief. (C) is incorrect. It is true that a class representative may continue to pursue a class action even though the representative’s own claim has become moot, as long as the claims of others in the class are still viable, but this was an individual claim for individualized relief so that exception does not apply. (D) is incorrect. When there is a reasonable expectation that the same complaining party will be subjected to the same action again and would again be unable to resolve the issue because of the short duration of the action (that is, when the controversy is capable of repetition yet evading review), the controversy will not be deemed moot. This exception doesn’t apply to these facts for two reasons. First, the exception focuses on cases involving “the same complaining party”; here, the woman has now been a resident for more than three years, so she is no longer subject to the higher tuition rate (that is, her claim isn’t “capable of repetition” because she’d get the discounted rate). Second, the facts indicate that it took 15 months for the case to come to trial because of unexpected and unusual delays (that is, it doesn’t “evade review”). If the woman moved away for a year, returned as a new resident, and again complained about the tuition law, her claim could be resolved before it became moot.

How well did you know this?
1
Not at all
2
3
4
5
Perfectly
23
Q

A landlord entered into a written four-year lease with a tenant for an apartment in the landlord’s apartment house. The tenant’s lease, and all leases in the apartment house, prohibited the playing of musical instruments between 10 p.m. and 8 a.m. The lease required the tenant to pay the rent on a monthly basis. Two years into the lease, the tenant assigned the lease to a nurse with the landlord’s permission. The nurse then assigned the lease to his brother with the landlord’s permission. The brother went into possession. A neighboring tenant in the same apartment house insisted upon playing a trumpet in a loud manner between 2 a.m. and 4 a.m. The brother complained to the landlord without success. Unable to sleep each night, the brother abandoned his apartment after occupying it for two months.

If the landlord sues the nurse for the rent due during the period after the nurse’s brother left, what would be the nurse’s best defense?

A) A breach of the landlord’s covenant of quiet enjoyment.

B) Estoppel, because the landlord consented to a further assignment.

C) Lack of privity of estate.

D) Constructive eviction.

A

C) Lack of privity of estate.

The nurse is only an assignee of the tenant. The nurse is not directly contractually obligated to the landlord. Once an assignee of the nurse goes into possession of the premises (especially with the consent of the landlord), the privity of estate between the landlord and the nurse, the prior assignee, is terminated. An assignee has no contractual obligation under the original lease and is only liable for rent during the period of his estate, i.e., his possession. Thus, in this case, there is no longer any privity of estate between the nurse and the landlord, and the landlord cannot sue the nurse on the rent covenant. (A) is incorrect. The covenant of quiet enjoyment was not breached here because no one with superior title interfered with possession of the apartment. Moreover, the duty to provide quiet enjoyment is owed by the landlord only to the tenant and his successors in interest (i.e., those in privity of estate). Here, there is no privity of estate between the landlord and the nurse because the nurse is an assignee who is no longer in possession at the time of his brother’s abandonment, and that is the nurse’s best defense. (B) is incorrect. The defense of estoppel may be invoked where a party, acting in good faith, has been induced by the conduct of the adverse party to do something that it otherwise would not have done, that resulted in his harm, and that the adverse party had cause to know would so result in harm. These facts do not support the use of the theory of estoppel to avoid the application of landlord-tenant law, because the landlord has made no representations which were relied upon by the nurse to his detriment. (D) is incorrect, if only because there is a better defense available to the nurse. The defense of constructive eviction might be available to the nurse’s brother, who was in privity of estate with the landlord during the month in question. However, it is less clear that it would be derivatively available to the nurse. Moreover, it is a difficult factual defense. The landlord must be shown to have the ability to control the noise and to have failed to exercise that control. The noise must be of sufficient magnitude to deprive the nurse’s brother of the reasonable beneficial use of the apartment. The defense of lack of privity of estate is clear-cut and much more beneficial to the nurse.

How well did you know this?
1
Not at all
2
3
4
5
Perfectly
24
Q

A county ordinance permits individuals, businesses, and other entities to use animated signs that comply with specified rules if the sign is located on the same premises as the party expressing the message (that is, an on-premises sign). However, the same ordinance provides that animated signs cannot be used when the sign is not located on the same premises as the party expressing the message (that is, an off-premises sign). The city passed the ordinance because numerous studies show that off-premises animated signs result in a significant increase in car accidents, but on-premises animated signs do not result in a similar increase.

A toy store filed for a permit to erect an animated sign across the street from the store on property owned by a development company, noting on the application that the proposed sign complied with the rules for on-premises signs. When the application was rejected, the store filed suit claiming that the ordinance is an unconstitutional restriction on speech under the First Amendment as applied to states through the Fourteenth Amendment. It is undisputed that the proposed sign is not misleading and does not promote illegal activities, and evidence shows that the toy store has not been treated differently than any other sign applicant.

Is the toy store likely to prevail in its claim?

A) No, because the ordinance is viewpoint-neutral and reasonably related to a legitimate government purpose.

B) No, because the ordinance is content-neutral, advances important government interests, and is narrowly tailored to further those interests.

C) Yes, because commercial speech is protected under the First Amendment as long as it is not misleading and does not promote illegal activities.

D) Yes, because the ordinance is content-based since the county can only determine whether a proposed sign is on- or off-premises by reading the sign.

A

B) No, because the ordinance is content-neutral, advances important government interests, and is narrowly tailored to further those interests.

The Supreme Court has held that an ordinance that distinguishes between on-premises and off-premises signs generally is content-neutral. The fact that some examination of the speech or expression on the sign is necessary to determine into which category the sign falls does not mean the ordinance involves content regulation. While content-based regulation of speech is subject to strict scrutiny, content-neutral speech regulations generally are subject to intermediate scrutiny-they will be upheld if the government can show that (i) they advance important interests unrelated to the suppression of speech, and (ii) they do not burden substantially more speech than necessary or are narrowly tailored to further those interests. Here, the ordinance appears to be narrowly tailored to an important safety interest as it applies only to a set of signs that studies show are particularly dangerous. Notably, the statute doesn’t prohibit all off-premises signs and doesn’t prohibit all animated signs. (Note that the general standard for content-neutral restrictions of speech applies rather than the commercial speech standard because the ordinance is not restricted to signs carrying commercial messages.) (A) is incorrect. The stated test-that the regulation is viewpoint-neutral and reasonably related to a legitimate government purpose-applies to regulations of speech on government-owned nonpublic or limited public forums. Here, the proposed sign would be erected on privately owned land. Therefore, forum analysis does not apply. (C) is incorrect. As an initial matter, as discussed above, the ordinance is not limited to commercial signs. Further, while it is true that the First Amendment protects some commercial speech, that protection is not unlimited, so this choice would be incomplete even if the ordinance applied only to commercial messages. A regulation of commercial speech that concerns a lawful activity and is not misleading or fraudulent will be valid if it (i) serves a substantial government interest, (ii) directly advances the asserted interest, and (iii) is narrowly tailored to serve the substantial interest. Note that the last prong of this test does not require that the least restrictive means be used; rather, there must be a reasonable fit between the legislation’s end and the means chosen. (D) is incorrect. As discussed above, the Court has specifically held that the distinction between on- and off-premises signs is not content-based. The fact that someone may need to read the sign to determine which set of rules apply does not make it a content-based restriction.

How well did you know this?
1
Not at all
2
3
4
5
Perfectly
25
After having much too much to drink at the corner bar, the defendant decided it was time to go home. However, because of her intoxication, she did not notice that she grabbed an expensive and distinctive fur coat instead of her inexpensive cloth coat from the bar's coat room. The owner of the fur coat, who noticed the defendant staggering out of the bar and down the street, ran out of the bar and shouted for the defendant to stop. A police officer on foot patrol heard the shout and immediately apprehended the defendant. If charged with larceny of the fur coat, should the defendant be found guilty? A) Yes, because her mistake was unreasonable. B) Yes, because voluntary intoxication is not a defense to a charge of larceny. C) No, because the defendant did not reach a place of temporary safety and thus did not complete the crime. D) No, because she was too intoxicated to realize the property did not belong to her.
D) No, because she was too intoxicated to realize the property did not belong to her. The defendant should be found not guilty. Larceny is a specific intent crime that requires that the defendant intend to permanently deprive the owner of his property. If the defendant can show that she really believed the property was her own, there is no intent to deprive the owner of his property, and therefore the defendant would not be guilty of larceny. This is true whether or not the mistake was reasonable or whether the mistake was attributable to intoxication. In the instant case, the defendant did not realize that she had taken another's coat, and thus did not have the specific intent to commit larceny. (A) is incorrect. A mistake of fact that negates the state of mind requirement for specific intent crimes need not be reasonable. Any mistake of fact, reasonable or unreasonable, that negates specific intent would be a defense. For much the same reason, (B) is incorrect. For specific intent crimes, intoxication may be a defense when it prevents the defendant from formulating the requisite state of mind. (C) is incorrect because it implies that the asportation requirement for larceny must be substantial. It need not be; any movement of the property is sufficient to satisfy the requirement. Here, the defendant walked out of the bar with the fur coat, which is a sufficient asportation. Exam Note: Reaching a place of temporary safety may be important in a question involving killings committed during the course of a felony because it may cut off liability; it has nothing to do with the asportation requirement for larceny.
26
An owner of land who was also a home contractor agreed in writing with a buyer to build a house to the buyer's specifications on his (the contractor's) land and then sell the house and lot to the buyer. The contract provided that the house was to be completed nine months later, by March 1, with full payment due at that time. On March 1, due to delays in the delivery of materials, the contractor needed 20 more days to finish construction. On March 5, after discovering that the house had not yet been completed, the buyer notified the contractor in writing of her election to cancel the contract because of the contractor's failure to deliver the house by March 1. The contractor responded that, due to an unanticipated strike at his supplier's company, performance had been unforeseeably delayed and that the house would be ready by March 20. The buyer responded that she would no longer accept delivery of the house and land. The contractor then brought an action to recover damages for breach of contract. Who will prevail? A) The buyer, because the express date listed in the contract indicates that time was of the essence. B) The buyer, because delivery by March 1 was a condition precedent to the buyer's performance. C) The contractor, because the strike was truly an unforeseeable intervening event. D) The contractor, because there was no material breach.
D) The contractor, because there was no material breach. The contractor will prevail because he did not materially breach the contract. The failure to perform on time is a breach of contract, but in this case, it was a minor breach. Unless the nature of the contract is such as to make performance on the exact day agreed upon of vital importance, or the contract provides that time is of the essence, failure to perform at the stated time is not a material breach. Here, the home was nearly complete, and the delay was relatively short. The contract did not specify that time was of the essence; thus, the breach was minor. The remedy for a minor breach is damages; the aggrieved party is not relieved of her duty to perform. (A) is incorrect because merely stating a date for performance does not indicate that time is of the essence. There must be some explicit statement indicating that time is of the essence. (B) overlooks the fact that, although the delivery on March 1 is a condition precedent to the buyer's duty to pay, the condition is excused by substantial performance. The test for whether a party has substantially performed is the same as the one for assessing whether a breach is minor or material. Here the breach is minor, the contractor substantially performed, and the condition is excused. (C) is incorrect because an unforeseeable event does not discharge a party's duty to perform. A strike at the contractor's supplier does not rise to the level of impossibility or impracticability, which would discharge his duty to perform. The contractor could have procured the supplies elsewhere.
27
A homeowner agreed to sell his home to an accountant. He gave the accountant a general warranty deed and the accountant gave him $86,000, his asking price. The deed was recorded. A few years later, the accountant sold the property to a doctor, who paid her $125,000 for the property. To save on attorney's fees, the accountant went to her local office supply store to purchase a general warranty deed form. The store was out of those forms, but the clerk suggested that she use the form labeled "Quitclaim Deed," asserting that it would transfer the title just as well as the other form. The accountant purchased the form and filled in the blanks with the appropriate information she copied from her old deed. The doctor did not hire a lawyer to represent him in the purchase of the house. The doctor accepted the deed from the accountant and gave her $125,000. Soon after the doctor moved into the house, it was discovered that the homeowner's title was not good. The true owner now demands that the doctor vacate. Title is judicially determined to be with the true owner, and the doctor is forced out. Does the doctor have any action against the homeowner or the accountant based on any covenant for title? A) Yes, the doctor can sue both the homeowner and the accountant. B) Yes, the doctor can sue the accountant, but not the homeowner. C) Yes, the doctor can sue the homeowner, but not the accountant. D) No, the doctor can sue neither the homeowner nor the accountant.
C) Yes, the doctor can sue the homeowner, but not the accountant. The doctor can sue the homeowner but not the accountant. The homeowner gave covenants for title to the accountant when he gave her a general warranty deed. With a general warranty deed, the grantor covenants against title defects created both by themselves and by all prior titleholders. These warranties runs with the land and can be enforced by any subsequent purchaser, such as the doctor here. The doctor cannot sue the accountant because she gave only a quitclaim deed, which contains no covenants for title. (A) and (B) are incorrect because, as explained, the doctor cannot sue the accountant. (D) is incorrect because, as explained, the doctor can sue the homeowner because the homeowner gave the accountant a general warranty deed which covenants against any title defects the grantor (or their predecessors) may have created.
28
A defendant was charged with murdering his cousin, whose body was found near the defendant's office. The prosecution alleged that the cousin, who lived in another state, confronted the defendant to tell him about an affair that the cousin was having with the defendant's wife. The defendant denied having even seen the cousin for several weeks before he was murdered. The prosecution wishes to introduce a properly authenticated letter from the cousin to the defendant's wife, dated the day before the cousin's death, that stated: "I am going to go to your husband's office and tell him face-to-face that he must let us be together." Defense counsel objects. How should the judge rule on the objection? A) Overruled, because the statement in the letter falls under the present state of mind exception to the hearsay rule. B) Overruled, because the statement in the letter is being offered to show the defendant's motive for murder rather than for the truth of the matter asserted. C) Granted, because the statement in the letter is hearsay and does not fall within any exception. D) Granted, because the cousin's state of mind is not in issue.
A) Overruled, because the statement in the letter falls under the present state of mind exception to the hearsay rule. The judge should overrule the objection because the statement falls within the exception for statements of present state of mind. Under that exception, declarations of existing state of mind are admissible as circumstantial evidence to show subsequent acts of the declarant in conformity with that intent. Here, an issue in this case is whether the cousin confronted the defendant at his office, as the prosecution alleges. His statement that he was intending to do so is circumstantial evidence that he did so. It should therefore be admissible under the present state of mind exception. (B) is incorrect. While the cousin's confrontation with the defendant may have triggered the murder, the prosecution is offering the statement in the letter for its truth (i.e., that the cousin planned to confront the defendant and subsequently did so), in large part to rebut the defendant's assertion that he had not seen the cousin in several weeks. (C) is incorrect because, as discussed above, the statement falls within the hearsay exception for statements of present state of mind. (D) is incorrect because, while the cousin's state of mind is not directly in issue, it is relevant to show that he acted in conformity with that state of mind.
29
The neighbors to the north and south of a poorly maintained house, a mechanic and a doctor, decided to "send a message" by placing a smoke bomb on the owner's front porch. The doctor and mechanic consulted with another neighbor, an army veteran who was known to be an explosives expert, to help them. The army veteran agreed, but unbeknownst to the two neighbors, the army veteran manufactured a bomb containing a lethal quantity of explosives because he wanted to make sure the owner had a good scare. He gave the bomb to the mechanic and doctor, along with a timed fuse. In the middle of the night, the mechanic set the bomb on the owner's porch. A few minutes later, a huge explosion rocked the neighborhood. Although nobody was injured, the blast blew away the front of the owner's house. Who, of the following, is likely to be found guilty of attempted murder? A) The doctor, mechanic, and the army veteran. B) The army veteran and the mechanic, but not the doctor. C) The army veteran, but not the doctor or the mechanic. D) Neither the mechanic nor the doctor nor the army veteran.
D) Neither the mechanic nor the doctor nor the army veteran. None of these people would be guilty of attempted murder because, on these facts, they did not possess the specific intent to kill necessary for the crime of attempted murder. At common law, all attempts were specific intent crimes. Thus, before a criminal conviction for attempt would lie, it had to be established that the actor had the specific intent to engage in the behavior or to cause the harm prohibited by the criminal statute that the actor was charged with attempting to violate. Thus, before someone could be guilty of attempted murder, he had to possess the specific intent to kill. In this question, it is clear that the mechanic and the doctor did not have the specific intent to kill. Their intent was only to place a smoke bomb on the owner's porch. The situation with respect to the army veteran's intent is not quite as clear, but a fair reading of the facts indicates that he did not have a specific intent to kill. It is true that he made a bomb containing a lethal quantity of explosives, but the question goes on to state that he only intended to scare the slovenly owner. Had someone died in the blast, the army veteran could be found guilty of manslaughter due to his recklessness, or perhaps he could be found guilty of murder under the depraved heart theory. However, since no one died, he will not be guilty of attempted murder because there was no specific intent to kill. Thus (D) is correct, and (A), (B), and (C) are wrong.
30
The owner of a self-propelled riding mower had just started the engine to mow his front lawn when the clutch of the mower suddenly engaged, causing it to lurch forward rapidly and throw him off. By the time the owner caught up with the mower, it had started into the street. A motorist swerved to avoid the mower and struck a tree on the opposite side of the street. An investigation revealed that the sudden shift of the clutch was caused by a defective gear in the transmission. If the motorist brings a negligence action for personal injuries and property damage against the owner, will she prevail? A) No, because the owner was physically unable to react swiftly enough to prevent the harm. B) No, because the manufacturer of the mower is liable for the defective gear in the mower. C) Yes, because her damages were caused by the owner's operation of a dangerously defective piece of machinery. D) Yes, because a landowner owes a duty to passersby to exercise reasonable care in activities on his land.
A) No, because the owner was physically unable to react swiftly enough to prevent the harm. The motorist will not prevail under these facts because the owner does not appear to have breached his duty of care to her. A prima facie case for negligence consists of: (i) a duty on the part of the defendant to conform to a specific standard of conduct for the protection of the plaintiff against an unreasonable risk of injury; (ii) breach of that duty by the defendant; (iii) such breach being the actual and proximate cause of the plaintiff's injury; and (iv) damage to the plaintiff's person or property. While the owner owed a duty of care to the motorist because she was passing by his property while he was operating his riding mower, the facts do not suggest a breach of duty. Immediately upon starting the mower, he was thrown off by its sudden forward movement and could not catch up to it before it entered the street. Absent the owner's breach of duty, the motorist cannot recover. (B) is incorrect because the fact that the manufacturer of the mower could be liable to the motorist because of the defective gear in the mower does not preclude the owner from also being liable if he had operated the mower negligently. Here, however, nothing suggests that he was negligent in starting the mower or failing to discover or guard against the defect, and choice (A) addresses his conduct while operating the mower. Hence, (A) is a better choice than (B) because it addresses the issue of the owner's liability. (C) is incorrect because the motorist must prove that the owner knew or should have known of the defect and was therefore negligent in operating the mower while it was in such a defective condition. Merely showing the defective condition of the mower, without more, will not suffice to impose liability on the owner, and nothing in the facts indicates that the owner knew of the defect. (D) is incorrect even though a landowner does owe a duty of reasonable care to passersby, such as the motorist. As discussed above, nothing in the facts indicates that the owner breached that duty.
31
For many years, a farmer has grown and harvested a sizeable wheat crop on his land. On May 1, the farmer entered into an agreement with a bakery to supply the bakery with 10,000 bushels of wheat at $15 a bushel. The agreement called for the farmer to deliver the bushels to the bakery's plant on or before September 1. In mid-July, it became apparent that the year's wheat crop grown in the United States was going to be substantially smaller than originally anticipated. As a result, the price of wheat on August 1 rose to $35 a bushel. On August 10, the farmer informed the bakery that he would be unable to deliver any bushels to the bakery on September 1 because he had sold his entire harvested crop to other consumers. If the bakery sues the farmer for breach of contract, will the bakery prevail? A) Yes, as long as it files suit on or after September 1. B) Yes, regardless of whether it sues before or after September 1. C) No, because the sharp increase in the price results in a commercial frustration of the original deal, which excuses the farmer's performance. D) No, because the unforeseen rise in price is a substantial change of circumstances that excuses both parties.
B) Yes, regardless of whether it sues before or after September 1. The bakery will prevail regardless of when it sues. The farmer's statement of August 10 that he would be unable to deliver because he had sold all of his crop to others constitutes a repudiation of the contract, which entitles the bakery to sue at once rather than wait for the September 1 performance date. [See UCC §2-610] The bakery will prevail unless there have been some unanticipated circumstances that would excuse performance, but a rise in price due to poor crop performance is not sufficient to excuse performance under the doctrine of commercial impracticality. The farmer appears to have repudiated merely to make a greater profit. (A) is incorrect-because the farmer repudiated, the law does not require the bakery to wait until the due date for performance to arrive. It may sue at once. [See UCC §2-610] (C) is incorrect because the facts do not indicate that the farmer has been harmed at all by the price increase, or that he cannot perform the contract with the bakery. As a matter of fact, he is simply attempting to gain a windfall by repudiating the contract with the bakery to sell at a higher price. (D) is incorrect because the facts do not indicate that the circumstances that led to the poor crop were unforeseen. Crop performance is risky and variable, and a poor crop can certainly result in a price increase by virtue of the operation of the law of supply and demand. The farmer must be assumed to have anticipated such possibilities and taken such risks.
32
A corporation filed a breach of contract action against an individual in federal district court. Attached to the complaint was a written contract providing that the individual agreed to pay the corporation $100,000 over a period of years for specified services. The contract also contained the individual's signature. The individual told his lawyer that the signature was a forgery and that she had never signed or entered into the contract. On the basis of the individual's statement, the lawyer drafted, signed, and filed an answer. The answer denied the claim on a number of grounds and denied that the signature on the contract was that of the individual. The individual's lawyer later served on the corporation a request for production of documents. When the corporation objected to some of the requests, the lawyer filed a motion to compel production. Shortly before the hearing on the motion to compel, the individual advised the lawyer that the signature on the contract was in fact hers, but she and the lawyer agreed that she nonetheless should not be liable on the contract for other reasons. The lawyer thus continued to assert the motion to compel production. At the hearing on the motion, the lawyer referred to the complaint and the answer to justify the relevancy of the requests for production, but he did not mention the signature in any way. Has the lawyer violated Rule 11 of the Federal Rules of Civil Procedure? A) No, because when the lawyer signed the complaint, he had evidentiary support in the expected testimony of the individual who denied signing the contract. B) No, because the lawyer failed to state in the motion hearing that the signature on the contract was not that of the individual. C) Yes, because when the lawyer referred to the answer in the motion hearing, he renewed his certification that facts in the answer had evidentiary support at the time of the hearing. D) Yes, because the lawyer should not have signed the answer without having the individual sign a sworn statement that the signature on the contract was not hers.
C) Yes, because when the lawyer referred to the answer in the motion hearing, he renewed his certification that facts in the answer had evidentiary support at the time of the hearing. The lawyer has violated Rule 11. By "presenting" a document to the court, Rule 11 provides that a lawyer certifies that he believes the denials of factual contentions in the document are warranted on the evidence and that his belief is formed after a reasonable inquiry. One "presents" a document not only by signing or filing it, but also by "later advocating" it. Thus, Rule 11 imposes a continuing certification requirement, applicable any time a matter is presented to the court. At the time he signed the complaint, the lawyer believed there was evidence that the individual did not sign the contract based on the individual's own statement. When the lawyer referred to the answer in the motion hearing, the lawyer "presented" the answer to the court anew and renewed his certification, despite the fact that he never discussed the signature. At that time, the lawyer no longer believed that all the facts in the answer had evidentiary support. He knew that he lacked evidence to support the answer's denial that the signature on the contract was that of the individual. Thus, (C) is correct and (B) is incorrect. (A) is incorrect because it does not take into consideration the "later advocated" basis for presenting a document under Rule 11. (D) is incorrect because an attorney need not have his client sign a sworn statement for every fact that the client tells the attorney.
33
A music show promoter was handing out leaflets at a county fair, advertising a commercial country and western music concert scheduled at the fairgrounds two weeks after the county fair was due to end. The fair manager approached the promoter and politely asked him to stop distributing leaflets because it was in contravention of the fair's rules. The promoter refused to stop, and the manager summoned a county police officer. In the officer's presence, the manager again told the promoter to stop handing out the leaflets. The promoter ignored the manager again and continued to hand out leaflets, at which point the promoter was arrested for criminal trespass and charged in municipal court. At trial, the promoter defended against the charges by claiming a violation of his First Amendment rights. Which of the following, if true, is most damaging to the promoter's First Amendment claims? A) The place where the promoter distributed leaflets was private property leased to the county for holding the fair. B) The rules of the county fair clearly prohibit all leafleting or other solicitations on fair property, except at designated hours other than the time the promoter was engaged in distributing leaflets. C) The means of communication was printed leaflets rather than oral speech. D) The message on the leaflets promoted a commercial event.
B) The rules of the county fair clearly prohibit all leafleting or other solicitations on fair property, except at designated hours other than the time the promoter was engaged in distributing leaflets. The most damaging fact to the promoter's First Amendment claims is that the fair's rules clearly prohibit leafleting except at designated hours other than the time the promoter was distributing leaflets. The First Amendment freedom of speech is not absolute. To avoid chaos and to protect other governmental interests, government is allowed to adopt reasonable time, place, and manner regulations on speech in public forums and designated public forums. To avoid strict scrutiny and be upheld, such regulations must be content-neutral, be narrowly tailored to serve an important government interest, and leave open alternative channels of communication. The fact that the rule here allows solicitation at specified times indicates that it is narrowly tailored and leaves open alternative channels of communication. The rule also seems to be content-neutral and it serves the government's important interest of keeping the fair orderly. Therefore, the rule would probably be valid if (B) is true. (A) would not be damaging to the promoter because the First Amendment protects against infringement on speech by the government, regardless of whether the speech is on private or public property; the fact that the fairgrounds are private property does not preclude the property from being a public forum during the time that the county was leasing it and conducting a fair on it. Hence, the fairgrounds would be considered to be government property for purposes of application of the public forum rule. (C) is not damaging because the First Amendment protections extend to most forms of speech, including certain symbolic acts and the right not to speak. It certainly also extends to leafleting. (D) is not very damaging because the First Amendment protects commercial speech, although the standard for regulation differs from other speech-the government may ban false or misleading commercial speech and may regulate (including content regulation) other commercial speech with narrowly tailored regulations that directly advance a substantial government interest. The regulation here appears to meet this standard.
34
A small parish hired a contractor to build a school wing onto its church for a cost of $200,000 by August 31. The parties entered into a written contract that provided for five progress payments of $40,000 each at various stages of completion. On July 18, after the contractor had spent $160,000 on performance and received $120,000 in progress payments, he notified the parish that he was quitting the project because a more lucrative job came up. The parish hired some local laborers who did construction work for a living to finish the job by August 31 for $120,000, which was a reasonable price given the short notice. If the parish sues the contractor for breach of contract, and the contractor countersues to recover the $40,000 in costs that he has not been paid, what is the likely award? A) The parish can recover $120,000, the cost to complete construction of the wing. B) The parish can recover $40,000, the difference between the contract price and the total amount that the parish paid for construction of the wing. C) The contractor can recover $40,000, the difference between the amount it expended on performance and the amount it was paid, to prevent the parish's unjust enrichment. D) Neither party can recover anything, because the $40,000 extra that the parish had to pay to complete construction of the wing is offset by the $40,000 difference between the contractor's expenditures and the payments that the parish made to the contractor.
B) The parish can recover $40,000, the difference between the contract price and the total amount that the parish paid for construction of the wing. The parish can recover $40,000, which is the amount above the contract price that it cost the parish to get the wing completed. In construction contracts, when the builder breaches after partially performing, the standard measure of damages to which the owner is entitled is the cost of completion plus reasonable compensation for any delay in performance (unless completion would involve undue economic waste). In addition, most courts allow the builder to offset or recover for work performed to date if necessary to avoid the unjust enrichment of the owner. Here, the parish paid the contractor $120,000 and had to pay the laborers an additional $120,000. Thus, the total cost of the wing was $240,000, which is $40,000 more than the contract price. No offset will be allowed for the reasons discussed in (C) and (D) below. Hence, the parish can recover $40,000. (A) is incorrect because the cost of completion is determined from the perspective of the owner, i.e., how much additional money did the owner have to pay above the contract price to have construction completed? Here, the owner (the parish) paid $40,000 more to have the wing completed than it would have paid had the contract been performed to completion. The parish would be unjustly enriched if it could recover another $80,000 beyond the damages it suffered. (C) and (D) are incorrect because the parish was not unjustly enriched by the additional amount that the contractor expended in performance over the progress payments that it received. The parish still had to pay $40,000 more than the contract amount for completion of the wing because of the contractor's breach. Thus, the parish received no windfall as a result of the contractor's additional expenditures.
35
In an automobile collision case, the defendant's attorney called the defendant to the stand and asked, "Was the traffic light red, amber, or green when you entered the intersection?" The defendant replied, "It was green." Next, the defendant's attorney asked, "What did you tell the first police officer who arrived on the scene about the condition of the traffic light when you entered the intersection?" Before the defendant could reply with "I told him it was green," the plaintiff's attorney objected. How should the court respond to the objection? A) Sustain it, because the statement is hearsay not within any recognized exception to the hearsay rule. B) Sustain it, because the testimony is an irrelevant prior consistent statement. C) Overrule it, because the statement is made from personal knowledge and, therefore, is nonhearsay. D) Overrule it, because the defendant is in court and is subject to cross-examination by the plaintiff's attorney.
A) Sustain it, because the statement is hearsay not within any recognized exception to the hearsay rule. The statement is hearsay not within any exception. This question involves the proposed testimony of a witness (the defendant) about his prior out-of-court statement (telling the officer that the light was green) that is consistent with his in-court testimony. It appears that the prior statement is being offered to prove the truth of the matter asserted in the statement-that the light was green when the defendant entered the intersection. Thus, the proposed testimony is hearsay evidence and does not fall within any recognized exception to the hearsay rule. (B) is wrong because it states that the objection to the defendant's proposed testimony should be sustained because the prior consistent statement is irrelevant. Federal Rule 401 establishes a standard for relevance that is very easy to meet: relevant evidence is evidence having any tendency to make the existence of a consequential fact more probable or less probable than it would be without the evidence. The color of the traffic light at the time the defendant entered the intersection is clearly a consequential fact in the plaintiff's tort claim against the defendant. That the defendant immediately told the officer that the light was green would have some tendency to prove that the light was in fact green. The problem with the defendant's proposed testimony is not that it is substantively irrelevant, but rather that it is an unacceptable form of evidence, i.e., hearsay. (C) is wrong because, if an item of evidence is hearsay and not within any hearsay exception, it does not become admissible simply because it was made from personal knowledge. Declarants typically make hearsay statements based on personal knowledge; that does not transform the statements into nonhearsay or somehow render them admissible. (D) is incorrect; the fact that the defendant is in court and subject to cross-examination does not change the nature of the statement. Federal Rule 801(d)(l)(B) defines when a witness's prior consistent statement is not considered hearsay evidence: (i) when it is offered to rebut an express or implied charge that the witness is lying or exaggerating because of some motive (e.g., bias), provided the prior consistent statement was made before the onset of the alleged motive to lie or exaggerate; or (ii) when it is offered to rehabilitate a witness whose credibility has been impeached on some non-character ground other than a charge of recent motivation to lie or exaggerate (e.g., an alleged inconsistency or sensory deficiency). Here, there are no facts indicating that the defendant has been impeached by any method, so the hearsay exclusion for prior consistent statements does not apply. (Note also that the testimony may be inadmissible on the additional ground that it is improper bolstering of an unimpeached witness.)
36
Police officers suspected that an 18-year-old living with his mother was selling the illegal drug Ecstasy to his high school classmates. The police went to the young man's house without a search warrant. The young man was not at home, but his mother was, and she allowed the police officers entry. She also gave them permission to search the teen's bedroom. In the bedroom, the officers found a locked suitcase under the teen's bed. The mother said that it was her son's suitcase and she did not have a key for it. But she told them they could search it if they could get it open. The officers then broke the lock, opened the suitcase and found a substantial amount of what they believed and later tests revealed to be Ecstasy. The young man was charged with possession of Ecstasy with intent to distribute it. If the young man brings a pretrial motion to suppress the Ecstasy, how should the court rule on the motion? A) Grant it, because the mother was not informed of her right to refuse consent to the search. B) Grant it, because the mother's consent to search the suitcase was invalid. C) Deny it, because the mother had authority to consent to a search of anything within her home. D) Deny it, under the doctrine of in loco parentis.
B) Grant it, because the mother's consent to search the suitcase was invalid. The young man's motion should be granted. A search is valid if the police reasonably believe that they obtained valid consent for the search. Generally, a homeowner has authority to consent to a search of her own home, so long as she has apparent access to the place searched. Here, nothing indicates that the young man's bedroom door was locked. Thus, the mother had apparent authority to grant permission to search his room. However, it appears unreasonable for the police officers to believe that the mother had authority to consent to a search of the suitcase because it was locked and she did not have a key. The parent-child relationship does nothing to change this analysis. A parent of an adult teenager does not have automatic authority to search the adult teen's locked things (the rule probably is different for young children). (A) is incorrect. To be effective, a consent to search must be voluntary and given under no threat or compulsion. However, police officers are under no obligation to inform the defendant or other person of the right to refuse entry. Therefore, the fact that the officers did not inform the woman of her right to refuse entry does not make the search invalid. The issue is whether her consent was sufficient to allow the police to search the son's suitcase. (C) is incorrect because, as explained above, a homeowner does not necessarily have authority to consent to a search of everything in her home. Where it is apparent that she does not have access (for example, when something is locked and she does not have a key), police officers may not reasonably believe she may consent to the search of the locked area. (D) is incorrect. The doctrine of in loco parentis is a tort doctrine under which the state is charged with a parent's responsibilities over a child in its care. The doctrine does not apply here.
37
Late one night, a young couple were killed instantly when their car was struck by a speeding truck as the couple's car crossed an intersection with the light green in their favor. Several weeks later, a burglar awaiting trial on burglary charges asked a jail officer to let him speak with a highway patrol officer. When the highway patrol officer came to the cell, the burglar told him that he was the driver of the truck that had struck the car and had been speeding away from a burglary when the accident occurred. The burglar was charged with felony murder, on the theory that he had not yet reached a place of temporary safety when the accident occurred. At trial, the prosecution seeks to introduce the burglar's statements to the highway patrol officer regarding the events of the night of the accident. The burglar's attorney objects. Which of the following is the strongest argument for permitting the statements into evidence? A) The burglar had not been charged in connection with the auto accident at the time the statements were made to the highway patrol officer. B) The burglar made the statements spontaneously, without inducement or interrogation by the police. C) The highway patrol officer had no connection with the burglary investigation for which the burglar had been incarcerated. D) The burglar's statements were not the product of coercion by the police officers.
B) The burglar made the statements spontaneously, without inducement or interrogation by the police. The best argument is that the burglar made the statements spontaneously. Prior to a suspect's being charged with a crime, the Fifth Amendment privilege against compelled self-incrimination is the usual basis for ruling on the admissibility of a confession. [Miranda v. Arizona (1966)] Under Miranda, statements made during custodial interrogations are inadmissible unless the defendant is first warned of his right to remain silent and his right to an attorney. Thus, Miranda applies only when the defendant is in custody and only when the defendant's statements are the result of interrogation. Although almost any words or actions on the part of the police that they should know are reasonably likely to elicit an incriminating response qualify as interrogation, Miranda does not apply to spontaneous statements not made in response to interrogation. Here, the police did nothing to solicit the statement from the burglar; it was spontaneous. Thus, (B) is correct. (A) is incorrect because the defendant need not yet be charged for Miranda rights to apply as long as he is in custody (i.e., not free to leave). Being in jail on another charge (as the burglar was) satisfies the custody requirement. (C) is incorrect because the fact that the officer who took the burglar's admission had nothing to do with the investigation of the burglary does not alter the rules of Miranda-questioning that is totally unrelated to the matter for which the accused is in custody may still violate the accused's Miranda rights. (D) is incorrect. Due process requires that a confession be voluntary (i.e., not the product of police coercion). The Miranda rule, however, goes beyond voluntariness. It makes inadmissible all statements obtained without Miranda warnings or without a valid waiver of Miranda rights, not just statements actually coerced by the police.
38
A mechanic noticed that his neighbor had a "for sale" sign on his old car. The mechanic thought it would be fun to have an old car to fix up and customize, and he asked his neighbor how much he wanted for the car. The neighbor told him that he would sell the car for $400, and the mechanic responded that he would buy the car for $400. They agreed that the mechanic would come to the neighbor's house by 6 p.m. the next day with the money. At 9:15 the next morning, the neighbor called the mechanic and told him that when he agreed to the sale the preceding day, he forgot that he had just put two new tires on the car and, therefore, would need to be paid an extra $50. The mechanic agreed to bring $450 in cash to his neighbor's house at six o'clock that night. Is the mechanic legally bound to pay his neighbor the additional $50? A) Yes, because the original contract was not in writing. B) Yes, because the contract, as modified, does not exceed the minimum dollar amount required to invoke the Statute of Frauds. C) No, because no additional consideration was given for the oral modification. D) No, because contracts for the sale of goods can only be modified by merchants.
B) Yes, because the contract, as modified, does not exceed the minimum dollar amount required to invoke the Statute of Frauds. The mechanic is bound to pay the neighbor the additional $50. A contract for the sale of goods may be modified without consideration to support the modification if the modification was sought in good faith. No writing is required under the Statute of Frauds unless the contract, as modified, is within the Statute. Here, the parties formed a contract for the sale of goods (a car) when the mechanic agreed to buy the car for $400. (B) is correct because the contract as modified is under $500, so it is enforceable, even though it is not evidenced by a writing. (A) is incorrect because the fact that the original contract was not in writing is irrelevant. The original contract was for the sale of goods under $500 in value, and the Statute of Frauds does not apply. Thus, the mechanic could have enforced the original oral contract if he had not agreed to the modification. (C) is incorrect because under UCC section 2-209, no consideration is needed for the modification of a contract for the sale of goods. (D) is incorrect because all Article 2 contracts can be modified, if done in good faith.
39
The plaintiff in a negligence action in federal district court was injured when the defendant's car crashed into his. The defendant's friend was a passenger in the defendant's vehicle at the time of the accident. The defendant's attorney sent an email to the passenger asking the passenger to describe in detail what the passenger remembered about the accident and the events leading up to it. In response, the passenger sent the defendant's attorney an email describing the events. The plaintiff served on the defendant the following request for documents: "Please produce for inspection or copying any and all statements obtained by you or your attorney that relate in any way to the events and/or issues that are the subject of this legal action." Must the defendant produce the passenger's statement? A) Yes, because the statement is relevant to the parties' claims and defenses. B) Yes, because the work product doctrine does not apply to emails and other digitally stored information. C) No, because the statement is protected from discovery pursuant to the work product doctrine. D) No, because the statement is privileged.
C) No, because the statement is protected from discovery pursuant to the work product doctrine. The defendant does not need to produce the passenger's statement. Work product prepared in anticipation of litigation is discoverable only on a showing of substantial need and undue hardship in obtaining the substantial equivalent of the work product. Here, there is no indication that the passenger is unavailable or cannot recall the accident. Thus, it is unlikely that the plaintiff will be able to show substantial need and undue hardship in obtaining a statement from the passenger as to her recollection of the accident. As a result, the statement is protected from discovery. (Note that this analysis is applicable to the statement only; the existence of the passenger as a witness must be disclosed either as an initial disclosure-assuming the defendant is going to use passenger to support her claim or defense-or in response to a properly submitted interrogatory.) (A) is an overbroad statement of the requirements for discovery. Federal Rule 26(b)(3) specifically exempts documents prepared in the anticipation of litigation from discovery, and the passenger's statement falls into this category. (B) is an incorrect statement of the law; the fact that the statement was electronic does not prevent it from becoming protected under the work product doctrine. (D) is incorrect because the statement is not "privileged" per se (such as the doctor-patient evidentiary privilege), but rather is exempt from discovery under the work product doctrine under Federal Rule 26(b)(3). As described above, there are exceptions to the work product doctrine.
40
A city creates a program that provides a $5,000 annual grant to organizations offering free or low-cost adult day care if they meet certain requirements. A charity affiliated with a religious organization provides free adult day care that offers attendees the option of participating in religious activities. The head of the charity applied for a city grant. The grant was denied despite meeting all of the city's grant requirements. When the head of the charity asked the relevant city official why their grant application was denied, the official explained that the city could not provide direct financial aid to an organization affiliated with a religious institution. The charity brings a claim in federal court alleging that the denial violates the First Amendment. How is the court likely to rule? A) In favor of the charity, because the city may not limit eligibility for the grant to nonreligious organizations. B) In favor of the charity, because the religious activities are optional and therefore providing the grant would not result in excessive government entanglement with religion. C) In favor of the city, because providing the grant to a charity affiliated with a religious organization would violate the Establishment Clause. D) In favor of the city, because providing the grant would involve providing funding directly to the religiously affiliated charity rather than to religiously neutral benefit recipients.
A) In favor of the charity, because the city may not limit eligibility for the grant to nonreligious organizations. The charity is likely to succeed because the denial violates the Free Exercise Clause of the First Amendment. A state may not limit eligibility for a governmental benefit to nonreligious organizations. Even if a state has no obligation to create a benefit, once it has done so, it may not ban religious groups similarly situated to eligible secular parties from receiving the benefit merely because they are religious. Denying a generally available benefit to an otherwise eligible group based solely on their religious character violates the Free Exercise Clause because the program is not religiously neutral, as required under the Free Exercise Clause, and effectively penalizes the exercise of religion. Further, the Court has made it clear that treating religious organizations the same as secular ones-which here would mean providing the grant to the charity-does not violate the Establishment Clause. (B) is incorrect. First, the fact that the religious content is optional is irrelevant based on the facts. The charity was denied the grant because of its religious affiliation rather than because the content of its services. Further, the Court has moved away from an earlier test that for Establishment Clause violations that considered "excessive entanglement." Instead, the modern Establishment Clause inquiry focuses on avoiding coercion, pursuing neutrality, and considering history and the understanding of the framers of the Constitution. (C) is incorrect. As stated above, including the religiously affiliated charity in the generally available grant program would not violate the Establishment Clause. (D) is incorrect. It is true that the government may give aid in the form of financial assistance to a defined class of persons as long as the class is defined without reference to religion or religious criteria, and it is also true that a neutral benefit program is valid even if, as a result of the independent choices of private benefit recipients, it allows public funds to be directed to a religious organization (such as parents using vouchers to pay for tuition at a religious private school). However, those types of program are not the only types of programs involving government funds being directed to religious institutions that have been deemed constitutionally permissible. Here, denying the grant violates the Free Exercise Clause and providing it would not violate the Establishment Clause.
41
A man and woman who shared an apartment were arrested and charged with murder. The woman's attorney moved to have the defendants tried separately. The motion was granted. The woman was tried first and was acquitted of the crime. The man's trial began one month later. The man's attorney called a witness, who was now living with the man, to testify that, shortly after she was acquitted of murder, the woman had told the witness that she (the woman) had killed the victim because the victim owed her money, and that the man was not involved at all in the victim's death. Is the witness's testimony admissible? A) Yes, as a statement by a co-conspirator. B) Yes, as a declaration against penal interest. C) No, because it is hearsay not within any exception. D) No, because the woman is estopped from contradicting the jury verdict of acquittal.
C) No, because it is hearsay not within any exception. The witness's testimony would clearly qualify as hearsay. Because there is no applicable exception to the hearsay rule, his testimony is inadmissible. (A) is incorrect because the statement was not made during and in furtherance of the conspiracy, a requirement for admitting statements by a co-conspirator. (B) is incorrect because the woman has already been acquitted of the crime in question; hence, she is not subject to further prosecution. (D) is an incorrect statement of law.
42
A widget manufacturer entered into a written agreement with a retailer to sell the retailer 500 widgets for a total price of $10,000. Prior to the date set for execution of the contract, the price of the raw material essential to manufacture of widgets had soared because of a civil war in the country that produces 80% of the world supply of the material. The manufacturer informed the retailer that it would now cost $11,000 to manufacture the widgets and requested that the contract price be adjusted to $12,000 for the 500 widgets. The retailer agreed orally to pay the $12,000, but no written confirmation was exchanged between the parties. Shortly thereafter, the civil war ended and the raw material became available again at prewar price levels. The manufacturer shipped 500 widgets to the retailer. On receipt, the retailer sent the manufacturer a certified check in the amount of $10,000, marking it "payment in full." The manufacturer did not cash the check, but telephoned the retailer demanding an additional $2,000. The retailer refuses to pay the additional sum. May the manufacturer enforce his demand for an additional $2,000 in a court of law? A) No, because no new consideration was given for the modification. B) No, because of the Statute of Frauds. C) Yes, because the parol evidence rule allows evidence of a subsequent modification. D) Yes, because the modification was made in good faith.
B) No, because of the Statute of Frauds. The manufacturer will not be able to enforce his demand because the contract, as modified, is within the Statute of Frauds, and there is no writing evidencing the modification. Because the contract here is for the sale of goods, the UCC will govern. Under the UCC, a contract for the sale of goods priced at $500 or more must be evidenced by a writing signed by the party to be charged to be enforceable. A contract modification must be evidenced by such a signed writing if the contract, as modified, is within the Statute. Here, the contract as modified is well over $500 and the modification is not in writing; thus the modification is unenforceable. (A) is incorrect because a modification of a contract for a sale of goods is enforceable even though it is not supported by consideration (which is required under the common law), as long as the modification is sought in good faith. Here, the modification was sought in good faith (the civil war raised the price of the raw material needed to manufacture widgets, making the contract unprofitable). Therefore, had the modification complied with the Statute of Frauds, it would have been enforceable despite the lack of consideration.(C) is incorrect because even though the parol evidence rule does not bar evidence of a subsequent (as opposed to a prior or contemporaneous) expression, the modification here is unenforceable due to the Statute of Frauds. (D) is incorrect. because, as noted above, the modification did not comply with the Statute of Frauds. Although it is true that modifications without consideration are enforceable under the UCC if made in good faith, the manufacturer will be unable to enforce the modification because it is not evidenced by a signed writing.
43
A mother had her lawyer prepare and execute two notarized deeds: one conveying a 640-acre farm to her daughter and the other conveying a 590-acre farm to her son. The mother produced the deed to the daughter, who told her mother to keep the deed just in case the daughter misplaced it. The mother held on to the deed. The next day, the mother went to her lawyer's office to have the deeds recorded, but she forgot the daughter's deed at home. The mother gave the lawyer the son's deed and promised to bring the daughter's deed in the next day. Later that afternoon, the lawyer recorded the son's deed. That night, the mother suffered a stroke and died in her sleep. The daughter's deed was never recorded but was found in the mother's home after her death. In her will, the mother left all of her property to her daughter, her son, and the local animal shelter, in three equal shares. Aside from her modest home and some small bank accounts, the mother had no appreciable estate. In this jurisdiction, delivery is presumed if a deed is validly recorded. If the animal shelter challenges the deeds to the farms on grounds of lack of proper delivery, what is the likely result? A) The shelter will win on both farms. B) The shelter will lose on both farms. C) The shelter will win on the 640-acre farm but lose on the 590-acre farm. D) The shelter will win on the 590-acre farm but lose on the 640-acre farm.
B) The shelter will lose on both farms. The animal shelter will lose on both farms. As to the deed to the son conveying the 590-acre farm, delivery is presumed because the deed was validly recorded, even though the son did not physically receive it and does not seem to have known of its existence. As to the deed to the daughter conveying the 640-acre farm, the fact that she did not physically keep the deed when it was presented to her does not show lack of delivery under the circumstances. The key to delivery is the grantor's intent to pass all legal controls. Because the daughter simply asked her mother to safeguard the deed, we can presume delivery under these circumstances. But because the deed to the daughter was never recorded, we cannot rely on the rule that provided valid delivery in the son's case. (A) is incorrect. Both deliveries were valid. The son's deed was valid by reliance on the presumption that recordation shows a valid delivery, and the daughter's deed on the circumstances of the gift that show proper intent. (C) is incorrect. The animal shelter does lose on the 590-acre farm, but does not win on the 640-acre farm because the daughter's having her mother keep the deed does not show lack of delivery. (D) is incorrect. The animal shelter does lose on the 640-acre farm, but does not win on the 590-acre farm because the recordation of the son's deed gives a presumption of valid delivery.
44
A man and his friend were walking across a bridge on their way home from watching a wrestling event when they began to playfully reenact their favorite wrestling moves. As part of one of the wrestling moves, the man lightly shoved his friend. The friend exaggeratedly stumbled backward into the bridge's railing. Unbeknownst to the man and his friend, the bridge's railing was weak and in need of replacement, and the friend fell through the railing and into the water below. The friend died as a result of the fall. In a jurisdiction that follows common law, what is the most serious crime, if any, with which the man can be charged? A) No crime. B) Murder. C) Voluntary manslaughter. D) Involuntary manslaughter.
A) No crime. The man is not guilty of any crime because he lacked the necessary intent to kill or harm his friend. (B) is incorrect because the man did not have the requisite intent necessary to be charged with murder. At common law, murder is the unlawful killing of another human being with malice aforethought. Malice aforethought exists if the defendant has one of the following states of mind: (1) intent to kill, (2) intent to inflict great bodily injury, (3) reckless indifference to an unjustifiably high risk to human life, or (4) intent to commit a felony. Here, the man did not intend to kill or inflict great bodily harm on his friend or commit a felony, and the man's actions do not reflect a reckless indifference to an unjustifiably high risk to human life. The facts indicate the man lightly shoved his friend while they consensually reenacted wresting moves. The friend's exaggerated stumble is what ultimately caused the friend to fall to his death, and neither party was aware of the danger resulting from the bridge's faulty railing. (C) is incorrect because voluntary manslaughter is a killing that would otherwise be murder but for the existence of adequate provocation. Here, as stated above, the man would not be guilty of murder. Thus, the existence of mitigating factors, or provocation, is not required. (D) is incorrect because the man did not act with criminal negligence. Involuntary manslaughter is a death caused by criminal negligence. A person acts negligently when he fails to be aware of a substantial and unjustifiable risk and that failure constitutes a substantial deviation from the standard of care that a reasonable person would exercise under the circumstances. Here, a reasonable person likely would not expect that the man's light shove would cause his friend to fall through the bridge's railing and die.
45
Thirty years ago, a telephone company purchased an easement from an owner to install, inspect, repair, replace, and maintain underground transmission cables within a specified portion of the owner's property. The deed granting the easement was validly recorded, and the telephone company shortly thereafter laid underground long distance lines traversing the owner's land beneath the area described in the easement deed. Fifteen years later, the owner sold a 50-acre portion of his land to a buyer. This portion included part of the easement granted to the telephone company. The buyer erected a residence on the purchased land and planted 49 acres of walnut trees. He granted an easement to the telephone company to string aerial telephone wires. The underground easement was never disclosed. Last year, the telephone company informed the buyer that it would have to excavate underground cables and replace them with new ones. This was the first the buyer knew of the cables lying under his land. The cables, lying within the easement, are directly beneath a row of black walnut trees running the length of the buyer's property. Black walnuts have become extremely valuable, and the buyer estimates that their value is approximately $400,000. If the buyer seeks an injunction prohibiting the telephone company from undertaking the planned excavation, for which party should the trial court rule? A) The telephone company, because the benefits of installing the new cables outweigh the harm done to the buyer. B) The telephone company, because ownership of the easement permits it to make the excavation. C) The buyer, because the telephone company concealed the existence of the underground easement from the buyer when it acquired the overhead easement from him. D) The buyer, because the deed by which he purchased his property made no mention of the easement, and he had no actual knowledge of it.
B) The telephone company, because ownership of the easement permits it to make the excavation. The court should rule for the telephone company. This question turns simply on whether or not there was a valid easement. The easement granted by the owner to the telephone company 30 years ago was properly recorded. That alone would put subsequent purchasers of the property on constructive notice of the easement. The fact that the easement was not in the deed from the owner to the buyer is irrelevant. A valid easement passes with title whether or not it is mentioned in the conveyance. For the subsequent purchaser to avoid the easement, that person must have taken without notice of it. Although there was no actual notice of the easement, a proper title search would have disclosed the easement, and the buyer is bound as if he knew. (A) is incorrect. If an easement is valid, there is no need to balance the hardships in order to determine whether the holder of the easement may make proper use of it. An easement to lay cables underground reasonably carries with it the right to inspect those cables or make replacements as is necessary. Hardship to the holder of the servient estate is not relevant if the easement is proper and the excavation is considered reasonable use of that easement. (C) is incorrect because the telephone company was under no obligation to mention its underground cable easement, and its failure to do so is meaningless. (D) is incorrect. Valid easements are conveyed along with the servient estate, regardless of whether they are mentioned in the conveyance or not. Because the easement was recorded, a title search would have disclosed it, and thus the buyer will be held to have constructive knowledge of it.
46
A seller entered into a written land sale contract with a buyer on May 20, whereby the seller agreed to sell a home to the buyer for $60,000. The closing date was set at August 1. The buyer put up $6,000 as earnest money, as provided by the contract. The contract stated that if the buyer failed to perform by tendering the balance due on the house on August 1, the $6,000 could be treated as liquidated damages "at the option of the seller." On July 21, the week before the closing, the house burned to the ground because of a freak lightning strike during a thunderstorm. When August 1 arrived, the buyer refused to tender $54,000 to the seller. The buyer asked the seller for the refund of his earnest money because the house had been destroyed. The seller refused and filed suit, asking for specific performance. The buyer countersued, demanding refund of the $6,000 earnest money. How should the court rule on the suits? A) The court will order specific performance by the buyer. B) The court will order the seller to return the $6,000 to the buyer because of frustration of purpose of the contract. C) The court will award the seller $6,000 as liquidated damages. D) The court will award neither party the relief sought because of mutual mistake.
A) The court will order specific performance by the buyer. The court should order specific performance by the buyer. Because land is considered unique, specific performance is always appropriate for the enforcement of a valid land sale contract. This option is available to either the buyer or the seller. The contract gave the seller the option of using the liquidated damages provision if the seller wished, but the seller has sued for specific performance. Because that is appropriate, it will be granted. It is not of significance that the house burned to the ground. When a contract for the sale of land is signed, equitable conversion takes place and it is, for all practical purposes, the buyer's land and the buyer's risk. Here, the risk of loss shifted to the buyer upon the signing of the contract. (B) is incorrect because frustration of purpose is not applicable. The doctrine of equitable conversion shifted the risk of loss to the buyer when the contract of sale was signed. The fact that an improvement on the property (the house) is no longer present is not relevant to the grant of specific performance. (C) is incorrect. Based upon these facts, the seller could request as a remedy either specific performance or liquidated damages of $6,000. Because the seller elected the specific performance remedy, the court will not award liquidated damages. (D) is also incorrect because there are no facts which would indicate mutual mistake. The burning down of the house would not qualify.
47
A gardener entered into a contract with a landowner under which, for a monthly fee of $200, the gardener would provide the landowner with monthly gardening services for the calendar year. On June 1, the gardener told the landowner that she could no longer afford to work for $200 a month and demanded that she be paid $400 a month. The landowner refused, stating that he already had a valid contract with her and that he did not have to pay any more. The gardener again insisted that she would not work for less than $400. As a result, the gardener and the landowner agreed in writing on July 1 that the gardener would thereafter be paid $400 a month for the gardening services and that the term of the agreement would be extended through the end of next year. Is the gardener legally entitled to receive a monthly fee of $400 from the landowner after July 1 of this year? A) Yes, but only for the period from January 1 to December 31 of next year. B) Yes, because the gardener and the landowner mutually agreed to modify their original contract. C) No, because the gardener used economic duress in order to obtain the landowner's agreement to the modification. D) No, because the gardener was under a preexisting duty to perform the gardening services for $200 a month.
B) Yes, because the gardener and the landowner mutually agreed to modify their original contract. The gardener is entitled to receive the fee. The common law rule is that a promise to modify a contract must be supported by consideration. Here, the gardener agreed to work an extra year in return for the landowner's promise to pay extra money. In the absence of facts indicating duress or unconscionability (facts not present here), courts will not inquire into the adequacy of the consideration. There is no indication that the landowner had no "meaningful choice" but to accept the price increase, and because the landowner received a year's extension in exchange, there was adequate consideration to support a modification. (A) is incorrect because, as discussed above, the modification is valid and the price increase can take effect as agreed. (C) is incorrect for two reasons: First, economic duress generally is not a good defense to contract unless caused by the party seeking to enforce the contract. Second, as mentioned above, there are no facts indicating that the landowner had no "meaningful choice" but to accept the proposed modification; nothing indicates that the landowner could not have found another gardener or could not have sued the gardener for breach if she had not performed the original contract in the absence of modification. (D) is incorrect because the gardener did not merely promise to perform a preexisting duty in exchange for the promise to pay $200 a month more. She promised a year's extension of services, and this is additional and valuable consideration.
48
A farmer fleeing from a bull that had gotten out of its pen bolted for the nearest safety-the fence between the pen and a melon grower's melon patch. Leaping over, the farmer landed on and shattered several melons. Hearing the melee, the melon grower ran from a hiding place where he had been waiting to try to catch some children who had been stealing melons. He pointed his shotgun, which was loaded with rock salt, and exclaimed, "Stop or I'll shoot." Just then two juvenile melon thieves decided to make their escape and ran from where they had been crouching. The melon grower aimed his shotgun at them and prepared to fire but slipped, and the load of rock salt struck the farmer, whom the melon grower had recognized and had not intended to hit. Has the melon grower committed an assault against the farmer? A) Yes, because he actually pointed the shotgun at the farmer. B) Yes, if the farmer reasonably believed that the melon grower might shoot him. C) Yes, because assault is included within the tort of battery, and the melon grower committed a battery when he shot the farmer in the shoulder. D) No, because the shotgun was only loaded with rock salt.
B) Yes, if the farmer reasonably believed that the melon grower might shoot him. An assault is committed when the victim is put in reasonable apprehension of an offensive touching. If the farmer believed that the melon grower might shoot him, and the belief was objectively reasonable, then the melon grower's action amounts to assault. The conditional threat, "stop or I'll shoot," is sufficient for an assault. Because the melon grower was pointing a rifle at the farmer at the time the threat was made, the requirement of immediacy was fulfilled. The melon grower cannot claim defense of property because the farmer had a privilege based on private necessity to enter the melon grower's land. (B) is a better answer than (A), because the melon grower would only have committed an assault if the farmer had a reasonable apprehension of a harmful or offensive contact. (C) is a misstatement of the law. Assault is not a lesser included tort within battery. (D) is incorrect because what is contained in the shotgun is irrelevant. The gun created apprehension of a harmful or offensive contact.
49
An impatient driver who was fed up with jaywalking pedestrians drove straight at one of them, leaning on the horn and intending to make her jump. She did not hear him or change her pace, however, because her music player was turned to full volume. A bystander on the curb rushed out to pull her to safety. She tripped as she was being pulled to the curb, fracturing her kneecap. If the pedestrian sues the driver for assault, what will be the likely result? A The driver wins, because the pedestrian did not know at the time that she was in danger from the driver. B The driver wins, because he did not intend for the pedestrian to be injured by his conduct. C The pedestrian wins, because the driver intended to create in her an apprehension of immediate harmful contact. D The pedestrian wins, because the driver's conduct was a substantial factor in causing her injury.
A The driver wins, because the pedestrian did not know at the time that she was in danger from the driver. The driver is not liable for assault because he did not cause the pedestrian to reasonably apprehend an immediate harmful contact. The prima facie case for assault requires (i) an act by defendant causing a reasonable apprehension in plaintiff of immediate harmful or offensive contact to plaintiff's person, (ii) intent by defendant to bring about in plaintiff apprehension of that contact, and (iii) causation. For there to be apprehension, plaintiff must be aware of defendant's act at the time that it is occurring. Here, because the pedestrian was oblivious to the driver's attempt to scare her, the driver is not liable for assault. (B) is incorrect because whether the driver had an intent to injure the pedestrian is irrelevant for purposes of assault. (C) is incorrect. While the driver did have the intent to commit an assault, his act does not meet the requirements for the prima facie case because his act did not cause reasonable apprehension. (D) is incorrect even though the driver's conduct was a substantial factor in causing the pedestrian's injury (i.e., the causation element would have been satisfied if damages were required for assault). Because the driver did not cause an apprehension of contact on the pedestrian's part, the driver is not liable for assault.
50
Force may not be used by __________. A a property owner to defend property from tortious interference B a citizen in effecting a misdemeanor arrest C a landowner to regain real property after being tortiously dispossessed D an owner of chattel to recapture the chattel
C a landowner to regain real property after being tortiously dispossessed A landowner may not use force to regain real property after being tortiously dispossessed. Most states today do not allow resort to "self-help"; one who has been wrongfully excluded from possession of real property may bring an ejectment action or other summary procedure to recover possession. Hence, the owner who uses force to retake possession is liable for whatever injury she inflicts. (In former years, under the common law, a landowner tortiously dispossessed of real property could use reasonable force to regain possession, if she acted promptly upon discovery of the dispossession.) An owner of chattel may use force to recapture the chattel. An owner may use reasonable force to recapture a chattel when in "hot pursuit" of the tortfeasor. A demand for return of the chattel must be made before force is used, unless the demand would be futile or dangerous. However, force can be used only against the tortfeasor or a third party who knows that the chattel was tortiously obtained. If an innocent third party has obtained the chattel, the owner is no longer privileged to use force to effect a recapture of the chattel. A citizen may use force to effect a misdemeanor arrest. However, the citizen is allowed to use only the amount of force necessary to effect the arrest and never deadly force. A property owner may use force to defend the property from tortious interference. Although a property owner may use reasonable force to defend property, she may not use force that will cause death or serious bodily harm. Furthermore, one may not use indirect deadly force such as a trap, spring gun, or vicious dog when such force could not lawfully be directly used, e.g., against a mere trespasser.
51
A college student owned a very popular video game system that was out of stock in most stores. He agreed to let his friend use the system for a few days, on condition that he return the system by the weekend because the student was hosting a small party. On the morning of the party, the friend still had not returned the game system, so the student went to the friend's apartment and demanded it back. The friend refused, so the student grabbed the system and wrestled it out of the friend's hands. If the friend sues the student for battery, will he recover on this claim? A No, because the student used reasonable force in attempting to seize the game system. B No, unless the student proves that the friend's delay in the return of the game system was unreasonable. C Yes, because the student had originally agreed to lend the game system to the friend. D Yes, because the student had to give the friend a reasonable period of time after demand in which to return the game system.
C Yes, because the student had originally agreed to lend the game system to the friend. The friend will recover for battery because the student did not have the right to use force. The defense of recapture of chattels is limited by the circumstances of the original dispossession. When another's possession of the owner's chattel began lawfully, the owner may use only peaceful means to recover the chattel. Force may be used to recapture a chattel only when in "hot pursuit" of one who has obtained possession wrongfully (e.g., by theft). Here, the friend's initial possession of the game system was a bailment, because the student consented to his borrowing it. Thus, the student is not entitled to use force to recover it, and his wrestling it away from the friend constituted the requisite harmful or offensive contact to make the student liable for battery. (A) is incorrect. One who is entitled to use force to recapture chattels is only permitted to use reasonable force, but here the student is not entitled to use any force at all because the friend's initial possession of the game system was lawful. (B) is incorrect because it is not relevant whether the friend's delay in returning the game system was unreasonable; the student is not entitled to use force because he lent the system to the friend originally. (D) is incorrect because the requirement that a timely demand to return the chattel must precede the use of force applies only if the owner of the chattel is entitled to use force. Here, as discussed above, the student did not have the right to use force.
52
Which of the following persons is considered to be an invitee of the landowner? A A hiker hiking on the landowner's open land with permission. B A child accompanying a customer of the landowner. C A firefighter fighting a fire on the landowner's property. A firefighter fighting a fire on the landowner's property. - no response given D A customer of the landowner who goes through a door marked "employees only.
B A child accompanying a customer of the landowner. A child accompanying a customer of the landowner is considered an invitee because she came onto the property for a purpose connected to the business. Under traditional landowner liability rules, a landowner owes an invitee a general duty to use reasonable and ordinary care in keeping the property reasonably safe for the benefit of the invitee. This general duty includes the duties owed to licensees (to warn of nonobvious, dangerous conditions known to the landowner and to use ordinary care in active operations on the property). A landowner also owes invitees a duty to make reasonable inspections to discover dangerous conditions and make them safe. A customer of the landowner who goes through a door marked "employees only" is no longer an invitee. A person loses his status as an invitee if he exceeds the scope of the invitation-if he goes into a portion of the premises where his invitation cannot reasonably be said to extend. A firefighter fighting a fire on the landowner's property is not treated like an invitee. Under the "firefighter's rule," police officers and firefighters are generally treated like licensees, based on public policy or assumption of risk grounds. They cannot recover for a landowner's failure to inspect or repair dangerous conditions that are an inherent risk of their law enforcement or firefighting activity. A hiker on the landowner's open land is not considered an invitee. If an owner or occupier of open land permits the public to use the land for recreational purposes without charging a fee, the landowner is not liable for injuries suffered by a recreational user unless the landowner willfully and maliciously failed to guard against or warn of a dangerous condition or activity.
53
A boy was playing softball in a neighborhood park when a ball was hit over the fence and into a neighbor's yard. The boy knocked on the neighbor's door and obtained permission from her to retrieve the ball from her yard. As he bent to retrieve the ball in some bushes, the boy brushed against an exposed electric wire that was partially hidden by the bushes and received a severe electric shock and burns. The neighbor had failed to maintain the bushes, allowing them to become overgrown, and was not aware of the exposed wire. If the boy sues the neighbor in a jurisdiction that applies the traditional rules for landowners and possessors of land, what is the likely result? A The neighbor is liable because the boy entered with her permission. B The neighbor is liable because she failed to repair a dangerous condition on her property. C The neighbor is liable because she failed to reasonably inspect the property, which would have made her aware of the dangerous condition of the wire. D The neighbor is not liable because she did not know of the condition of the wire.
D The neighbor is not liable because she did not know of the condition of the wire. The neighbor is not liable because she did not know of the condition of the wire and the boy was a licensee. In jurisdictions that distinguish between invitees and licensees, a licensee is a person who enters land with the owner's permission, for his own purpose or business rather than for the owner's benefit. The owner or occupier of land owes the licensee a duty to warn of or make safe a dangerous condition known to the owner or occupier that creates an unreasonable risk of harm to the licensee and that the licensee is unlikely to discover. However, the owner or occupier has no duty to a licensee to inspect for defects nor to repair known defects. The boy was a licensee because he entered the neighbor's land with her permission for his own purpose (retrieving the ball) rather than for any benefit of the neighbor's. The exposed electric wire created an unreasonable risk of death or serious injury to the boy as he reached into the bushes. Because the neighbor did not know of the presence and condition of the wire, she had no knowledge of any risk of harm to the boy. Thus, no duty to warn the boy of the wire was triggered. (A) is incorrect because the mere fact that the neighbor gave the boy permission to enter her land will not subject her to liability for his injuries incurred thereon. The neighbor is not strictly liable for injuries to a licensee, but only for any injuries caused by a breach of her duty to warn of dangerous conditions known to her and that the licensee is unlikely to discover. (A) would impose liability even where the neighbor had no knowledge of the condition of the wire. (B) is incorrect because, as noted above, an owner of land does not owe a duty to a licensee to repair defects or dangerous conditions. Likewise, (C) is incorrect because the owner of land is not under a duty to a licensee to make an inspection to discover defects or dangerous conditions.
54
A shopper at a grocery store slipped and fell when he stepped in some water that had seeped out from a malfunctioning freezer case. The fall caused the shopper to break an ankle, so he filed suit against the store in a jurisdiction applying the traditional rules for landowners and possessors of land. At trial, the shopper presented evidence of the above facts, and testified that the floor around the water appeared dirty. To survive a motion for summary judgment by the store, what additional evidence must the shopper present? A No additional evidence B He was planning to make a purchase at the store. C The store employees knew that the freezer case was leaking. D His attention was diverted by store displays so that he did not notice the water on the floor.
A No additional evidence The shopper's lawsuit will survive a motion for summary judgment by the store without any additional evidence. Under the facts here, the shopper was an invitee as to the store because he came onto the premises for a purpose connected with the store's business. The store therefore owed him the duty to warn of nonobvious dangerous conditions and to make reasonable inspections to discover dangerous conditions and make them safe. The shopper's testimony that the floor around the water appeared dirty suggests that the floor had not been swept or mopped for some time. This is enough evidence to allow the jury to decide whether the store employees failed to reasonably inspect or make safe an area in which its invitees would walk, which would breach its duty to the shopper. (B) is incorrect because the shopper need not show that he planned to make a purchase to have the status of an invitee. Even if he came just to return an item or browse the aisles and compare prices, he qualifies as an invitee. (C) is incorrect because the store could be liable even if its employees did not know that water was leaking onto the floor. Because the shopper was an invitee, the store owed a duty to make reasonable inspections to discover unsafe conditions. (D) is incorrect because the shopper need not establish his due care here. Even if the shopper was not distracted by displays and should have seen the water had he been watching where he was walking, he can still recover some damages under pure comparative negligence, which allows recovery against a negligent defendant no matter how great plaintiff's negligence is. It will be an issue for the jury to determine whether and to what extent the shopper was at fault.
55
Which of the following best describes res ipsa loquitur? A The fact that a particular injury occurred establishes breach of duty as a matter of law B Proof that a defendant violated a statute establishes the existence of a duty owed and breach thereof Proof that a defendant violated a statute establishes the existence of a duty owed and breach thereof - no response given C The fact that a particular injury occurred tends to establish the breach of a duty owed D Custom or usage establishes the standard of care in a given case
C The fact that a particular injury occurred tends to establish the breach of a duty owed Res ipsa loquitur deals with those situations where the fact that a particular injury occurred may itself tend to establish the breach of a duty owed, because the type of injury that occurred would not normally occur in the absence of negligence. Where the facts are such as to strongly indicate that the plaintiff's injuries resulted from the defendant's negligence, the trier of fact may be permitted to infer the defendant's liability. Res ipsa loquitur does not refer to custom or usage establishing the standard of care. Custom or usage may be introduced to establish the standard of care in a given case. Unlike res ipsa loquitur, however, customary methods of conduct do not furnish a test that conclusively controls the question of whether certain conduct amounted to negligence. Res ipsa loquitur also does not refer to the defendant's violation of a statute as establishing the existence of a duty and breach thereof. Proof that a defendant violated an applicable statute may establish the existence of a duty owed to a plaintiff and a breach thereof, but that is not an application of circumstantial evidence to show breach of duty, as res ipsa loquitur is. Res ipsa loquitur does not establish breach of duty as a matter of law. It is still up to the trier of fact to accept or reject the evidence.
56
Which of the following is NOT a prerequisite for the plaintiff to rely on the doctrine of res ipsa loquitur? A The plaintiff was injured. B The defendant had actual possession of the instrumentality causing the injury. C There is evidence connecting the defendant with the negligence. D The accident causing the injury is the type that would not normally occur unless someone was negligent.
B The defendant had actual possession of the instrumentality causing the injury. It is not necessary for application of res ipsa loquitur to show that the defendant had actual possession of the instrumentality causing the injury, even though this may be one way to connect the defendant with the negligence that occurred. The circumstantial evidence doctrine of res ipsa loquitur deals with those situations where the plaintiff was injured and the fact that a particular injury occurred may itself establish or tend to establish a breach of duty owed. Res ipsa loquitur requires that the accident is of a type that normally does not occur in the absence of someone's negligence and the evidence connects the defendant to the negligence (in other words, this type of accident ordinarily happens because of the negligence of someone in the defendant's position).
57
A fire broke out in a home that had been recently remodeled, destroying the house and injuring the homeowner. An investigation by the fire marshal established that the fire started from a short in some wiring behind a wall. A small section of wiring that ran to an outlet through a narrow gap between a furnace chimney and a hot water pipe had part of its outer sheath cut off. The homeowner filed suit against the electrical company that did the rough wiring. The parties stipulated for trial that the company had installed the wiring in compliance with the blueprints, and that the wiring had been inspected and approved by the building inspector before the chimney and the water pipe had been installed and the walls put up, all by different contractors. At trial, the homeowner introduced the report of the fire marshal establishing how the fire started, and evidence of his medical expenses and other damages. At the end of the homeowner's case, the electrical company's attorney rested her case and moved for a directed verdict. The homeowner's attorney also moved for a directed verdict. How should the court rule on the directed verdict motions? A Deny the electrical company's motion and grant the homeowner's motion for a directed verdict, because a short in the wiring caused the homeowner's injuries. B Deny the electrical company's motion and grant the homeowner's motion for a directed verdict, because the company failed to rebut the presumption of negligence that the homeowner has established. C Deny the homeowner's motion and grant the electrical company's motion for a directed verdict, because the wire could have been damaged by another contractor. D Deny both directed verdict motions, because the homeowner has presented enough evidence to submit the case to the jury.
C Deny the homeowner's motion and grant the electrical company's motion for a directed verdict, because the wire could have been damaged by another contractor. The court should grant the electrical company's motion for a directed verdict because the homeowner has not established a prima facie case of negligence on the company's part. The homeowner has established that the electrical company owed a duty to him and that he has suffered harm from the fire caused by the short in the wiring. However, he has not established that the company breached any duty to him. While breach of duty is ordinarily a question for the trier of fact, a plaintiff's failure to offer any evidence on that element of the prima facie case will permit a directed verdict for the defendant. Under certain circumstances, the fact that a particular injury occurred may itself establish or tend to establish a breach of duty owed, permitting the trier of fact to infer the defendant's liability. This is the doctrine of res ipsa loquitur ("the thing speaks for itself"). However, for the doctrine to apply, the plaintiff must show that the accident causing his injury is the type that would not normally occur unless someone was negligent and the negligence was attributable to the defendant. The second requirement can often be satisfied by showing that the instrumentality causing the injury was in the exclusive control of the defendant. Here, however, the wiring was exposed to work done by other contractors in installing a chimney and a hot water pipe nearby and putting up the walls, and the homeowner has offered no evidence that the cut in the outer sheath of the wiring was present when the electrical company finished its work. Instead, the fact that the wiring had been approved by the building inspector suggests that the wiring was intact when the electrical company finished. Given these facts, the homeowner has not presented evidence that the negligence was attributable to the defendant. Since res ipsa loquitur does not apply and no other evidence of breach of duty was established, the electrical company's motion for a directed verdict should be granted. (C) is therefore correct, and (B) and (D) are incorrect. (B) is also incorrect because the homeowner's motion for a directed verdict would be denied even if he had established res ipsa loquitur. Establishing res ipsa loquitur merely creates a permissible inference of negligence; it does not create a presumption of negligence. Where the res ipsa loquitur element has been proved, the plaintiff has established a prima facie breach of duty on the defendant's part and no directed verdict may be given for the defendant. However, it does not require the defendant to present evidence to rebut a presumption. The trier of fact is free to accept the inference of negligence that has been created and find for the plaintiff or reject the inference of negligence and find for the defendant, even if the defendant offers no other evidence on the issue. Thus, the court would not grant the homeowner's motion for a directed verdict even if he had established res ipsa loquitur. (A) is incorrect because the electrical company is not strictly liable for the short in the wiring. The homeowner's failure to offer some evidence of negligence on the part of the electrical company will allow the electrical company to prevail.
58
Which of the following statements regarding proximate cause is true? A In direct cause cases, the unusual manner in which the injury occurred is not relevant. B In indirect cause cases, another force comes into play before the defendant's negligent act and combines with it to cause the injury. C A defendant may have proximately caused the plaintiff's injury even though she did not actually cause it. D Negligence of rescuers generally is not foreseeable.
A In direct cause cases, the unusual manner in which the injury occurred is not relevant. A direct cause case is one where the facts present an uninterrupted chain of events from the time of the defendant's negligent act to the time of plaintiff's injury. If a particular harmful result was at all foreseeable from the defendant's negligent conduct, the unusual timing of cause and effect or the unusual manner in which the injury occurred is not relevant to the defendant's liability. Negligence of rescuers is generally foreseeable because a rescue attempt is a common intervening force that is a normal response or reaction to the situation created by the defendant's negligent act (i.e., "danger invites rescue"). It is not true that a defendant may have proximately caused the plaintiff's injury even though she did not actually cause it. In fact, the opposite is true: A defendant may have actually caused the plaintiff's injury but not have proximately caused the injury, because proximate cause is a limitation on liability. In indirect cause cases, another force comes into play AFTER the defendant's negligent act and combines with it to cause the injury.
59
If a statute providing for a criminal penalty is applicable to a common law negligence case, a clearly stated specific duty imposed by the statute will replace the more general common law duty of care. Most courts hold that violation of an applicable statute __________. A is negligence per se B raises a rebuttable presumption as to duty and breach C raises a rebuttable presumption as to breach D is only prima facie evidence of negligence
A is negligence per se Most courts still adhere to the rule that violation of an applicable statute is "negligence per se." This means that the plaintiff will have established a conclusive presumption of duty and breach of duty by showing a violation of the statute. (The plaintiff still must establish causation and damages to complete the prima facie case for negligence.) For a statute to apply in a negligence case: (i) The plaintiff must show that she is in the class intended to be protected by the statute; and (ii) The plaintiff must show that the statute was designed to prevent the type of harm that the plaintiff suffered. The minority view holds either that: (i) a rebuttable presumption as to duty and breach arises, or (ii) the statutory violation is only prima facie evidence of negligence.
60
While driving his car down the road, the defendant, who had no history of heart problems, experienced a heart attack. The defendant’s car crossed the center line of the highway, in violation of a motor vehicle statute, and headed directly at a car driven by the plaintiff that was exceeding the speed limit. The plaintiff, seeing the defendant’s car heading toward him, swerved to avoid the collision. In so doing, the plaintiff’s car spun out of control and crashed into a ditch, causing the plaintiff injury. The plaintiff brought suit against the defendant for the injuries sustained in the accident. The jurisdiction retains traditional contributory negligence rules. Will the plaintiff prevail? A Yes, because the defendant’s act was a substantial factor in causing the plaintiff’s car to swerve. B Yes, because the defendant violated a statute by crossing the center line. C No, because the defendant had no prior history of heart trouble. D No, because the plaintiff was exceeding the speed limit.
C No, because the defendant had no prior history of heart trouble. The plaintiff will not prevail because the defendant had no history of heart problems. For the plaintiff to prevail in a claim against the defendant, the plaintiff must show (i) duty, (ii) breach of duty, (iii) actual and proximate cause, and (iv) damages. While drivers owe a duty of care to other drivers on the road, a driver would not be deemed to have breached that duty if he had a surprise heart attack while driving, given that he had no history of heart trouble. (A) is wrong because the fact that the defendant’s car was a cause of the accident does not establish breach of duty. (B) is wrong because violation of an applicable statute, which may sometimes establish breach of duty, will be excused where compliance was impossible. Here, because the defendant suffered a sudden heart attack, he was unable to comply with the motor vehicle statute. (D) is wrong. The facts do not establish whether the accident would have happened regardless of the plaintiff’s contributory negligence, but the trier of fact will not reach that issue because the plaintiff will not be able to establish the prima facie case.
61
To fight drug abuse, a state enacted a statute forbidding the selling of model airplane glue to anyone under the age of 18 except in small quantities in prepackaged model kits. Violation of the statute was penalized by fines or, in cases of multiple violations, possible imprisonment. The statute also required that all elementary and secondary schools licensed by the state provide comprehensive drug education programs. Neither the legislature nor the courts of the state have abolished the common law tort defense of assumption of the risk. The owner of a hobby shop in the state sold a large tube of airplane glue to a 15-year-old boy who reasonably appeared to be at least 18 years old. The boy had received drug education in his school, as mandated by the statute, including coverage of the dangers of glue sniffing. The boy understood the anti-drug instruction, but he wanted to experience it for himself. The boy sniffed the glue repeatedly and suffered permanent brain damage. If the boy's parents file suit on the boy's behalf against the store owner, for whom is the court likely to rule? A For the boy, because the store owner violated the statute when she sold the glue to the boy. B For the store owner, because the boy is not a member of the class of persons meant to be protected by the statute. C For the store owner, because the boy was aware of the danger when he sniffed the glue. D For the store owner, because the boy could reasonably have been mistaken for an 18-year-old by the store owner
A For the boy, because the store owner violated the statute when she sold the glue to the boy. The boy will prevail because the store owner's violation of the statute resulted in the boy's injuries. The applicable standard of care in a cause of action can be established by proving the applicability to that action of a statute providing for a criminal penalty. If this is done, a clearly stated specific duty imposed by the statute will replace the more general common law duty of care. For the statutory standard to be applicable, the plaintiff must show that (i) he is within the class intended to be protected by the statute, and (ii) the statute was designed to prevent the type of harm that the plaintiff suffered. Here, the statute clearly prohibited the transaction that took place between the boy and the store owner. It establishes the standard of care because the boy is a minor purchasing a large quantity of model airplane glue, and the serious injury he suffered from sniffing the glue was one of the harms that the statute was designed to prevent. The effect of establishing a violation of the statute is that a conclusive presumption of duty and breach of duty is established. The boy should then be able to establish that the store owner's sale of the glue was the actual cause and proximate cause of the boy's injuries, completing the prima facie case of negligence on the store owner's part. (B) is incorrect because minors such as the boy who were experimenting with glue sniffing were a primary target of the statute. (C) is incorrect because the boy's assumption of the risk is not a defense under these circumstances. The facts indicate that the state has retained the common law tort defense of assumption of the risk. Under this defense, a plaintiff will be denied recovery in a negligence action if he either expressly or impliedly knew of the risk of injury and voluntarily proceeded in the face of the risk. However, courts refuse to permit an assumption of risk defense in some situations because of public policy considerations. When a statute applies and is enacted to protect a class, members of that class will not be deemed to have assumed any risk. Here, even though the boy was aware of the danger when he voluntarily sniffed the glue, the statute was enacted to protect minors such as the boy from the dangers of glue sniffing. Thus, the store owner cannot rely on assumption of risk as a defense. (D) is incorrect because the statute does not provide for a reasonable mistake to excuse its violation. If the common law duty of reasonable care were applicable here, the reasonableness of the store owner's mistake would be relevant. However, the statute's specific duty replaces the more general duty of reasonable care, and violation of a statutory standard will only be excused where compliance would cause more danger than violation or would be beyond the defendant's control. Neither situation is indicated here, so the statute applies.
62
A state child safety statute required children under eight years of age to be in a government-approved car seat when riding in a motor vehicle. A father was driving to a ballgame with his seven-year-old child, who was buckled in the back seat with a regular seat belt. The father did not notice when the child unbuckled himself and started climbing into the front seat. The child grabbed the steering wheel "to help daddy steer," causing the car to swerve into the other lane and collide with another motorist's car. The motorist was seriously injured from the collision; the father and his child were unhurt. The motorist sued the father to recover damages for her injuries. At trial, the motorist presented evidence of the statute, her injuries, and the facts stated above. At the conclusion of the proofs, both parties moved for a directed verdict. How should the trial judge proceed? A Grant the motorist's motion, because the father's violation of the statute constituted negligence per se. B Grant the father's motion, because the motorist offered no evidence that the statute was intended to prevent the harm that occurred. C Deny both motions and submit the case to the jury, because the jury could find that the father is liable for his child's negligent conduct. D Deny both motions and submit the case to the jury, because the jury could find that the father breached his duty of care owed to the motorist.
D Deny both motions and submit the case to the jury, because the jury could find that the father breached his duty of care owed to the motorist. Negligence 8 of 88 of 8 Items 07:24 Question A state child safety statute required children under eight years of age to be in a government-approved car seat when riding in a motor vehicle. A father was driving to a ballgame with his seven-year-old child, who was buckled in the back seat with a regular seat belt. The father did not notice when the child unbuckled himself and started climbing into the front seat. The child grabbed the steering wheel "to help daddy steer," causing the car to swerve into the other lane and collide with another motorist's car. The motorist was seriously injured from the collision; the father and his child were unhurt. The motorist sued the father to recover damages for her injuries. At trial, the motorist presented evidence of the statute, her injuries, and the facts stated above. At the conclusion of the proofs, both parties moved for a directed verdict. How should the trial judge proceed? Responses A Grant the motorist's motion, because the father's violation of the statute constituted negligence per se.Grant the motorist's motion, because the father's violation of the statute constituted negligence per se. - no response given B Grant the father's motion, because the motorist offered no evidence that the statute was intended to prevent the harm that occurred.Grant the father's motion, because the motorist offered no evidence that the statute was intended to prevent the harm that occurred. - no response given C Deny both motions and submit the case to the jury, because the jury could find that the father is liable for his child's negligent conduct.Deny both motions and submit the case to the jury, because the jury could find that the father is liable for his child's negligent conduct. - incorrect D Deny both motions and submit the case to the jury, because the jury could find that the father breached his duty of care owed to the motorist.Deny both motions and submit the case to the jury, because the jury could find that the father breached his duty of care owed to the motorist. - not selected, this is the correct answer Answer Discussion - Incorrect The court should deny both motions and submit the case to the jury, because the jury could find that the father was negligent in not preventing his child from grabbing the steering wheel. Under ordinary negligence principles, the father owed a duty to other motorists to maintain control of his vehicle. The jury could find that the father breached this duty of care by not noticing when his child unbuckled the seat belt and started climbing into the front seat, and not preventing the child from grabbing the steering wheel. Hence, the case should go to the jury for a determination of whether the father was negligent. (A) is incorrect because the facts do not establish that the statutory standard of care is applicable to these facts. The precise standard of care in a common law negligence case may be established by proving that a statute providing for a criminal penalty applies to the particular case. If that is done, the statute's more specific duty will replace the more general common law duty of care. Violation of the statute establishes negligence per se-a conclusive presumption of duty and breach of duty; the plaintiff must then establish causation and damages to complete the prima facie case of negligence. To prove that the statutory standard applies, the plaintiff must show that (i) she is in the class intended to be protected by the statute, and (ii) the statute was designed to prevent the type of harm that occurred. Here, the statute likely was intended primarily to protect children from injuries caused by not being properly restrained in a vehicle involved in a collision. The motorist has presented no evidence that the statute was intended to protect her from the harm that she suffered. (B) is incorrect because even though the motorist did not establish that the statute applies to her claim, she has presented facts sufficient to allow the jury to find that the father breached the common law duty of care that he owed to other motorists. (C) is incorrect because the father is not vicariously liable for his child's negligent conduct at common law. Any liability of the father in this case would arise from his own potential negligence in failing to control his child while driving.
63
The driver of a tanker truck was transporting radioactive waste from a nuclear power plant to a permanent storage facility in a remote western region of the United States. After driving all night, the driver fell asleep at the wheel and the truck crossed over the center line, off the road, and onto a homeowner's property, coming to rest after crashing into several glass cases containing the homeowner's collection of poisonous snakes, the keeping of which was permitted by local ordinance. When the driver exited the truck, he was bitten on the leg by one of the poisonous snakes and became seriously ill. The driver brought an action against the homeowner for his injuries. The parties stipulated to the above facts, and that the driver violated a state statute by driving off of the road. Both parties moved for judgment as a matter of law on the liability issue. How should the court rule? A Grant the driver's motion and deny the homeowner's motion, because the homeowner is strictly liable for the injury caused by the snake. B Deny the driver's motion and grant the homeowner's motion, because the driver was a trespasser on the homeowner's property. C Deny the driver's motion and grant the homeowner's motion, because the driver's violation of the state statute establishes contributory negligence as a matter of law. D Deny both parties' motions, because both parties were engaged in an activity for which strict liability is imposed.
B Deny the driver's motion and grant the homeowner's motion, because the driver was a trespasser on the homeowner's property. The court should grant the homeowner's motion for judgment as a matter of law because the driver has not established a prima facie case against the homeowner. An owner of wild (dangerous) animals is strictly liable for injuries caused by those animals as long as the person injured did nothing, voluntarily or consciously, to bring about the injury. However, strict liability generally is not imposed in favor of undiscovered trespassers against landowners in the absence of negligence, such as when the landowner knows that the trespassers are on the land and fails to warn them of the animal. Here, despite the fact that the driver did not intend to enter the homeowner's land (and thus would not be liable for the intentional tort of trespass), his status on the homeowner's land is that of a trespasser rather than a licensee or invitee. The driver has presented no evidence of negligence on the homeowner's part and therefore has not established a prima facie case against the homeowner. (A) is wrong because, as discussed above, the homeowner is not strictly liable to the driver because the driver was a trespasser. (C) is incorrect because the driver will not prevail regardless of whether he was contributorily negligent, because he cannot establish a prima facie case against the homeowner in either negligence or strict liability. (D) is incorrect for several reasons: While the driver's transport of radioactive waste may have been an abnormally dangerous activity, that danger had nothing to do with the accident that occurred. Furthermore, the fact that the driver may have been engaged in an abnormally dangerous activity would not prevent him from recovering damages from another tortfeasor if he established the requisite prima facie case. Finally, the fact that the parties were engaged in activities potentially creating strict liability has nothing to do with whether issues of fact regarding liability still exist that would require denying both motions and going to trial.
64
A bolt of lightning struck a tree, causing it to fall on a farmer's fence which enclosed a pasture containing the farmer's large bull. The bull escaped through the broken fence and entered the neighbor's property. It gored a hiker who was crossing the neighbor's property without permission. In the hiker's action against the farmer based on strict liability, is the hiker likely to prevail? A Yes, because the bull caused harm while trespassing on another's property. B Yes, because bulls have known dangerous propensities. C No, because a bull is a domestic animal. D No, because the hiker was a trespasser.
C No, because a bull is a domestic animal. The hiker will not prevail because strict liability does not apply to a bull, which is a domestic animal. The owner of a domestic animal, including a farm animal, is not strictly liable for injuries it causes, as long as the owner has no knowledge that the animal has abnormally dangerous propensities (i.e., propensities more dangerous than normal for that species). A bull is a domestic animal, and nothing in the facts suggests that the bull was more dangerous than normal for that type of animal. Hence, strict liability will not apply. (A) is incorrect because the rule for trespassing animals does not apply. The owner of a trespassing animal is strictly liable for harm done by the trespass as long as it was reasonably foreseeable. Here, the bolt of lightning caused the fence to break and allowed the bull to escape. This unforeseeable intervening force was the cause of the trespass; hence, the strict liability rule for trespassing animals does not apply here. (B) is incorrect because, as discussed above, strict liability does not apply for domestic animals with normal dangerous propensities. Only domestic animals with propensities more dangerous than normal for the species may subject the owner to strict liability. (D) is incorrect because the hiker's status as a trespasser on the neighbor's land is irrelevant as to the farmer's liability. If the hiker were a trespasser on the farmer's land, strict liability would not apply even if the bull were abnormally dangerous, but the farmer's liability is not affected by the hiker's status as to the neighbor. Note that if strict liability applied for harm from an animal trespassing on a neighbor's property, the hiker's status as a trespasser might be relevant because strict liability applies only to injured persons who were rightfully on the property. However, as discussed above in (A), that liability is inapplicable here because the bull's trespass was unforeseeable.
65
Which of the following may prevent establishing causation against a manufacturer in a strict products liability action? A The retailer's labeling of the product as its own B The destruction of the product because of its dangerous defect C The failure of a retailer to take action after discovering a dangerous defect D The negligent failure of a retailer to discover a dangerous defect
C The failure of a retailer to take action after discovering a dangerous defect The failure of a retailer to take action after discovering a dangerous defect may prevent establishing causation against a manufacturer in a strict products liability action. The same concepts of proximate cause that govern negligence and strict liability actions are applicable to strict liability actions for defective products. As with products liability cases based on negligence, the negligent failure of a retailer to discover a dangerous defect does not cut off the supplier's strict liability. On the other hand, when the intermediary's conduct becomes something more than ordinary foreseeable negligence, it becomes a superseding cause. Hence, the conduct of a retailer who discovered a dangerous defect and then took no action (such as alerting the manufacturer, warning the consumer, or removing the product from sale) constitutes more than ordinary foreseeable negligence and may cut off the manufacturer's liability. The destruction of the product because of its dangerous defect does not prevent establishing actual cause. If the product has been destroyed, the plaintiff may rely on an inference that this type of product failure ordinarily would occur only as a result of a product defect. The retailer's labeling of the product as its own will not affect the liability of the manufacturer.
66
Which of the following statements of law does NOT relate to proving actual cause in a strict products liability case? A The defendant cannot avoid liability by showing negligent failure of an intermediary to discover the defect. B If the defect is difficult to trace, the plaintiff may rely on an inference that such a product failure ordinarily would occur only as a result of a defect. C If the defect has inadequate warnings, the plaintiff is entitled to a presumption that an adequate warning would have been read and heeded. D The defect in the product must have existed when the product left the defendant's control.
A The defendant cannot avoid liability by showing negligent failure of an intermediary to discover the defect. The rule that the defendant cannot avoid liability by showing negligent failure of an intermediary to discover the defect does not relate to actual cause; rather, it relates to the proximate cause principle that negligence of an intermediary is foreseeable and not a superseding cause. Under this principle, the intermediary's negligence does not cut off the defendant's liability for supplying a defective product. The basic requirement to show actual cause is that the defect in the product must have existed when the product left the defendant's control. When a defect is difficult to establish (such as if the product is destroyed), the plaintiff may rely on an inference that such a product failure ordinarily would occur only as a result of a defect (similar to res ipsa loquitur). To show that inadequate warnings were an actual cause of the injury, the plaintiff is entitled to a presumption that an adequate warning would have been read and heeded (i.e., but for the lack of an adequate warning, the plaintiff would not have been injured).
67
A consumer purchased a grass trimmer from a hardware store. He took it out of the sealed box and assembled it according to the instructions. He noticed that there were bolts and screws left over and some joints that could have accepted additional fasteners, but he just disposed of the extra hardware. As he was using the trimmer, the housing came apart and a hard piece of plastic flew off. His neighbor, who was standing nearby, was struck in the eye by the piece of plastic and suffered permanent injuries. The neighbor sued the hardware store and the manufacturer of the trimmer in a strict liability action. Through discovery, it was determined that the instructions omitted a critical step in the assembly process that would have used the extra hardware, which is why the housing came apart, and that the manufacturer had received some complaints about the instructions previously. The hardware store had no knowledge of any complaints regarding any of the manufacturer's products. As to the hardware store, the neighbor will: A Recover, because the manufacturer had reason to know that the design was defective because of faulty instructions. B Recover, because the consumer's failure to recognize the improper assembly does not cut off the store's liability. C Not recover, because the neighbor is not a consumer and can only recover against the manufacturer of the product. D Not recover, because the hardware store had no opportunity to inspect the product and no reason to anticipate that the instructions were faulty.
B Recover, because the consumer's failure to recognize the improper assembly does not cut off the store's liability. (B) The neighbor will recover against the hardware store. In a products liability action based on strict liability, the plaintiff need show only (i) the defendant is a commercial supplier, (ii) the defendant produced or sold a product that was defective when it left the defendant's control, (iii) the defective product was an actual and proximate cause of the plaintiff's injury, and (iv) the plaintiff suffered damages to person or property. Here, the hardware store is in the chain of supply of the product. The product was defective because the instructions omitted a critical part of the assembly process. Finally, the omission was an actual and proximate cause of the neighbor's injury, allowing him to recover against the hardware store. As indicated by choice (B), an intermediary's negligent failure to recognize the danger does not cut off the supplier's strict liability. (A) is incorrect because the manufacturer's awareness of the faulty instructions does not affect whether the hardware store will be liable. The hardware store was not aware of any problems with the product, but it is nevertheless liable as a commercial supplier. (C) is incorrect. The neighbor, as a bystander, is within the foreseeable zone of danger and is therefore a foreseeable plaintiff who can recover in this action. (D) is incorrect because the fact that the hardware store was not at fault and had no opportunity to inspect is irrelevant. It is liable because it is a commercial supplier of a defective product and the neighbor is suing under a strict liability theory.
68
A researcher and an analyst had employment contracts with a corporation. The corporation fired them because it believed they had conspired to give its trade secrets to a competitor. Knowing that the conspiracy allegations were not true, the researcher and the analyst sued the corporation in federal district court for breach of their employment contracts. The researcher seeks $100,000 in damages and the analyst seeks $50,000. They are both citizens of State A. The corporation is incorporated in State B, and its principal place of business is in State B. Does the federal district court have subject matter jurisdiction over both of the claims? A Yes, the court has diversity of citizenship jurisdiction over both claims because the amounts of their related claims may be aggregated to satisfy the amount in controversy requirement. B Yes, the court has diversity of citizenship jurisdiction over the researcher's $100,000 claim and supplemental jurisdiction over the analyst's $50,000 claim. C No, the court does not have subject matter jurisdiction over the analyst's $50,000 claim because the claims of both plaintiffs may not be aggregated to satisfy the amount in controversy requirement. D No, the court does not have subject matter jurisdiction over either claim because the breach of contract claim is based on state law.
B Yes, the court has diversity of citizenship jurisdiction over the researcher's $100,000 claim and supplemental jurisdiction over the analyst's $50,000 claim. The court has jurisdiction over both claims but under different bases. Generally, every claim in federal court must have a basis for federal subject matter jurisdiction. There are three main flavors of federal subject matter jurisdiction-diversity of citizenship jurisdiction, federal question jurisdiction, and supplemental jurisdiction. Diversity of citizenship jurisdiction is available when (i) there is complete diversity of citizenship, meaning that each plaintiff is a citizen of a different state from every defendant; and (ii) the amount in controversy exceeds $75,000. A natural person's citizenship is the state that is the person's permanent home. A corporation is a citizen of every state in which it is incorporated and the one state in which it has its principal place of business. The amount in controversy is determined by the plaintiff's good faith allegation. In the instant case, the facts state that the researcher and the analyst are citizens of State A, and that the corporation was incorporated and has its principal place of business in State B. As a result, complete diversity exists. The researcher claims $100,000, presumably in good faith. Therefore, the researcher may claim diversity jurisdiction to have his claim heard in federal court. The analyst, on the other hand, claims only $50,000 in damages; thus, he may not invoke diversity jurisdiction to have his claim heard in federal court. However, the analyst might invoke supplemental jurisdiction to have his claim heard in federal court. When the federal court has subject matter jurisdiction over one claim, it has discretion to exercise supplemental jurisdiction over related claims that derive from the same common nucleus of fact and are such that a plaintiff would ordinarily be expected to try them in a single judicial proceeding. Here, the analyst's claim is derived from the corporation's wrongful termination of both plaintiffs based on an allegation that they conspired to sell a trade secret to a competitor. As a result, the analyst's claim springs from the same common nucleus of fact as the researcher's claim. There are several restrictions on the use of supplemental jurisdiction when joined with a claim based solely on diversity; however, a co-plaintiff with a below-limit claim generally can use supplemental jurisdiction to have his claim heard in federal court so long as the co-plaintiff's presence in the suit does not destroy complete diversity. Here, the analyst's presence does not destroy complete diversity. Thus, the analyst may invoke supplemental jurisdiction to have his claim heard in federal court, making (B) correct. (A) is incorrect because multiple plaintiffs may not aggregate their claims against a single defendant unless they are seeking to enforce a joint right in which they have an undivided interest. Such a case is not presented under these facts. (C) is incorrect. Although it is true that the claims of both plaintiffs may not be aggregated, the answer ignores the possibility of there being supplemental jurisdiction. (D) is incorrect because state law claims may be heard in federal court under the court's diversity jurisdiction.
69
An investor filed a civil action against her broker in a State A state court seeking $30,000 for violation of federal securities statutes. The investor is a citizen of State B and the broker is a citizen of State A. May the broker remove the action to federal district court? A No, because the amount in controversy is too small for federal subject matter jurisdiction. B No, because the defendant is a citizen of the state from whose court the action is removed. C Yes, the broker may remove the action to federal district court on the basis of federal question jurisdiction. D Yes, the broker may remove the action to federal district court on the basis of federal diversity of citizenship jurisdiction.
C Yes, the broker may remove the action to federal district court on the basis of federal question jurisdiction. The broker may remove the action on the basis of federal question jurisdiction. A defendant may remove an action that could have originally been brought in the federal courts, either on the basis of a federal question being presented or on the basis of diversity of citizenship. Federal question jurisdiction is available when the plaintiff, in his well-pleaded complaint, alleges a claim that arises under federal law. In the instant case, the cause of action is based on a violation of federal securities law. Thus, a federal question has been presented, making the case removable on the basis of federal question jurisdiction. (A) is incorrect. Although diversity of citizenship jurisdiction has an amount in controversy requirement of more than $75,000, there is no amount in controversy requirement for federal question jurisdiction. (B) is also incorrect. When jurisdiction of the federal court is based solely on diversity, and one of the defendants is a citizen of the state in which the action was brought, the action is not removable. There is no similar restriction on removal when removal is based on federal question jurisdiction. (D) is incorrect because the amount in controversy ($30,000) is not sufficient to invoke diversity jurisdiction (i.e., it does not exceed $75,000).
70
A zoologist was moving to State A from State B, where she had lived her entire life. She was driving a truck and trailer that contained all of her possessions across the country when she was involved in a severe accident in State C. After being released from the hospital, but before suit was filed, the zoologist proceeded to State A, where she rented an apartment and found a job. The driver of an automobile involved in the severe accident intends to file a negligence action against the zoologist in federal district court. The driver is a citizen and resident of State D. In which federal district(s) is venue proper? A The District of State C only. B The District of State C and the District of State B. C The District of State C and the District of State A only. D The District of State C, the District of State D, and the District of State A.
C The District of State C and the District of State A only. Venue is proper in the District of State C and the District of State A. Federal venue in civil actions is proper in (i) the district where any defendant resides, if all defendants are residents of the state in which the district is located; and (ii) the district in which a substantial part of the events or omissions giving rise to the claim occurred. If there is no district anywhere in the United States that satisfies (i) or (ii), the action may be brought in a judicial district in which any defendant is subject to the court's personal jurisdiction with respect to such action. Thus, the District of State C is a proper venue because a substantial part of the events giving rise to the claim occurred there, and the District of State A is a proper venue because that is where the zoologist/defendant was domiciled at the time the case was filed. Accordingly, (A) and (B) are incorrect. (D) is incorrect because, in federal court, venue cannot be based on the plaintiff's residence.
71
A bar prep company discovered that its copyrighted content was being used in an online simulated exam that its competitor was administering in a few days. The company filed a petition for an ex parte order to direct the competitor to immediately remove that content from its website. The company submitted an affidavit specifying why immediate and irreparable injury will result if the exam is administered, and offered to provide security for any costs or damages incurred by the competitor if it was determined that the order was wrongfully issued. Should the court issue the order? A Yes, because the company submitted an affidavit with specific facts showing immediate and irreparable injury. B Yes, because the competitor will receive notice of the order once it is issued by the court. C No, because a court cannot issue an injunction unless the adverse party has notice of the hearing. D No, because the company has not provided sufficient certification for obtaining an ex parte order.
D No, because the company has not provided sufficient certification for obtaining an ex parte order. The court should not issue the ex parte order. A temporary restraining order may be granted by a court when it is necessary to prevent irreparable injury to a party, and the injury will result before a preliminary injunction hearing can be held. As a general rule, notice of the hearing for the issuance of the order must be given before it is issued. However, a court may grant an ex parte temporary restraining order without notice of the hearing to the adverse party if the moving party does the following: (i) gives specific facts in an affidavit or a verified complaint to establish that immediate and irreparable injury will result to the moving party before the adverse party can be heard in opposition; (ii) certifies in writing all efforts it made to give notice to the adverse party and why notice should not be required; and (iii) provides some security to pay for any costs and damages incurred by the adverse party if it is wrongfully enjoined or restrained. Here, the company alleged irreparable injury and offered to provide security, but nothing in the facts indicates that it certified efforts to give notice to the competitor or why notice should not be required. The facts do not establish that it is impossible to provide notice to the competitor before a restraining order is issued. (A) is incorrect because the company's affidavit is not enough to justify the issuance of an ex parte order, as discussed above. (B) is incorrect because, while actual notice of the order also is required, notice of the hearing is required as well unless the three requirements stated above are met. (C) is incorrect because it is too broad. A court may issue an ex parte order without prior notice to the adverse party if the three requirements stated above are met.
72
A plaintiff filed a single cause of action against a defendant on a claim based on products liability in state court. The plaintiff contends that a space heater that he was using during the winter short-circuited, causing a fire that burned down his house. In July, the plaintiff's case against the defendant went to trial, and the jury determined that the fire was caused by faulty wiring in the plaintiff's home. The following September, the plaintiff filed a breach of warranty action against the defendant in federal court, properly invoking the court's diversity jurisdiction. The defendant, after properly being served with process, files an answer with an affirmative defense of claim preclusion as his first pleading or motion. The defendant, at an appropriate time, then files a motion for summary judgment based on the assertion of claim preclusion. How should the court rule on the defendant's motion? A Deny the motion, because the application of claim preclusion is a factual issue for the jury to determine. B Deny the motion, because the plaintiff's breach of warranty theory is not the same cause of action as the plaintiff's products liability claim. C Grant the motion, because claim preclusion applies to bar the claim. D Grant the motion, because the defendant, by raising the issue in his first pleading or motion, did not waive the defense of claim preclusion.
C Grant the motion, because claim preclusion applies to bar the claim. The court should grant the motion on claim preclusion grounds. Before claim preclusion (also called res judicata) applies, it must be shown that (i) the earlier judgment is a valid, final judgment "on the merits"; (ii) the cases are brought by the same claimant against the same defendant; and (iii) the same "cause of action" (or "claim") is involved in the later lawsuit. While various tests have been used to define "cause of action," the modern approach is to require assertion of all claims arising out of the same transaction or occurrence that is the subject matter of a claim asserted by the claimant. Here, the products liability claim was determined in the defendant's favor by a jury. Thus, there is a valid, final judgment. The same plaintiff is suing the same defendant, and the same claim is involved, as both the products liability and breach of warranty claims arise out of the same transaction or occurrence-the alleged malfunction of the space heater that burned down the plaintiff's home. Thus, claim preclusion applies to bar the claim. (Note that when the plaintiff loses the earlier case, claim preclusion "bars" further litigation. When the plaintiff wins the earlier case, the different theories "merge" to prevent further litigation.) (A) is incorrect because claim preclusion is not a factual issue for the jury to determine. It is a legal issue for the judge to determine. (B) is incorrect because, as explained above, a later claim is deemed to be the same claim if it arises from the same transaction or occurrence as the earlier claim. (D) is incorrect because a defense of claim preclusion is not necessarily waived if it is not included in the defendant's first pleading or motion. A judge would have discretion to allow a defendant to amend his answer to include an omitted defense, and she will likely do so when no prejudice to the defendant results. (Leave to amend is freely granted when justice so requires.)
73
A principal will be vicariously liable for the tortious acts of her independent contractor __________. A if the principal negligently supervised the independent contractor B if the principal negligently selected the independent contractor C if the independent contractor is engaged in inherently dangerous activities D under the doctrine of respondeat superior
C if the independent contractor is engaged in inherently dangerous activities
74
A developer constructed several small stores in a commercial district. She received a bid from a contractor to install awnings on the front windows of the stores. The developer had heard that the contractor did shoddy work, but the price was right and the contractor expressly assumed all of the risk of any liability. The developer subsequently sold one of the stores to a barber. A few months later, an awning collapsed without warning, injuring a customer who was about to enter the barbershop. An investigation by the building inspector revealed that the awning collapsed because the brackets used by the contractor were cheaper and weaker than the required brackets, although they looked the same. The developer and the contractor are now both bankrupt. If the customer sues the barber for his injuries, is the customer likely to prevail? A Yes, because the barber had a nondelegable duty to keep the premises safe for customers and those passing by. B Yes, because the developer did not exercise reasonable care in hiring the contractor to install the awnings. C No, because the contractor assumed all of the risks from his work. D No, because the barber had no opportunity to oversee the contractor's actions.
D No, because the barber had no opportunity to oversee the contractor's actions. The customer is not likely to prevail because the barber had no opportunity to oversee the contractor's work. A property owner owes a duty to those adjacent to the premises to take due precautions to protect them from dangerous conditions, and a business owes its customers a duty to make reasonable inspections to discover and make safe any dangerous conditions. Further, that duty cannot be delegated to an independent contractor; the owner remains vicariously liable. However, nothing in the facts establishes that the barber breached his duty to the customer. There was nothing wrong with the brackets evident from a reasonable inspection, and the awning collapsed without warning. Further, the barber was not involved in hiring or supervising the contractor and would not be responsible for the contractor's negligence. Hence, because no facts point to negligence by the barber, the customer is not likely to prevail. (A) is incorrect even though it is true that a business owner would be vicariously liable to customers and passersby injured by the negligent work of an independent contractor that he hired. Here, however, the barber did not engage the contractor and is not responsible for the contractor's conduct. (B) is similarly incorrect. While the developer arguably was negligent in hiring a contractor who does shoddy work, her negligence will not be imputed to the barber. (C) is incorrect because it is irrelevant. Had the barber hired the contractor, the fact that the contractor contractually assumed all of the risks of liability would not preclude the customer from recovering against the barber.
75
Acting pursuant to a valid search warrant, the police entered and searched the defendant's garage and discovered a cardboard box containing cocaine in the rafters storage area. The box was securely taped and bore a freight label addressed to the defendant's friend. At his trial for violation of the jurisdiction's statute making it a felony to knowingly possess cocaine, the defendant testified that his friend had brought him the package a week before it was seized by the police, telling him that he needed to store it in the defendant's garage. The defendant also testified that he had not asked the friend what it contained. What additional facts must the prosecution prove to establish the defendant's liability for the charged felony? A That he knew or believed that the box contained cocaine and had moved or handled the box. B That he knew or believed that the box contained cocaine. C That he should have known that the box contained cocaine and had moved or handled the box. D No additional facts.
B That he knew or believed that the box contained cocaine. The defendant should be found guilty of the charged felony if he knew or believed that the box contained cocaine. The defendant is being tried for "knowingly" possessing cocaine. A person does not act knowingly unless he is aware that his conduct is of the proscribed nature or that the proscribed circumstances exist. Thus, the defendant could not have acted knowingly unless he knew or believed that the box contained cocaine. (A) is incorrect because criminal statutes that penalize the possession of contraband generally require only that the defendant have control of the item for a long enough period to have had an opportunity to terminate the possession. Thus, the defendant need not have moved or handled the box. (C) is incorrect for the same reason as (A), and also because the defendant's failure to know when he should have known would constitute negligence-failure to be aware of a substantial risk that prohibited results will follow or that circumstances exist-and negligence is not sufficient to establish knowledge. Note, however, that a defendant may not consciously avoid learning the true nature of the item possessed; knowledge may be inferred when the defendant is aware of a high probability of the true nature of the item and deliberately avoids learning the truth. (D) is incorrect because, as discussed above, the statute requires that the defendant knew or believed that the box contained cocaine.
76
An employee of the state government always received his state paycheck on the last workday of the month. The employee was not a good money manager, and just barely managed to make it from paycheck to paycheck each month. On the second to the last workday of the month, the employee had $45 in his checking account, and, needing to buy a birthday gift for his sister, he wrote a check to a gift boutique for $100. He knew that he would be receiving his paycheck the next day, so he could deposit the paycheck before the check would be sent to the bank. However, unbeknownst to the employee, the state legislature was having a budget impasse. Because the state constitution prohibited any deficit spending, state employees were not paid as usual. Without a paycheck to deposit, the check written to the gift boutique was returned for insufficient funds. The merchant complained to the police, who arrested the employee and charged him under a statute that prohibited "issuing a check knowing that it is drawn against insufficient funds, with intent to defraud the payee of the check." What should be the outcome of the employee's prosecution? A Not guilty, because the employee intended to deposit his paycheck the next day. B Not guilty, because it was reasonable for the employee to expect that he would receive his paycheck as usual. C Guilty, because the employee knew when he wrote the check that he did not have sufficient funds in his account to honor it. D Guilty, because reliance on a future source of income does not vitiate the employee's violation of the statute when he wrote the check.
A Not guilty, because the employee intended to deposit his paycheck the next day. Given that the employee intended to deposit his paycheck before the checks cleared, he lacked the intent to defraud required by the statute. The statute under which the employee is being prosecuted is a variation of the offense of false pretenses. As with false pretenses, the statute requires a specific intent, i.e., an intent to defraud. If the employee intended to deposit sufficient funds to honor the check before it reached his bank, then the employee did not intend to defraud the gift boutique. Thus, the employee lacked the specific intent that is a necessary element of the crime charged. (B) is incorrect because the employee's expectation that he would receive his paycheck as usual need not have been reasonable. Even if such an expectation were unreasonable, the employee would not be guilty if he did not intend to defraud the payees, as required by the statute. (C) is incorrect because it would result in a verdict of guilty without requiring intent to defraud. Knowledge that the check was drawn against insufficient funds is just one element of the statute. The intent to defraud is also required to convict under the applicable statute. (D) also incorrectly assumes that the employee violated the statute merely by knowingly writing a check on insufficient funds. As explained above, the requisite intent to defraud is absent. Thus, there is no "violation" to be vitiated.
77
The defendant and the victim got into a minor verbal altercation, concluding with the defendant lightly shoving the victim. The victim lost his balance and struck his head on the pavement, causing serious bodily injury. The defendant was charged with battery, which is defined in the jurisdiction as "purposely or knowingly causing serious bodily injury to another." Should the defendant be convicted of battery? A No, because the defendant did not know that the victim would be seriously injured. B No, because the defendant did not strike a serious blow to the victim. C Yes, because the defendant purposely shoved the victim. D Yes, because the victim suffered serious bodily injury.
A No, because the defendant did not know that the victim would be seriously injured. The defendant should not be convicted of battery. Under the statute's fault standards, a defendant must have acted purposely (i.e., with conscious intent to cause the result) or knowingly (i.e., with knowledge that his conduct will necessarily or very likely cause the result) as to the harmful result. The apparent inference to be drawn from the facts is that the defendant did not consciously desire, nor contemplate to a practical certainty, the serious injury to the victim that actually occurred. Had the defendant intended to cause such severe harm, he no doubt would have dealt the victim a strong blow rather than simply giving the victim a light shove. Therefore, as to the nature of the result, the defendant did not act with "purpose" or "knowledge" as those terms are defined in the Model Penal Code and modern criminal codes. (B), while close, is not as good an answer as (A) because it does not address the state of mind issue in the problem. A light shove might be sufficient for a battery as defined under a different set of facts (e.g., if the defendant believes that the victim would fall down stairs with a light shove). (C) is incorrect because it addresses the act but not the result. As defined in this question, battery must not only be committed by a purposeful act, but also be done with a "purposeful" or "knowing" state of mind as to the result. (D) is incorrect for much of the same reason-the state of mind requirement also applies to the result, as discussed above. The injury must have been purposely or knowingly caused, and that concept is not contained within choice (D).
78
In a property settlement after a divorce, the wife was awarded all personal property that had been accumulated during the marriage, including the husband's classic 19-inch black-and-white TV set. In order to get his prized TV set back, the husband lied to his friend, telling him that the wife took the TV set in violation of the property settlement. The friend remembered that the wife gave the friend's wife a key to her new home, and he volunteered to go with the husband to get the TV back while the wife was at work. The husband and the friend went to the wife's house, but, unbeknownst to them, the wife had taken the day off work. After the friend noisily opened the back door with his wife's key, the wife called the police, who quickly arrived and arrested the husband and the friend. As to a charge of common law conspiracy to commit larceny, how should the friend be found? A Not guilty, because he did not intend to steal. B Not guilty, because he did not have a corrupt motive. C Guilty, because there was an agreement, and the opening of the locked door was sufficient for the overt act. D Guilty, because good motives are not a defense to criminal liability.
A Not guilty, because he did not intend to steal. The friend should be found not guilty because he did not intend to steal. At common law, conspiracy consists of (i) an agreement between two or more persons, (ii) an intent to enter into an agreement, and (iii) an intent to achieve the objective of the agreement. The object of the agreement must be something unlawful. Here, the friend did not intend to achieve the objective of the conspiracy-to permanently deprive the owner of her property-because the friend thought the husband was the true owner of the TV. (C) is incorrect because there must be an agreement to reach an unlawful objective. Because the friend thought he was achieving a lawful objective, he did not have the intent required for conspiracy. (B) is incorrect because a "corrupt motive" is not an element of a crime. A person could be found guilty of a crime even if he did not have a corrupt motive, assuming all required elements for a crime are present. (D) is incorrect for a similar reason. "Good motive" is largely irrelevant; the intent, or lack thereof, is what is important.
79
A new homeowner had two dogs that frequently barked at birds and squirrels in the yard, especially during the day while the homeowner was at work. A neighbor who worked nights was aggravated by the barking, which disturbed his sleep, and decided to let the homeowner know how he felt. One evening, upon learning that the homeowner was entertaining her boss and several clients, the neighbor came to her front door with a boombox and started playing a recording of the dogs barking, putting it at full volume. When the homeowner came to the door, he began yelling at her and berating her in front of her guests for having no consideration for her neighbors, while continuing to play the recording. The homeowner was very upset, especially because her guests decided that they had better leave, and she ended up losing a bonus that her boss was going to give her at the end of the evening. If the homeowner asserts a claim based on intentional infliction of emotional distress against the neighbor, what will be the probable result? A The homeowner will prevail because the neighbor's conduct was extreme and outrageous. B The homeowner will prevail because she suffered pecuniary harm from the neighbor's conduct. C The neighbor will prevail because the homeowner suffered no physical harm. D The neighbor will prevail if the barking from the homeowner's dogs is judged to constitute a nuisance.
A The homeowner will prevail because the neighbor's conduct was extreme and outrageous. The homeowner will probably prevail on a claim for intentional infliction of emotional distress because the neighbor's conduct was sufficiently extreme and outrageous and the other elements of the tort are present. Intentional infliction of emotional distress requires: (i) an act by defendant amounting to extreme and outrageous conduct; (ii) intent to cause severe emotional distress or recklessness as to the effect of defendant's conduct; (iii) causation; and (iv) damages. "Outrageous conduct" is extreme conduct that transcends all bounds of decency. The neighbor's use of the recording and his insults against the homeowner for the benefit of her guests would probably qualify as extreme and outrageous conduct, particularly because there is no evidence that he had previously tried to resolve the problem with the homeowner in a more civilized manner. The neighbor had the requisite intent (either he intended to cause emotional distress or he was reckless as to its effect), there was causation, and the homeowner suffered damages (that is, she was severely distressed) as a result of the neighbor's actions. (B) is wrong because pecuniary harm is not required for purposes of this tort-all that is required is severe emotional distress. (C) is wrong because, in contrast to negligent infliction of distress, intentional infliction of distress does not require proof of physical harm to recover. (D) is wrong because the fact that the barking constituted a nuisance would not be a defense to conduct amounting to intentional infliction of distress; abatement of a private nuisance by self-help must be preceded by notice to the other party and must be conducted in a reasonable manner.
80
A college student borrowed his roommate's notebook computer without permission because he needed to write a term paper that was due the next day. While the computer was sitting open on the student's desk overnight, a water pipe in the ceiling began leaking and water dripped down on the computer, rendering it inoperable. A computer repair service estimated that it would cost $500 to repair all the damaged components. At the time it was damaged, the computer was worth $700. If the roommate sues the student for the damage caused to the computer, what will be the extent of his recovery? A Nothing, because the damage occurred through no fault of the student. B Loss of use damages for the time it was in the student's possession. C $500 in damages. D $700 in damages.
D $700 in damages. The roommate can recover $700 in damages from the student for conversion. To establish a prima facie case of conversion, the following elements must be proved: (i) an act by defendant interfering with plaintiff's right of possession in the chattel, (ii) intent to perform the act bringing about the interference with plaintiff's right of possession, (iii) causation, and (iv) damages-an interference that is serious enough in nature or consequence to warrant that the defendant pay the full value of the chattel. Even if the conduct is wholly innocent, liability may attach where the interference is serious in nature. Accordingly, accidentally causing damage to another's chattel may constitute a conversion when the damage occurred while the defendant was using the chattel without permission. Here, the student interfered with the roommate's right of possession in the computer by taking it without permission, and it sustained damages of over 70% of its value while in the student's possession. Hence, the student has committed a conversion. The plaintiff in a conversion case is entitled to damages for the fair market value of the chattel at the time and place of the conversion, which in this case was $700. (A) is incorrect because even though the student was not at fault in the water pipe leaking, the damage occurred while the computer was wrongfully in his possession. (B) is incorrect. Had the computer not been damaged, the roommate's recovery would be limited to loss of use damages under a trespass to chattels theory. However, the serious damage that occurred while the computer was in the wrongful possession of the student warrants a recovery for conversion. (C) is incorrect because the damages remedy for conversion is the fair market value; in effect, there is a forced sale of the item. The student may keep the computer but he is liable to the roommate for the entire value of the computer rather than just the cost of repairs.
81
A homeowner bought "20-pound test" fishing line for hanging potted plants on his porch. "20-pound test" in the fishing industry means that fishing line will not break under an initial stress of up to 20 pounds when a hooked fish tugs against the line, but not that it will support a constant 20-pound weight. Most sportfishers are aware of this technical meaning, but most laypersons are not, and the manufacturer put no warnings or explanations on the package in which the line was sold. The homeowner hung a 15-pound basket from his front porch, directly above an old-fashioned porch swing. A friend visiting the homeowner was sitting on the swing when the line holding the basket broke, causing the plant to fall and strike the guest on the head. In a jurisdiction following the traditional rules for landowners and possessors of land, will the guest prevail against the homeowner in a suit to recover damages for her injuries? Responses A Yes, because she was a social guest. B Yes, because the homeowner was negligent in hanging the plant. C No, because the homeowner could not be expected to know the technical meaning of "20-pound test." D No, because she was not a foreseeable plaintiff.
C No, because the homeowner could not be expected to know the technical meaning of "20-pound test." The guest will not prevail in a suit against the homeowner. As a social guest of the homeowner's, the guest is deemed to be a licensee; that is, one who enters onto land with the owner's permission for her own purpose or business rather than for the owner's benefit. In a jurisdiction following the traditional rules for landowners and possessors of land, the owner owes a licensee the duty to warn of or make safe a dangerous condition known to the owner that creates an unreasonable risk of harm to the licensee and that the licensee is unlikely to discover. The owner has no duty to a licensee to inspect for defects nor to repair known defects. The homeowner, as a person who was not involved with fishing, had no reason to suspect that a fishing line that was "20-pound test" could not support the constant weight of a 15-pound basket. Thus, the homeowner did not know of the dangerous condition present in the form of the basket overhanging his porch. Because the homeowner was unaware of the danger, he was under no duty to warn the guest, a licensee, of the dangerous condition. Having violated no duty owed to the guest, the homeowner will not be held liable for her injuries. (A) is accurate in stating that the guest was a social guest. However, as detailed above, the duty owed to a guest is simply to warn of concealed dangerous conditions of which the owner is aware. The homeowner had no duty to warn of a danger of which he neither knew nor had reason to know. (B) is incorrect because there is no indication either that the homeowner hung the basket in a negligent manner or that he was negligent in failing either to warn the guest or to be aware of the danger. The homeowner appears to have acted as would a reasonable person with no knowledge of the meaning of technical terms of fishing. (D) is incorrect because a social guest would indeed be a foreseeable plaintiff. If the homeowner had been negligent in hanging the basket directly above the swing, it would have been reasonably foreseeable that an injury would befall any person who sat on the swing. Thus, (D) reaches the correct result that the guest will not prevail, but for an incorrect reason.
82
A swimmer went to a privately owned lake resort whose owner charged a fee for admission. The beach had a roped-in swimming area and large signs directing swimmers not to swim anywhere but within the ropes. The lifeguards regularly enforced this rule. The resort also rented canoes and rowboats to its patrons, who could take them anywhere on the lake. The swimmer and two of his friends had rented a canoe and started to paddle out toward the other side of the lake when the swimmer saw a volleyball game starting on the beach that he wanted to join. He left his friends in the canoe and started swimming to shore. He was only a few yards outside of the roped-in swimming area when he started, but he angled away from the swimming area toward the area of the beach where the volleyball net was set up. Although the lifeguard on duty saw him, she did not warn him to return to the swimming area. When the depth of the water was about four feet, he put his foot down and was severely cut by the jagged edge of a rusted metal stake protruding a few inches out of the bottom of the lake. The swimmer had not seen the stake even though the water was clear and it was visible if he had looked down. If the swimmer sues the resort in a jurisdiction that applies the traditional rules for landowners and possessors of land, is he likely to recover? A No, because the stake could have been seen by the swimmer. B No, because he was swimming outside of the roped-in area. C Yes, because the lifeguard on duty saw him and did not warn him to return to the swimming area. D Yes, because he is a public invitee of the resort.
B No, because he was swimming outside of the roped-in area. The swimmer cannot recover from the resort because he did not have invitee status when he was injured. In jurisdictions following the traditional rules for landowners and possessors of land, the nature of a duty of an owner or occupier of land to those on the premises depends on the legal status of the plaintiff in regard to the property, i.e., whether the plaintiff is a trespasser, licensee, or invitee. An invitee is a person who enters onto the premises in response to an express or implied invitation of the landowner. Those who enter as members of the public for a purpose for which the land is held open to the public and those who enter for a purpose connected with the business or other interests of the landowner are considered invitees. However, a person will lose his status as an invitee if he exceeds the scope of the invitation-if he goes onto a portion of the property where his invitation cannot reasonably be said to extend. Here, the swimmer was an invitee of the resort in the areas to which it allowed its patrons to go. However, the resort clearly identified the boundaries of the area held open to swimmers, and the swimmer could not reasonably have believed that he was invited to swim in the area where he was injured. Because the swimmer was at most a licensee when he was injured, the resort did not owe him a duty to make reasonable inspections of that area to discover dangerous conditions and make them safe. At most, the resort had a duty only to warn the swimmer of known dangerous conditions that create an unreasonable risk of harm to him and that he is unlikely to discover, and nothing in the facts indicates that any employees of the resort knew of the stake under the water. The swimmer therefore cannot recover against the resort. (A) is not as good a choice as (B). While a landowner is not liable for a dangerous condition that is obvious to the entrant on the land, the fact that the stake was visible does not establish that it was obvious, given that the swimmer was looking forward rather than down. Whether a danger is obvious is determined by all of the surrounding circumstances, not just whether the danger is visible. The better reason why the swimmer cannot recover is because he was no longer an invitee. (C) is incorrect because the lifeguard's failure to direct the swimmer to the swimming area would not constitute an invitation to swim in the restricted area; at most, it would establish only that the swimmer was a licensee rather than a trespasser when he swam in that area. A licensee is one who enters onto land with the possessor's permission, express or implied, for his own purpose or business rather than for the possessor's benefit. The lifeguard's conduct may have constituted implied permission for the swimmer to exit the lake in a nonswimming area for his own benefit, but it does not establish that he reasonably believed that he was invited to swim in that area. (D) is incorrect because the swimmer lost his status as an invitee when he exceeded the scope of his invitation by swimming in an area where swimming was not permitted.
83
A bicyclist was riding his bicycle in the street when a negligently driven car struck the bike, knocking the bicyclist off the bike and breaking his right ankle. The driver of the car immediately stopped and went to his assistance. She got him to his feet and was slowly moving him toward the curb when a negligently driven taxicab struck him in the left leg. The bicyclist required surgery on both his right ankle and his left leg. If the bicyclist sues the driver and the cabbie, which of the following best states his right to recover? A He can recover from either the driver or the cabbie for all of his injuries because the driver and the cabbie are jointly and severally liable. B He can recover from the driver only for the injury to his right ankle and recover from the cabbie only for the injury to his left leg. C He can recover from either the driver or the cabbie for the injury to his left leg and recover from the driver only for the injury to his right ankle. D He cannot recover against the driver for the injury to his left leg unless the jury determines that the driver acted negligently when she came to his aid.
C He can recover from either the driver or the cabbie for the injury to his left leg and recover from the driver only for the injury to his right ankle. The bicyclist can recover from either party for the left leg injury but only from the driver for the right ankle injury. When two or more tortious acts combine to proximately cause an indivisible injury to a plaintiff, each tortfeasor is jointly and severally liable to the plaintiff for the entire damage incurred. Joint and several liability applies even though each tortfeasor acted entirely independently. However, if the actions are independent, plaintiff's injury is divisible, and it is possible to identify the portion of injuries caused by each defendant, then each will be liable only for the identifiable portion. Here, the cabbie would not be liable for the injury to the right ankle, because the cabbie did not cause the injury. (A) is therefore incorrect. With regard to the left leg, the cabbie was not the only cause of that injury. The original tortfeasor is liable for harm caused by the negligence of third persons when such negligence was a foreseeable risk created by the original tortfeasor's conduct. Here, as a result of the driver's original negligence, the bicyclist was in a position of danger while he was still in the street. The negligence of the cabbie in striking the bicyclist was a foreseeable risk while the bicyclist was in the street; it is therefore a foreseeable intervening force that will not cut off the driver's liability. Hence, both the driver and the cabbie will be jointly and severally liable for that injury. (B) is therefore incorrect. (D) is incorrect because the driver remains responsible for the foreseeable consequences of her original negligence in striking the bicyclist, regardless of whether she acted with due care when she came to his aid.
84
A small cruise ship struck a whale swimming underwater, causing the ship to suddenly lurch sideways. A passenger on the ship who was walking down a corridor lost his balance and bumped his head on the edge of a doorway. Because of a previously existing medical condition that made him susceptible to bleeding on the brain, he suffered a cerebral hemorrhage and permanent mental impairment, despite prompt medical attention on the ship. The passenger brought suit against the cruise ship owner for his damages. At trial, the passenger presented evidence of how he was injured as he walked down the hallway, his previous medical condition, and his medical expenses and other damages. The cruise ship owner presented evidence that the cruise ship was following its approved route and that the whale could not have been detected before impact, and that the bump would not have injured someone in ordinary health. At the close of the evidence, the cruise ship owner moved for a directed verdict. How should the court rule? A Grant the motion, because there is no evidence that the crew operated the ship negligently. B Grant the motion, because the cruise ship owner introduced uncontroverted evidence that a person in normal health would not have been injured by the bump. C Deny the motion, because the jury could find that the cruise ship owner, as a common carrier and innkeeper, breached its high duty of care to the passenger. D Deny the motion, because the fact that the severity of the passenger's injuries was not foreseeable does not cut off the cruise ship owner's liability.
A Grant the motion, because there is no evidence that the crew operated the ship negligently. The court should grant the cruise ship owner's motion because the passenger has not established a prima facie case of negligence against the cruise ship. To establish a prima facie case for negligence, a plaintiff must show (i) a duty of care, (ii) breach of that duty, (iii) actual and proximate cause, and (iv) damages. As a common carrier and/or an innkeeper, the cruise ship owed its passengers a high duty of care, and therefore would be liable for slight negligence. However, the passenger has offered no evidence to establish that the cruise ship employees breached that duty, and res ipsa loquitur is not applicable here because the collision with the whale swimming underwater is not the type of event that would occur only as a result of negligence. Because the passenger failed to establish breach of duty, the court should grant the cruise ship owner a directed verdict. (B) is incorrect because the cruise ship owner does not need that evidence to prevail. While evidence that a person in normal health would not have been injured by the bump supports the cruise ship's other evidence that it exercised due care, it is not necessary because the passenger has failed to offer evidence that the cruise ship owner breached its duty. On the other hand, if the cruise ship owner had breached its duty of care to its passengers, the fact that a person in normal health would not have been injured by the bump on the head would not be a defense to liability. If a defendant's negligence causes an aggravation of a plaintiff's existing physical illness, the defendant is liable for the damages caused by the aggravation. (C) is incorrect because, as discussed above, the passenger has failed to present evidence that the cruise ship owner breached the high duty of care that it owed to its guests. (D) is incorrect even though it is a true statement of law, as discussed above. The reason the cruise ship owner prevails is because the passenger has failed to establish a prima facie case.
85
A man working at a clothing store discovered that his girlfriend, a coworker, had been taking money from the cash register. Not wanting to be a party to the situation, he ended the relationship and found another job. Not long after this, the man's new boss, who knew why the man had quit, came into the clothing store. He asked the girlfriend if she missed her boyfriend working with her at the store. She replied, "Yes, but when we found that he was stealing from the cash register, we had no choice but to let him go." If the man sues his former girlfriend for defamation, the fact that the new boss knew the truth of why the man had left his job at the store will have what result? A It will act as a complete defense to an action for defamation. B It will establish that the man has not suffered any actual injury. C It may diminish the damages that the man would be entitled to recover. D It proves that the girlfriend had no reasonable ground for believing that the man was fired for dishonesty.
C It may diminish the damages that the man would be entitled to recover. The new boss's knowledge of the true circumstances behind the man's departure from the store may diminish the man's recovery. The girlfriend is liable for defamation because she made a defamatory statement about the man to a third person. As long as it is understood in its defamatory sense, an accusation need not be believed to be actionable. Because the statement that he was stealing at his job constituted slander per se, damages are presumed, completing the prima facie case. Nevertheless, the stated fact will diminish the amount of damages that the man will recover because his reputation was not likely harmed in the eyes of his boss. (A) is wrong because, as stated above, the man can establish a prima facie case for defamation even though the defamatory statement was not believed. (B) is wrong because actual injury encompasses not only damage to reputation but also humiliation and mental distress, for which the man could recover even if his reputation was not damaged. (D) is wrong because the fact that the new boss did not believe the statement does not prove lack of basis for the girlfriend to have made it.
86
A company that owned a tract of land believed to be rich in mineral deposits contracted with a licensed excavator for the removal of soil from the property and delivery of the soil to the company's laboratories. While one of the excavator's trucks was on the way to the laboratory, the rear gate broke loose, dumping three tons of soil onto the highway. A motorist who was driving a short but safe distance behind the truck was unable to stop in time and collided with the soil, causing her serious injury. The rear gate had been negligently secured by one of the excavator's employees. If the motorist sues the company for his injuries and does not prevail, what is the most likely reason? A The rear gate was secured by the excavator's employee. B The excavator had a license to transport soil on the highway. C The company's duty in respect to the movement of its soil on the highway was delegable. D The transportation of soil on the highways was a common practice in the area where the accident occurred.
C The company's duty in respect to the movement of its soil on the highway was delegable. The strongest basis for the motorist not prevailing is the absence of a nondelegable duty. The general rule is that a principal will not be liable for tortious acts of its agent if the agent is an independent contractor. However, a broad exception will impose liability on the principal if the duty is nondelegable because of public policy considerations. As long as the company was not subject to a nondelegable duty, it would not be liable for the negligence of the excavator's employee in the transportation of its soil. (A) is not as good an answer as (C) because the fact that the accident was caused by the negligence of the independent contractor's employee does not necessarily excuse the company from liability. (C) supplies the additional factor that enables the company to avoid liability. (B) is incorrect because the possession of a license by the excavator would not excuse the company from liability. (D) is incorrect because the fact that the transportation of soil was common to the area is relevant only for a strict liability action for abnormally dangerous activities, and the transport of soil by truck is not such an activity.
87
Assuming all other elements are met, in which of the following circumstances would the defendant most likely not be convicted of homicide? A The defendant stabs the victim in the heart; at exactly the same time, another person shoots the victim in the head. Either act would have independently caused the victim's death, but it is unclear which act caused the victim's death. B The defendant stabs the victim but does not kill him; due to construction at the hospital where the victim is admitted for treatment of the stab wound, the victim contracts asbestosis and dies from it one year later. C The defendant stabs and kills a victim who has terminal cancer. D The defendant stabs the victim, who has hemophilia; the victim bleeds to death as a result of the stabbing.
B The defendant stabs the victim but does not kill him; due to construction at the hospital where the victim is admitted for treatment of the stab wound, the victim contracts asbestosis and dies from it one year later. If the defendant stabs the victim but does not kill him, and due to construction at the hospital where the victim is admitted for treatment of the stab wound, the victim contracts asbestosis and dies as a result one year later, the defendant is not likely to be convicted of homicide. To be convicted of homicide, the defendant must have actually and proximately caused the death of the victim. An intervening act that presents a foreseeable risk will generally not break the chain of causation. However, an unforeseeable risk, such as an injury due to hospital construction, will most likely break the chain of causation. Stabbing and killing someone with terminal cancer is likely to be considered homicide. A defendant may be guilty of killing a victim who was going to die anyhow. Actual and proximate causation is not broken if the defendant ends the victim's life prematurely, even by a short time. If a defendant stabs a victim while another person shoots the victim, the defendant is likely to be found guilty of homicide. Simultaneous acts by multiple persons may be considered independent, sufficient causes of a single result. Thus, multiple persons may be convicted of homicide even though there was only one death. A defendant who stabs a victim with hemophilia who bleeds to death is likely to be found guilty of homicide. Any preexisting conditions that make a person more susceptible to death are essentially disregarded. The defendant "takes the victim as he finds him."
88
The victim owned a cottage in an ocean resort area. He stayed there only during the summer months, and left the cottage unoccupied during the balance of the year. The defendant, a resident of a neighboring cottage, was aware of this practice. For a change in his routine, however, the victim decided to spend a week at the cabin in the off-season. Unaware that the victim was occupying the cottage, the defendant decided to borrow a portable television set that he knew the victim kept in the cottage. To avoid being seen, he entered the cottage late at night, using a key under the front doormat. He found the television set, disconnected it, and headed for the rear of the house to leave. He opened the kitchen door and found the victim seated there in the dark, having a late night snack. Both men were startled and neither man recognized the other in the dark. The defendant assumed that the victim was a burglar, and was afraid that he might be armed. Trying to flee the kitchen as quickly as possible, the defendant dropped the television set in the middle of the kitchen floor. As the set hit the floor, the picture tube exploded with a loud noise. The noise so frightened the victim that he had an immediate heart attack and died. If the defendant is charged with felony murder as the result of the victim's death, what is his best defense? A He did not intend to kill the victim. B His only intent was to borrow the television set for a few days. C Larceny is not an inherently dangerous crime, and it was not being committed in an inherently dangerous manner. D The victim's heart attack was an unforeseeable consequence of the defendant's acts.
B His only intent was to borrow the television set for a few days. The defendant's best defense to felony murder is that he only intended to borrow the television set for a few days. By establishing this intent, the defendant will show that he did not have the intent to commit a felony and therefore cannot be guilty of felony murder. A killing (even if accidental) committed during the course of a felony is murder. Malice is implied from the intent to commit the underlying felony. To convict a defendant of felony murder, the prosecution must prove that he committed or attempted to commit the underlying felony. Here, the possible felonies being committed by the defendant, during which the victim's death occurred, would be larceny and burglary. Burglary requires the intent to commit a felony within the dwelling, and larceny requires the intent to permanently deprive a person of his interest in property. If the defendant's only intent was to borrow the victim's television set for a few days, then the defendant lacked the intent to permanently deprive the victim of his interest in the set; i.e., the requisite intent for larceny is missing. Likewise, the absence of intent to steal the set would mean that, at the time of breaking and entering the cottage, the defendant did not intend to commit a felony therein. Consequently, the defendant is not guilty of burglary. Because under these circumstances no felony would have been attempted or committed, it cannot be shown that the death of the victim occurred during the commission of a felony. Therefore, the defendant would not be guilty of felony murder. (D) is tempting, because generally a conviction of felony murder requires that the death must have been a foreseeable result of commission of the felony. However, some courts do not apply a foreseeability requirement and require only that the underlying felony be malum in se. Furthermore, even those courts applying a foreseeability requirement have been willing to find most deaths occurring during the commission of a felony to be foreseeable. Here, the defendant believed that the cottage was unoccupied for the winter. Thus, it was arguably unforeseeable that the defendant's entering the cottage and taking a television set would result in the death of an occupant, but it is by no means certain that a court would agree. Furthermore, in those jurisdictions that do not require foreseeability of death, the defendant could be convicted of felony murder if the death occurred during the commission of a burglary, because burglary is always classified as a malum in se felony. Because the circumstances in choice (B) would assure the defendant of avoiding conviction in all jurisdictions, (B) is a better answer than (D). (A) incorrectly focuses on intent to kill. Intent to kill is one of the states of mind by which a defendant is deemed to have malice aforethought, which is necessary for a killing to constitute murder. However, this question refers to felony murder, wherein malice aforethought exists in the form of intent to commit a felony. Thus, it is irrelevant whether the defendant intended to kill the victim. Regarding (C), it is true that most courts limit the felony murder doctrine to felonies that are inherently dangerous, and that larceny generally is not considered to be inherently dangerous. However, assuming the existence of the requisite intent, the defendant may have committed burglary, which is deemed to be inherently dangerous. Thus, (C) might provide no defense at all to a charge of felony murder.
89
The owner of a garage and one of his mechanics had a dispute over the amount of wages due for work that the mechanic had done on cars. The owner alleged that he paid the mechanic for work that the mechanic did not perform, while the mechanic contended that she did the work. After arguing over the matter for over an hour, the mechanic quit. The owner did not allow the mechanic to take her tools with her, stating that he was keeping her tools until she repaid the money that he had overpaid her. The mechanic met with a friend and discussed the situation with him. After hearing what had happened, the friend, believing that the owner was unlawfully retaining the mechanic's tools, suggested that he could pose as a garage client and retrieve the tools for the mechanic, and the mechanic agreed. The plan succeeded, except for the fact that the friend took tools that belonged to a new employee. The owner immediately realized what had happened, and he gave the friend's license number to the police. The friend was arrested before he could show the tools to the mechanic. Of the following, which is the best argument for the friend as a defense to a charge of larceny in a common law jurisdiction? A He had the consent of the tools' owner. B He thought the tools belonged to the mechanic. C He intended to return the tools to the mechanic rather than keep them. D He was apprehended by the police before the mechanic could inspect the tools.
B He thought the tools belonged to the mechanic. The friend's best argument is that he thought the tools belonged to the mechanic. Larceny is the taking and carrying away of the tangible personal property of another by trespass with the intent to permanently (or for an unreasonable time) deprive the owner (or person in possession) of his interest in the property. Thus, larceny is a specific intent crime, in that the defendant must intend to permanently deprive the person of his interest in the property. Importantly, this intent generally must exist at the time of taking (save for the continuing trespass doctrine). In this case, the friend believed that the mechanic was being unlawfully deprived of his property, and his intent was to return the tools to their rightful owner. Thus, when he took the tools, he lacked the intent to permanently deprive another of his interest in the property. This would be true even if the garage owner factually had a lawful right to retain the tools, as the mistake of law would negate the friend's intent to permanently deprive another of his interest in the property. (A) is wrong, of course, because he did not have the consent of the owner of the tools he took. (C) is wrong because it is not a defense to a charge of larceny that the defendant did not intend to keep the item taken for his own use. (D) is wrong because it prematurely raises the continuing trespass doctrine. Under the continuing trespass doctrine, a defendant may be convicted of larceny when he initially takes property with a wrongful state of mind, but without the intent to permanently deprive the owner of the property, and then later forms the intent to permanently deprive the owner of the property. Here, the friend had not yet had the opportunity to form that later intent (even assuming the initial taking was with a wrongful state of mind). Thus, (D) does not address the correct issue.
90
The defendant's neighbor owned an authentic major league baseball signed by Babe Ruth. The defendant asked if he could show it to some friends who were visiting. The neighbor agreed as long as he kept it in the display case, which the defendant promised to do. In fact, the defendant intended to use the ball in a pickup game. During the game, the ball was hit over the fence and into a yard with a guard dog, which had chewed up several other balls that had previously landed in the yard. The dog did the same to that ball. When the neighbor learned what happened to the ball, he pressed charges against the defendant. If the defendant is convicted, he will most likely be found guilty of what crime? A Common law larceny. B Embezzlement. C False pretenses. D Larceny by trick.
D Larceny by trick. The defendant is guilty of larceny by trick because he obtained possession of the baseball by means of a misrepresentation. Larceny is the taking and carrying away of tangible personal property of another by trespass, with intent to permanently (or for an unreasonable time) deprive the person of her interest in the property. The taking must be without the consent of the person in possession of the property. If such consent is induced by a misrepresentation of a past or existing fact, the consent is not valid. The resulting larceny is called larceny by trick. Here, the defendant obtained possession of the baseball with the owner's consent. However, this consent was obtained by means of the defendant's misrepresentation about friends visiting. This was a false statement of an existing fact, made with the intent that his neighbor rely on the statement, and the misrepresentation induced his neighbor's consent. At the time of this taking, the defendant intended to deal with the baseball in a manner that involved a substantial risk of damage or loss. This suffices as intent to permanently deprive. Therefore, all the elements are in place for larceny by trick. (A) is not as good a choice as (D) because the taking in this case is better characterized as larceny by trick rather than larceny, given that the defendant induced his neighbor to consent to his taking possession of the baseball. (C) is incorrect because the defendant obtained only possession of the baseball, not title. False pretenses differs from larceny by trick in what is obtained. If the defendant obtains only possession of the property, the offense is larceny by trick, whereas obtaining of title means that false pretenses has been committed. What the victim intended to convey to the defendant is determinative. The neighbor intended only to let the defendant borrow the baseball for a short time, not to convey title to him. Consequently, the only thing the defendant obtained was possession of the baseball. Because title to the baseball was not obtained, there can be no conviction of false pretenses. Regarding (B), embezzlement is the fraudulent conversion of property of another by a person in lawful possession of that property. In embezzlement, misappropriation occurs while the defendant has lawful possession of the property, while in larceny, it occurs generally at the time the defendant obtains wrongful possession of the property. Here, as detailed above, the defendant's taking of possession of the baseball was trespassory due to the manner in which he obtained consent to such possession. The crime of larceny was complete on the defendant's taking possession with the requisite intent to permanently deprive. Thus, at the time the baseball was destroyed, the defendant had already misappropriated it and was not in lawful possession of it. As a result, there can be no conviction for embezzlement.
91
A student and a few of his friends were making their way to spring break. Along the way, the old van that they were driving broke down. Not wanting to miss any part of spring break festivities, the student asked the mechanic on duty at the repair shop for a rush job. The mechanic provided the student with a repair estimate, and the student, on the basis of the estimate, authorized the repair and promised to pay when he came back to pick up the van. When the mechanic called the student to tell him that the van was repaired, the student, rather than paying for the repair, told one of his friends that the mechanic had agreed to finance the repair charges and that the only thing left to do was pick up the van in the garage's parking lot. The student handed the friend a key to the van and told him to go pick the van up so that they could continue their trip to spring break. The friend did so. The mechanic makes a criminal complaint against the student for larceny of the van. If the case is prosecuted, will the student likely be found guilty? A No, because it was the student's van to begin with. B No, because the friend took the van. C Yes, because the friend took the van from the mechanic without the mechanic's knowledge or permission. D Yes, because the student promised to pay the mechanic for his work when he came to get the van.
C Yes, because the friend took the van from the mechanic without the mechanic's knowledge or permission. The student will most likely be found guilty. Larceny is the taking and carrying away of the personal property of "another" with the intent to permanently deprive the other person of the property. It is possible to commit larceny of your own property if another person, such as a bailee, has a superior right to possession of the property at that time. Because the mechanic had a right to possession of the van until he was paid, the student committed larceny when he had his friend take the van without the mechanic's consent. (B) is wrong because a person can be guilty even though he did not personally engage in the behavior if he acts through an innocent agent. (D) is also incorrect. The student is guilty, but not for the reason stated in (D). He would be guilty even if he had not made the promise to pay for the van; he incurred an obligation to pay by having the repairs done.
92
Acting on a hunch, a police officer went to a young woman's apartment, broke in, and searched it. The officer found exactly what she was looking for under the woman's bed: a sack filled with jewels. The attached note read, "Sweetheart, here are the goods from the estate heist. Your loving boyfriend." It was well known in the community that the woman's boyfriend was a jewel thief. The officer also knew that the estate of a local socialite had been burglarized three days ago. Just as the officer finished reading the note, the woman returned. The officer immediately placed the woman under arrest as an accessory to the estate burglary. Based on the evidence obtained from the woman's apartment, a search warrant was issued for her boyfriend's apartment. The search yielded burglar tools and more jewels from the estate. The boyfriend was immediately arrested and charged with the estate burglary. At the boyfriend's trial for the estate burglary, his attorney files a motion to suppress the evidence consisting of the bag of jewels and note, the tools, and the jewels from the boyfriend's apartment. How should the court rule on the motion? A Grant the motion as to the bag of jewels and note, but deny it as to the evidence found in the boyfriend's apartment. B Grant the motion, because all of this evidence is fruit of the poisonous tree. C Deny the motion, because the police would have caught the boyfriend with the goods eventually. D Deny the motion, because the police had a warrant to search the boyfriend's apartment.
D Deny the motion, because the police had a warrant to search the boyfriend's apartment. The court should deny the motion to suppress because the police had a warrant to search the boyfriend's home. The boyfriend's expectation of privacy extended only to his own home, which was searched under a warrant. He does not have standing to assert a Fourth Amendment claim regarding the search of his girlfriend's apartment because her apartment was not his home, and he did not own it or have a right to possession of it. Thus, (A) is incorrect. Because the boyfriend cannot object to the search that provided the probable cause for the search of his apartment, (B) is also incorrect. (C) is not a valid justification because there is nothing to indicate that the seizure would fall under the "inevitable discovery" exception to the exclusionary rule.
93
The police received information linking a man to drug trafficking and went to the man's residence, where he lived with his mother. The police found the mother at home, and she told them that her son was not expected back until later. The police informed the mother that they suspected the man of selling drugs and asked if they could search his room. She replied, "I'm finished with that no-good bum; not only is he into drugs, but he has been stealing my money to pay for them, and all the time I'm making his bed and fixing his food. You can search his room. He likes to keep his private stuff under his pillow. I hope he goes to jail." The police searched the man's room and discovered a quantity of marijuana under the pillow of his bed. If before trial the man's attorney moves to suppress the marijuana on grounds that the search was invalid, should the court grant the motion? A Yes, because the man had a legitimate expectation of privacy in the area searched, and the police did not have a warrant. B Yes, because the man's mother's consent was given at a time when police knew her interests were in conflict with the man's. C No, because the man's mother had the authority to consent to the search of his room. D No, because with the mother's statement the police had probable cause to search the room.
C No, because the man's mother had the authority to consent to the search of his room. The man's motion to suppress should be denied because his mother had authority to consent to the search of his room. A search of a residence can be based on the voluntary consent of the occupant. Where a parent has general access to a room occupied by a son or daughter, the parent can give a valid consent to a general search of the room even if the son or daughter is an adult. The facts in the question indicate that the man's mother had general access to his room ("and all the time I'm making his bed"). Therefore, her consent is valid and eliminates the need for probable cause and a warrant. (A) is wrong. The man had a legitimate expectation of privacy in the area searched, but the consent of his mother eliminated the need for a warrant. (B) is wrong. At one time, some courts required an "amicable relationship" between the parties before the police could rely on a third party's consent. The "amicable relationship requirement" is no longer recognized by the courts. (D) is not a good answer. It is true that with the mother's statement the police had probable cause to search the man's room. However, probable cause alone would not validate the search. The police would need probable cause plus a warrant or a valid consent. In this question the search would have to be based on consent.
94
A defendant was convicted after a jury trial of violation of federal statutes prohibiting the sale of automatic weapons to foreign nationals. It was established at trial that the defendant had purchased a number of stolen United States Army heavy machine guns and attempted to ship them abroad. The trial court expressly based its imposition of the maximum possible sentence for the conviction on the defendant's refusal to reveal the names of the persons from whom he purchased the stolen weapons. His counsel argues that this consideration is reversible error. If the defendant appeals the sentence imposed, what should the appeals court do? A Reverse the trial court, because the consideration of the defendant's silence violates his Fifth Amendment privilege against self-incrimination. B Reverse the trial court, because the consideration of collateral circumstances in sentencing violates his due process rights. C Affirm the trial court, because the right to remain silent granted by the Fifth Amendment does not include the right to protect others from incrimination. D Affirm the trial court, because citizens must report violations of the criminal statutes.
C Affirm the trial court, because the right to remain silent granted by the Fifth Amendment does not include the right to protect others from incrimination. The appeals court should affirm the trial court because the right to remain silent does not include the right to protect others from incrimination. The defendant was not privileged to refuse revealing the names of the stolen weapon sellers. The United States Supreme Court held, in Roberts v. United States (1980), that a defendant's refusal to cooperate with an investigation of the criminal conspiracy of which he was a member may properly be considered in imposing sentence. This is because the Fifth Amendment right to remain silent does not afford a privilege to refuse to incriminate others. (C) is therefore correct and (A) is incorrect. (B) is incorrect because the court's consideration of the defendant's refusal to cooperate does not violate due process. (D) is not an accurate statement of the law.
95
A man was driving very erratically when he was stopped by state troopers and arrested for drunk driving. He was advised of his constitutional rights and invoked his right to remain silent. At trial for his drunk driving charge, the man testified in his own defense, stating that he had just left his doctor's office and had been administered medication without being told that it would seriously and immediately hamper his coordination. On cross-examination, the prosecutor asked whether the defendant just made up this medication story after the fact to evade legitimate liability for driving while intoxicated and the man said he had not. The prosecutor then asked why the defendant had not told the arresting officer about the medication, and defense counsel objects. The trial court should rule that the question is: A Improper, because to require the defense to inform the prosecution of defendant's testimony prior to trial would be unconstitutional pretrial discovery. B Improper, because use of defendant's post-arrest silence violates his right to due process of law. C Proper, because defendant's silence was not used as direct evidence but only for impeachment on cross-examination. D Proper, because defendant's post-arrest silence is a prior inconsistent statement which is admissible to show recent fabrication.
B Improper, because use of defendant's post-arrest silence violates his right to due process of law.
96
Two members of a backgammon club owned identically sized, red backgammon boards. The first member's board was made of cheap material while the second member's board was quite expensive. One night, after a competitive tournament, the two members met in the finals, playing on a borrowed board. The second member won and the first member, visibly upset, mistakenly grabbed the other's board and drove home. As was her custom, she left the board in the trunk of her car. Meanwhile, the owner of the board discovered the board switch and drove to the first member's apartment to make an exchange. The first member took the second to her parking place and saw that her car had been stolen. The police recovered the car days later, with no backgammon board in the trunk. The second member demanded a replacement board, but was refused. In an action to recover the board's value, will the second member recover? A Yes, because when the first member took the board she committed a trespass to chattel. B Yes, because when the board was stolen along with the car, the first member became liable for conversion. C No, because the first member believed in good faith that the board was hers when she took it from the club. D No, because the board was lost through no fault of the first member.
B Yes, because when the board was stolen along with the car, the first member became liable for conversion. The plaintiff will win because the defendant committed a conversion. A conversion occurs when the defendant intentionally causes serious interference with the chattel of the plaintiff. The intent involved refers to the physical act that results in the conversion, not to the defendant's desires regarding the ultimate disposition of the property. Therefore, the first member was guilty of conversion when she intentionally (i.e., volitionally) took the second's board, which resulted in its loss, even though the first member did not intend to lose it or even realize that she had taken the property of another. (A) is not the best answer because complete loss of a chattel is too serious an interference to be classified a mere trespass. (C) is wrong because the first member's good faith is irrelevant in a conversion action. (D) is wrong because the fact that the first member's car was stolen does not relieve her of liability.
97
A driver traveling the speed limit in the evening on a quiet country road rounded a curve and struck a bicyclist who was riding in the same lane. The driver stopped the car and inspected the bicyclist, who had a broken leg. The driver thought it best not to try to move the bicyclist, so he told him that he would go to get help. The driver drove away and left the bicyclist by the side of the road. After the driver had left the scene, he realized that he had forgotten his wife's birthday, so he stopped to buy a gift and hurried home. He did not remember the bicyclist until a few hours later, but assumed that by that time someone would have come along to render assistance. However, the bicyclist was not rescued until the following morning. By then, he had contracted pneumonia as a result of exposure. The bicyclist sued the driver to recover damages for his broken leg and the pneumonia. If the jury finds that the driver was not negligent in his operation of his automobile, for what harm will the bicyclist most likely recover? A Both the leg injury and the pneumonia. B The leg injury but not the pneumonia. C The pneumonia but not the leg injury. D Neither the leg injury nor the pneumonia.
C The pneumonia but not the leg injury. The bicyclist will most likely recover for the pneumonia but not for the leg injury. The facts and the call of the question indicate that the driver was not driving negligently when the accident occurred. Therefore, he is not liable for the leg injury caused by the accident, and (A) and (B) are incorrect. However, where the defendant's actions have placed another person in peril or caused another's injury, the defendant has a duty to make reasonable efforts to rescue the imperiled person or render aid to his victim. The driver's neglect of the bicyclist after injuring him will make him liable for the resulting pneumonia. Therefore, (B) and (D) are incorrect.
98
A tenant invited a friend over for dinner. On his arrival, the friend stepped on a split board on the front steps and the board broke, causing him to lose his balance and break his ankle. If the friend sues the tenant for his injuries and does not prevail in a jurisdiction that applies the traditional rules for landowners and possessors of land, what is the most likely explanation? A In the lease, the landlord had undertaken the duty to discover and repair dangerous conditions on the premises. B The friend arrived an hour earlier than his invitation specified. C The friend should have noticed the dangerous condition himself. D The tenant had stayed beyond the lease term and she no longer had the legal right to occupy the premises.
C The friend should have noticed the dangerous condition himself. If the friend does not prevail, it will be because he should have noticed the dangerous condition himself. In jurisdictions applying the traditional rules for landowners and possessors of land, the nature of the duty owed by an owner or occupier of land to those on the premises for dangerous conditions on the land depends on the legal status of the plaintiff in regard to the property, i.e., trespasser, licensee, or invitee. A licensee is one who enters on the land with permission for his own purpose or business and includes social guests. The owner or occupier owes a licensee a duty to warn of or make safe a dangerous condition known to the owner or occupier that creates an unreasonable risk of harm to the licensee and that the licensee is unlikely to discover. The owner or occupier does not have a duty to inspect for defects or to repair known defects. Here, the friend had been invited for dinner, making him a licensee. The facts do not indicate whether the tenant knew of the split board and neglected to alert the friend or simply was not aware of it, but the duty to warn does not extend to dangerous conditions that the licensee should reasonably have discovered. Hence, (C) presents the best basis for the friend not prevailing. (A) is incorrect because that fact would not make a difference to the tenant's liability. The tenant remains liable to the friend for dangerous conditions on the premises as the occupier of the land, regardless of the landlord's obligation to inspect and repair. (B) is incorrect because the friend still qualifies as a licensee even though he arrived sooner than his invitation specified. It is true that a person may lose invitee status and become a licensee by being on the premises at a time outside the scope of his invitation. However, there is no similar principle applicable here. The fact that the friend arrived an hour early does not make him a trespasser rather than a licensee. Hence, the fact in (B) would not affect the tenant's liability. (D) is incorrect because it also is irrelevant. Even if the tenant had no legal right to occupy the land, she still would be the possessor of the land as to the friend, and she owed the friend the duties owed to a licensee.
99
A camper at a state park built a campfire within a fire ring on a calm day according to approved procedures. Just as a sudden strong wind arrived and blew some embers onto the grass, a large bear came out of the woods and charged at the camper. The camper ran to his car, which was some distance away, with the bear in close pursuit. By the time the bear left and the camper was able to exit the car and summon assistance, the embers in the grass had started a brush fire. The fire destroyed another camper's equipment and automobile at a nearby campsite before it could be extinguished. The other camper sued the camper who started the fire. At trial, the parties stipulated to the above facts. The plaintiff introduced into evidence a state statute that prohibited leaving any campfires unattended and required them to be extinguished immediately if any embers were blown out of the fire ring. At the conclusion of the proofs, both parties moved for a directed verdict. How should the court rule on the motions? A Grant the plaintiff's motion, because the statute was intended to prevent the type of harm that occurred, making the statutory standard applicable. B Grant the plaintiff's motion, because a brush fire caused by a campfire does not ordinarily happen in the absence of negligence by the camper. C Grant the defendant's motion, because the plaintiff has not established a prima facie case of negligence. D Deny both motions, because the jury should make the factual determination of whether the defendant was negligent.
C Grant the defendant's motion, because the plaintiff has not established a prima facie case of negligence. The court should grant the defendant's motion because the plaintiff has not offered sufficient evidence of negligence on the defendant's part to go to the jury. The standard of care in a negligence case may be established by proving that a statute imposing a specific duty applies instead of the more general common law duty of care. However, violation of the statute may be excused where compliance would cause more danger than violation or where compliance would be beyond the defendant's control. Here, the statute regulating campfires is applicable because (i) the plaintiff, a fellow camper, is in the class intended to be protected by the statute, (ii) the statute was designed to prevent the escape of a campfire, which is what occurred here, and (iii) the statute clearly specifies what is required. However, even though the statute would apply to the defendant's conduct and the defendant violated the statute, the violation will be excused here because he was fleeing for his life from a bear and had to take refuge in his car. Hence, the defendant will not be held to the statutory standard of care here. Because the plaintiff has offered no other evidence that the defendant was negligent, the defendant's motion should be granted. (A) is incorrect because, as discussed above, even though the statute was intended to prevent the harm that occurred, the defendant's violation of the statute will be excused. (B) is incorrect because the plaintiff cannot rely on an inference of negligence here to establish breach of duty. Res ipsa loquitur permits the trier of fact to infer breach of duty where the facts strongly indicate that the plaintiff's injuries resulted from the defendant's negligence, but here the undisputed facts as to how the brush fire was caused are inconsistent with a finding of negligence. (D) is incorrect because there is neither a reasonable inference of negligence nor evidence of negligence, given that the violation of the statute will be excused here. Hence, the case should not be submitted to the jury because the plaintiff has not established a prima facie case.
100
A mother whose young son was riding on a roller coaster by himself for the first time walked some distance away to get a snack. She heard a commotion by the ride and saw a crowd gathered. When she came closer, she heard someone close to the scene say that a young boy had fallen off and was killed. She was very distraught but could not see through the crowd. In fact, it was not her son but another boy who had fallen off. That boy had struck her son while falling from a higher part of the track, resulting in injuries to her son. The mother, who was pregnant, ultimately suffered a miscarriage as a result of accident-related stress. In a previous suit by the parents of the boy who was killed, the ride operator was found liable for negligence in operating the ride. Can the mother recover damages for her distress and resulting miscarriage in an action against the ride operator for negligent infliction of emotional distress? A Yes, because her son was injured as a result of the operator's negligence. B Yes, because she was closely related to someone in the zone of danger from the operator's negligence. C No, because she was not within the zone of danger from the operator's negligence. D No, because her son was not the boy who was killed.
C No, because she was not within the zone of danger from the operator's negligence. The mother cannot recover. A duty to avoid negligent infliction of emotional distress may be breached when the defendant creates a foreseeable risk of physical injury to the plaintiff. For a bystander who is outside the zone of danger from the risk of physical injury but who suffers emotional distress from seeing the defendant negligently injure another, most states allow recovery if: (i) the plaintiff and the person injured by the defendant are closely related; (ii) the plaintiff was present at the scene of the injury; and (iii) the plaintiff personally observed or perceived the event. Here, the mother was some distance away and not in the zone of danger, so her distress was not caused by any perceived danger to her. Nor can she recover under the bystander rules. While she is related to her son, who was injured in part by the ride operator's negligence, she was not present at the scene of the injury and did not personally observe or perceive the event. Rather, her distress was due primarily to hearing someone say that a young boy had been killed and not being able to get close enough to the ride to see where her son was. Hence, she cannot recover damages for negligent infliction of emotional distress. (A) is incorrect. The mother's distress was not caused by the injuries her son suffered, but rather at her fear that he was the one killed. While she may be able to recover damages for his injuries, those would be distinct from any damages for her emotional distress. (B) is incorrect. As discussed above, the plaintiff's close relationship with the injured person is only one of the requirements for a plaintiff outside the zone of danger to recover emotional distress damages. Because the mother was not present at the scene and did not observe the event, she cannot recover. (D) is incorrect. While her son was not killed, he was injured by the boy striking him while falling. Had the mother been present and observed the boy falling and hitting her son, she could recover damages for her emotional distress. Conversely, even if the boy who was killed were her son, her damages would be compensable instead through a wrongful death action.
101
In support of a charity fundraising luncheon, three volunteers independently brought to the event a casserole dish made with ground beef. Each of them had prepared her dish in her own kitchen. Another volunteer combined the dishes onto one large serving platter, from which guests at the luncheon served themselves. One of the guests became seriously ill with what the health department later determined to be a bacterial infection from undercooked beef that was in the combined casserole. The guest brought an action against the three volunteers who made the casserole dishes, alleging negligent preparation of the ground beef. Assuming that the guest can establish only the above facts and his injuries, who is likely to prevail in the action? A The guest, because, under the doctrine of res ipsa loquitur, he has established an inference of negligence. B The guest, because he can require each of the volunteers to prove that she was not the actual cause of the injury. C The volunteers, because the guest cannot establish which of the volunteers breached her duty of care. D The volunteers, because they all were donating their time and food to the event.
C The volunteers, because the guest cannot establish which of the volunteers breached her duty of care. The volunteers are likely to prevail. The elements of the prima facie case for negligence are (i) a duty owed to the plaintiff, (ii) breach of that duty, (iii) actual and proximate cause, and (iv) damages. Here, the volunteers each owed a duty of care to anyone consuming the food they prepared, including the guest. The facts indicate that at least one of the volunteers breached that duty by improperly preparing or cooking the ground beef. That breach of duty caused the guest to become seriously ill. However, he cannot establish which of the volunteers breached the duty of care and was a factual cause of his injury. Absent additional evidence, the guest will not prevail. (A) is incorrect. Res ipsa loquitur does not apply because more than one person supplied the casserole dish. The res ipsa loquitur doctrine enables a plaintiff to establish breach of duty just from the fact that an injury occurred that would not ordinarily occur unless someone was negligent. However, the plaintiff must establish evidence connecting a particular defendant with the negligence to support a finding of liability against that defendant. When more than one person was in control of the instrumentality that caused the injury, such as here, res ipsa loquitur generally may not be used. The doctrine sometimes has been applied to multiple parties involved in a joint venture, but that does not apply in this case. Each person volunteered independently to make the casserole dish, and each of them worked individually with their own recipes in their own homes, and another person combined the three batches into one casserole dish. (B) is incorrect. The alternative liability or unascertainable cause approach of Summers v. Tice applies when two or more persons have been negligent but it cannot be determined which one caused the plaintiff's injury. The court will shift the burden of proof to each of the negligent defendants to show that his negligence was not a factual cause of the injury. Here, however, there is no evidence that all of the volunteers were negligent; most likely, just one of them was. Hence, the volunteers will not be required to prove that they did not cause the guest's injury. (D) is incorrect. Regardless of the fact that they were just donating their time and food, each of the volunteers owed a duty of care to anyone consuming the food they prepared, as stated above. If the guest could prove that a particular volunteer undercooked the ground beef in her casserole, he could recover damages for his illness that was caused by the undercooked beef.
102
A tenant's apartment was without hot water for over a week because of a broken water heater, even though the landlord had been notified right away and the lease provided that the landlord would make repairs promptly. The tenant heated a large pot of water on the stove and started to carry it to the bathroom so she could warm up her bath. Her young nephew, who was visiting for a few days, came around the corner suddenly and collided with her. The hot water spilled on the nephew, burning him. Because the nephew had a rare blood disorder, the burns resulted in several of the nephew's toes requiring amputation. The nephew's guardian brought a negligence action against the landlord in a jurisdiction that follows the traditional rules for landowner liability. If the jury finds in favor of the landlord, what is the most likely reason? A The nephew, as a social guest of the tenant, was not owed a duty by the landlord. B The tenant's conduct was the actual cause of the nephew's injuries. C The landlord's conduct was not the proximate cause of the nephew's injuries. D The nephew's injuries were not foreseeable.
C The landlord's conduct was not the proximate cause of the nephew's injuries. The most likely reason to find in favor of the landlord is that proximate cause is not established. To establish proximate cause in indirect cause cases, where an intervening force combines with the defendant's conduct to cause the plaintiff's injury, the plaintiff must show that the defendant's negligence caused a foreseeable harm or caused a foreseeable reaction from a foreseeable intervening force. Intervening forces that produce a harm outside of the scope of what would normally be anticipated from the defendant's negligence are generally deemed unforeseeable and superseding. Such a superseding event will break the chain of causation and relieve the defendant of liability. Here, it is ultimately a question for the jury whether the landlord's failure to fix the water heater was a proximate cause of the nephew's injury. However, the jury could very well find that the landlord's failure to do so, even if negligent, is not a proximate cause of the nephew's burn injuries because the conduct of the nephew and the tenant are superseding forces. Given the invalidity of the other choices, lack of proximate cause is the most likely reason for the landlord to prevail. (A) is incorrect. Not only the tenant but also the landlord owes a duty to the tenant's nephew. The landlord's duty to maintain hot water in the tenant's apartment extends to guests of the tenant as well. (B) is incorrect. The tenant's conduct was not the only actual cause of the nephew's injuries under the "but for" test for actual cause. The landlord's failure to repair the water heater promptly was also an actual cause, because but for his failure to do so, the tenant would not have been carrying a pot of hot water to the bathroom. (D) is incorrect. Under the rule that a tortfeasor takes his victim as he finds him, it is irrelevant that the extent or severity of the plaintiff's injuries was unforeseeable. If the landlord were liable to the nephew, he would be liable for all of his injuries even though some were not foreseeable.
103
The plaintiff was driving inattentively when she had to swerve to avoid two other negligently driven vehicles at a busy intersection, and her car struck a light pole. The plaintiff, who was the only driver injured, sued one of the other drivers to recover damages in a jurisdiction that has adopted pure comparative negligence. The jury determined that she suffered injuries of $100,000 and was 50% at fault. If the plaintiff is awarded a recovery of only $25,000 from the defendant, what will be the most likely reason? A The defendant's fault was less serious than that of the other tortfeasor. B The plaintiff's fault was as great as the total negligence of the other two drivers combined. C The jurisdiction applies contribution based on a pro rata approach rather than proportional fault. D The jurisdiction has abolished joint and several liability.
D The jurisdiction has abolished joint and several liability. If the plaintiff recovers only $25,000, it will be because the jurisdiction has abolished joint and several liability. Under joint and several liability, when two or more tortious acts combine to proximately cause an indivisible injury to a plaintiff, each tortfeasor is liable to the plaintiff for the entire damage incurred. Hence, the plaintiff could recover $50,000 from the defendant if joint and several liability applied (her total damages reduced by the amount of her own fault that contributed to her injury). The facts do not state what percent of fault was assigned to the defendant, but given that the other choices are incorrect, the defendant's fault must have been determined to be 25%, so that $25,000 would be the plaintiff's recovery in the absence of joint and several liability. (A) is incorrect regardless of whether joint and several liability applies. If the defendant's fault were less serious than that of the other tortfeasor, he would be liable to the plaintiff for less than $25,000 in the absence of joint and several liability. If joint and several liability did apply, he would be liable to the plaintiff for $50,000-the full amount of her damages. (B) is incorrect because the fact that the plaintiff's fault equals the combined fault of the other two tortfeasors is relevant only in a partial comparative negligence jurisdiction. It is irrelevant in a pure comparative negligence jurisdiction. (C) is incorrect because the rule of contribution, regardless of whether it is based on a pro rata approach or a proportional fault approach, does not affect how much the plaintiff receives from a defendant. Rather, it enables a defendant who has paid more than his share of damages to the plaintiff under joint and several liability to seek recovery against any other joint tortfeasor for the excess paid.
104
An infant was injured in an automobile accident when the vehicle, driven by the infant's mother, left the roadway and rolled over down an embankment. At the time of the accident, the infant was buckled into an infant carrier car seat. The carrier was designed to snap into a base that was secured in the back seat by the rear center seat belt. Prior to driving, the mother had snapped the car carrier onto the base and pulled up on the car carrier's handle to ensure that the carrier was indeed secured in the base. When the rollover occurred, however, the carrier came loose from the base and was thrown about the inside of the vehicle, causing injuries to the infant's neck and face. The mother brought a products liability action on behalf of the child against the manufacturer of the car carrier, alleging that the manufacturer was negligent in the design of the base and seat combination. If the mother establishes at trial that the force of the rollover was enough to knock the seat loose, and that a reasonable, economically feasible alternative design existed, which of the following, if true, would be most helpful to the manufacturer's defense? A The mother violated a statute by traveling too fast for conditions, which caused the rollover accident. B No one had reported a car carrier coming loose in a rollover prior to this accident. C The car seat conformed with federal labeling requirements. D The retailer who sold the car seat was negligent in failing to notice the defect.
B No one had reported a car carrier coming loose in a rollover prior to this accident. The most helpful fact is that no one had reported this type of problem previously. The mother is alleging that the manufacturer's negligence led to the supplying of a defective product. To establish this, the plaintiff must show that those designing the product knew or should have known of enough facts to put a reasonable manufacturer on notice about the dangers of marketing the product as designed. Negligent design is not shown, however, if the danger of the product becomes apparent only after the product reaches the public. Hence, the absence of any previous complaints about this problem would be most helpful to the manufacturer. (C) is less helpful than (B). Although compliance with government safety standards, such as labeling, is evidence that the product is not defective, it is not conclusive evidence, and federal labeling requirements do not preempt state products liability laws on defective warnings. (A) will not support the manufacturer's defense. The answer choice suggests that the mother was contributorily negligent; however, the contributory negligence of a parent is not imputed to the child. (D) does not help the manufacturer. An intermediary's negligent failure to notice a defect does not relieve the liability of a manufacturer whose original negligence was the cause of the defect.
105
Two friends entered a bar looking to get money to pay off a loan shark, but with no plan how to do so. They struck up conversations with two women. The first friend left the bar, having induced one of the women to return home with him. Once in his house, the first friend told the woman that she would not be allowed to leave unless she gave him all of her money. Fearing for her safety, the woman gave him all of the cash she had in her possession. Meanwhile, the second friend remaining at the bar noticed that the other woman left her credit card on the counter. When the woman looked away, the friend picked up the credit card and put it into his pocket. Shortly thereafter, the woman realized her card was gone and accused the man of taking it. The man pretended to be insulted, slapped the victim, and went off with the credit card in his pocket. Which of the two friends can be convicted for common law robbery? A Both can be convicted. B The first friend can be convicted, but the second cannot be convicted. C The second friend can be convicted, but the first cannot be convicted. D Neither of the two friends can be convicted of robbery.
B The first friend can be convicted, but the second cannot be convicted. The first friend can be convicted of common law robbery, but the second cannot be convicted. Robbery is the taking and carrying away of the personal property of another from the other's person or presence by force or intimidation. In the instant case, the first friend committed a robbery when he threatened the woman and told her that she could not leave without giving him all of her money. The only issue would be whether the asportation element is satisfied. However, the asportation element is satisfied by any slight moving, and it is likely that the first friend moved the money at some point during the robbery. Thus, a jury could find the first friend guilty of robbery. This makes (C) and (D) wrong answer choices. In the case of the second friend, however, a conviction for robbery is unlikely. Although a close call, the taking or retention of the property was not by force or intimidation in the second case. The crime against the property was already completed when the man slapped the victim. Furthermore, the slap was not to prevent the woman from physically taking the credit card back; rather, it was a ruse used to deflect the accusation that the second friend took the credit card. Thus, (A) is wrong, and this provides a second reason why (C) is incorrect.
106
A farmer was in the middle of plowing his field when his tractor broke down. While attempting to repair it, he discovered that he needed a special wrench. He knew that his neighbor used the same type of tractor and kept a large cache of tools in his basement. Not wanting to make the long drive into town to buy one wrench that he probably would not use much, the farmer went to his neighbor's house to borrow the wrench. However, no one was home so he decided to look in his neighbor's basement for the wrench, thinking that he would return it before the neighbor came back. To gain entry, the farmer opened an unlocked window and climbed through the opening to the basement. Once inside, the farmer found the tool and took it with him to work on the tractor. His neighbor returned soon after and contacted the police when he discovered that one of his tools was missing. The police determined that the farmer took the tool and he was charged with burglary. What is the farmer's best defense against that charge in a common law jurisdiction? A Nobody actually lived in the basement. B The farmer knew that the house was unoccupied and would not have entered without permission had the neighbor been home. C The farmer entered the house through an unlocked window. D The farmer intended only to keep the wrench for a couple of hours.
D The farmer intended only to keep the wrench for a couple of hours. The farmer's best defense is that he intended only to keep the wrench for a couple of hours. Given that the farmer intended merely to borrow the tool, he lacked the intent to commit larceny, and thus would not be guilty of burglary. Common law burglary consists of: (i) a breaking; (ii) and entry; (iii) of the dwelling; (iv) of another; (v) at nighttime; (vi) with the intent of committing a felony therein. The farmer entered his neighbor's house intending to remove the tool. Thus, the facts indicate that the only felony he could have intended to commit at the time of entry would be larceny. Larceny consists of: (i) a taking; (ii) and carrying away; (iii) of tangible personal property; (iv) of another; (v) by trespass; (vi) with intent to permanently (or for an unreasonable time) deprive the person of his interest in the property. At common law, if the defendant intended to return the property within a reasonable time, and at the time of the taking had a substantial ability to do so, such an unauthorized borrowing would not constitute larceny. Consequently, if the farmer intended to keep the tool only for the short time to fix his tractor, then he did not intend to permanently deprive his neighbor of his interest in the wrench. Because the farmer thus lacked the intent to commit a felony in his neighbor's home at the time he entered, the farmer would not be guilty of burglary. (A) is incorrect because, for purposes of the crime of burglary, a structure is deemed to be a dwelling simply if any part of it is used regularly for sleeping purposes. Thus, the fact that nobody lived in the basement is irrelevant. (B) is incorrect because the fact that the house was unoccupied is irrelevant to his culpability for burglary. The crime of burglary would have been complete if the farmer had broken and entered his neighbor's home with the intent of committing a felony therein, regardless of whether the home was currently unoccupied. Consequently, the farmer's knowledge that the house was unoccupied would provide him with no defense to a charge of burglary. (C) is incorrect because the breaking needed for burglary requires only minimal force to gain entry. Opening an unlocked window is a sufficient use of force to constitute a breaking.
107
A husband discovered his wife in bed with a neighbor. The neighbor ran out the back door. The husband screamed at his wife and vowed revenge. After consuming several drinks to build up his nerve and becoming intoxicated, the husband went to his friend's house and borrowed a gun, and then went to the neighbor's house. The neighbor had neglected to lock his front door, so the husband walked in. He found the neighbor trembling in the living room and pointed the gun at him. The neighbor immediately began apologizing and pleading for his life, but then suddenly he pulled a switchblade knife from his pocket. As the metal flashed, the husband fired a single shot at the neighbor, killing him. What is the most serious crime of which the husband can be convicted? A Murder. B Manslaughter, because the husband was still distraught over finding the neighbor in bed with his wife. C Manslaughter, because his intoxication prevented the husband from having the requisite intent for murder. D No homicide crime, because the neighbor was about to attack him with a knife.
A Murder. The husband can be convicted of murder. Murder is the unlawful killing of another human being with malice aforethought, which may be (i) intent to kill, (ii) intent to inflict great bodily injury, (iii) reckless indifference to an unjustifiably high risk to human life, or (iv) intent to commit a felony. Intentional use of a deadly weapon authorizes a permissive inference of intent to kill. Here, the husband uttered statements of revenge, confronted the neighbor with a loaded gun, and intentionally shot him when he pulled out a knife-more than enough evidence for a jury to find that the husband had the malice aforethought necessary for murder. Furthermore, none of the issues raised in the other choices will suffice to excuse the killing or reduce it to voluntary manslaughter. (B) is incorrect because the husband will not be able to meet all four tests for establishing the provocation necessary to reduce a killing from murder to voluntary manslaughter. The husband would have to offer evidence that (i) a provocation existed that would arouse sudden and intense passion in the mind of an ordinary person such as to cause him to lose his self-control, (ii) the husband was in fact provoked and lost his self-control, (iii) there was not sufficient time between the provocation and the killing for the passions of a reasonable person to cool, and (iv) the husband in fact did not cool off between the provocation and the killing. The husband can easily establish the first two elements, because discovery of one's spouse in bed with another person is virtually always considered adequate provocation by common law courts. However, the time interval between the provocation and the killing was probably sufficient for a reasonable person to cool off, and the facts strongly suggest that the husband did in fact cool off-he consumed several drinks to build up his nerve and went to a friend's house to get a gun before confronting the neighbor. Thus, a jury would probably reject a claim of voluntary manslaughter here. (C) is incorrect because the husband's voluntary intoxication would not preclude a finding of intent for murder. Because the husband became intoxicated to build up his nerve to kill the neighbor, a court would probably find that his intent at the time he began drinking would apply to his later conduct. Furthermore, voluntary intoxication is no defense to crimes involving recklessness. The husband can still be liable for murder based on a state of mind of reckless indifference to human life-his conduct in becoming intoxicated and then confronting the neighbor with a loaded gun is sufficient to establish that state of mind. (D) is incorrect because the homicide will not be excused on self-defense grounds. A person may use deadly force in self-defense only if (i) he is without fault, (ii) he is confronted with unlawful force, and (iii) he is threatened with imminent death or great bodily harm. The husband is not without fault, however, because he initiated the assault and prompted the neighbor to pull the knife. His status as the aggressor deprives him of the right to use force in his own defense under these circumstances.
108
One night when a man was very drunk, he took one of his rifles, loaded it, and fired a bullet through his front door. Unbeknownst to him, at the time he fired the rifle, someone was driving by the house. The bullet went through the front door, through the window of the car, and killed the driver. The shooter was convicted of murder and appeals. He contends that there was insufficient evidence to support a finding of murder. How should the court of appeals rule? A That the evidence is sufficient to prove that the killing was intentional. B That the evidence is sufficient to prove that the killing was done with malice aforethought. C That the evidence is insufficient, because the shooter did not know that the driver was driving by his house and therefore he could not have acted intentionally. D That the evidence is insufficient, because at most the shooter's conduct constituted gross negligence and involuntary manslaughter.
B That the evidence is sufficient to prove that the killing was done with malice aforethought. The court of appeals should rule that the evidence is sufficient to prove that the killing was done with malice aforethought. Under the facts of this case, to support a finding of murder, the trial court would have to find that the shooter acted either intentionally or with malice aforethought. The facts clearly indicate that the shooter did not know of the car, so it cannot be said that he shot at it intentionally, and therefore (A) is not correct. "Malice aforethought" can mean that the defendant is acting with reckless indifference to an unjustifiably high risk to human life. There is little question that shooting a rifle through a front door shows reckless indifference to an unjustifiably high risk to human life. Thus, the question is whether the shooter's intoxication was sufficient to negate this state of mind. Voluntary intoxication is not a defense to crimes requiring malice, recklessness, or negligence. In the case of recklessness, if a defendant's lack of awareness results from voluntary intoxication, his conduct will nevertheless be deemed reckless. (C) is not a correct analysis of the issue, because his intentional act was firing the rifle, not shooting at the car. (D) is not the best answer, because although there is the possibility that the prosecution might have been able to show only gross negligence, there is sufficient evidence to support a finding of malice aforethought and murder.
109
A felon intending to rob a market waited outside until there were no customers. When he saw that the market was empty, he went inside and walked up to the counter with his hand in his jacket pocket to simulate a gun. Before the clerk could turn around to see what the felon wanted, another customer entered the market, startling the felon, who turned and ran out the door. Should the felon be found guilty on a charge of attempted robbery? A No, because he used no actual force on the clerk nor threatened any. B No, because he withdrew successfully from the robbery attempt. C No, because he never entered the zone of perpetration. D Yes, regardless of whether he totally abandoned his plan when the customer entered the market.
D Yes, regardless of whether he totally abandoned his plan when the customer entered the market. The felon should be found guilty of attempted robbery. With the specific intent to commit a robbery, the felon went beyond mere preparation for the offense. Once a person has gone beyond preparation, abandonment is not a defense to attempt. A criminal attempt is an act that, although done with the intention of committing a crime, falls short of completing the crime. The defendant must have the intent to perform an act and obtain a result that, if achieved, would constitute a crime. Also, the defendant must have committed an act beyond mere preparation for the offense. If a defendant has, with the required intent, gone beyond preparation, the general rule is that abandonment is not a defense. Even in those jurisdictions in which abandonment is a defense, such abandonment must be: (i) fully voluntary and not made because of the difficulty of completing the crime or because of an increased risk of apprehension; and (ii) a complete abandonment of the plan made under circumstances manifesting a renunciation of criminal purpose, not just a decision to postpone committing it or to find another victim. Here, the felon intended to take money from the clerk at the market by means of the threat of having a gun (i.e., by simulating a gun). Thus, the felon intended to commit a robbery. In walking up to the market counter while simulating a gun with his hand, the felon committed an act that was a substantial step toward commission of the intended crime, and that strongly corroborated his intent and purpose to commit the crime. All that was missing to complete the crime was for the clerk to turn around and, upon seeing the felon apparently armed, be forced to give up the money. Thus, the felon went far beyond mere preparation for the crime of robbery. Having gone beyond mere preparation, with the intent to commit robbery, the felon is guilty of attempted robbery. And, as explained above, even if the felon abandoned his plan when the customer entered the market, such abandonment will not afford him a defense. Even in those jurisdictions in which abandonment is a defense, the felon will not have a defense because his abandonment apparently occurred when the customer's sudden presence increased the risk of apprehension. Thus, the abandonment was not fully voluntary and did not really manifest a renunciation of criminal purpose. (A) is incorrect because, to be guilty of attempted robbery, events need not have progressed to the point where the defendant has used or threatened to use force. Because the felon had the requisite intent for attempt and went beyond mere preparation by standing at the counter and simulating possession of a gun, he should be found guilty of attempted robbery. (B) is incorrect because, as detailed above, abandonment of an attempt does not afford a defense, and in any event, the felon's abandonment here was not fully voluntary because the felon abandoned his plan due to an increased risk of apprehension. (C) is incorrect because a conviction of attempt does not require entry into a "zone of perpetration." Rather, a defendant (with the requisite intent) need only have committed an act beyond mere preparation. The Model Penal Code and most state criminal codes require that the act constitute a substantial step towards commission of the crime and strongly corroborate the actor's criminal purpose.
110
The police of a resort town discovered that a well-known cat burglar was currently living in town under an assumed name. To try to catch her in the act of burglary, an undercover officer approached the burglar with a plan for a burglary. The undercover officer told the burglar that he knew who she was and that he had a plan to steal jewels from someone staying in one of the town's resorts. The burglar initially refused the offer; however, after lengthy cajoling, she finally agreed to the plan. As the time for the burglary drew near, the burglar had second thoughts. Three hours before the theft was scheduled to take place, the burglar called the police and told them of the plan. She told them that she was not going to show up, but that her cohort (the undercover officer) would be there, and told them how to recognize the undercover officer. Is the burglar guilty of conspiracy at common law? A Yes, because the burglar made an agreement with the undercover officer to commit the theft. B No, because there was no agreement. C No, because the intended crime was never completed. D No, because the burglar effectively withdrew.
B No, because there was no agreement. There was an insufficient agreement for conspiracy liability at common law. Conspiracy consists of: (i) an agreement between two or more persons; (ii) an intent to enter into an agreement; and (iii) an intent to achieve the objective of the agreement. In addition, most states require an act in furtherance of the conspiracy, although an act of mere preparation will usually suffice. The agreement requirement means that the parties must agree to accomplish the same objective by mutual action. There must be a meeting of at least two "guilty minds"; i.e., between two or more persons who are actually committing themselves to the scheme. If one person in a two-party conspiracy is only feigning agreement, the other person cannot be convicted of conspiracy under the common law bilateral approach. Here, the officer, in his undercover capacity, was simply trying to set up a situation in which the burglar would be caught in the act. Thus, the undercover officer merely pretended to reach an agreement with the burglar to commit a burglary. At no time did the undercover officer actually commit himself to the burglary. Therefore, there could have been no agreement of two "guilty minds." Absent the requisite agreement, the burglar cannot be guilty of conspiracy. (A) is incorrect because, as explained above, there was no agreement sufficient for a conspiracy conviction, since the undercover officer never intended to commit the burglary. (C) is incorrect because completion of the substantive crime is not necessary for a conviction of conspiracy. Consequently, although the actual burglary was not consummated, this would not preclude a conviction of conspiracy to commit burglary. (D) is incorrect because withdrawal is not a defense to a charge of conspiracy. Note that, by withdrawing, a person may limit her liability for subsequent acts of the other members of the conspiracy. However, this question pertains to the burglar's potential guilt for conspiracy. As applied to the conspiracy charge, withdrawal will not afford a defense to the burglar.
111
A former construction worker became intoxicated one night and decided to move some heavy construction equipment that was parked at a construction site. Ignoring "no trespassing" signs, the worker jumped the fence and climbed into a large dump truck and started it up. However, even though he knew how to operate the truck, he quickly lost control of it due to his intoxication. It rumbled a short distance and crashed into a trailer housing the main office of the construction site. The worker is prosecuted for recklessly damaging property. A separate statute in the jurisdiction prohibited the unauthorized operation of construction equipment. Should the worker be found guilty of recklessly damaging property? A Yes, because his actions constituted an unlawful operation of the construction equipment. B Yes, because he was intoxicated while attempting to move the construction equipment. C No, because at most he could be found guilty of criminal negligence. D No, because he must have been aware that his conduct would cause the damage to the trailer in order to be found guilty of reckless damage.
B Yes, because he was intoxicated while attempting to move the construction equipment. The worker should be convicted because he was intoxicated when he damaged the trailer. The worker is being charged with reckless damage to property. A person acts recklessly when he consciously disregards a substantial and unjustifiable risk that a prohibited result will follow, and this disregard constitutes a gross deviation from the standard of reasonable care. Attempting to move a large piece of construction equipment while intoxicated should be considered reckless conduct because of the great potential for destruction arising from the size and destructive power of the construction equipment. Therefore, (B) is correct. (A) is incorrect because merely operating the equipment in violation of the law would not necessarily be reckless. For instance, here, the statute prohibiting unauthorized operation of the equipment likely was enacted to prevent untrained persons from driving dangerous equipment, but the worker was trained to operate the truck in question; thus, if not for the fact that he was drunk, his action would not necessarily have been reckless. Violating the statute may be evidence of negligence, but negligence is insufficient to establish recklessness. (C) is incorrect for the same reason that (B) is correct-driving the equipment while intoxicated constitutes reckless conduct. Although voluntary intoxication is a defense to a crime that requires purpose or knowledge, it is no defense to crimes involving recklessness. Even though the worker's condition may in fact have precluded him from being consciously aware of the risk, one who is not consciously aware of a risk only because he was voluntarily intoxicated will be deemed to have acted recklessly with regard to the risk. (D) is incorrect because it states the mental state for knowing conduct-if the defendant is aware that his conduct will necessarily or very likely cause a certain result, he acts knowingly with respect to that result. Recklessness is a lesser standard of fault.
112
The defendant was at a bar with a couple of friends when he spotted a man who had gotten the defendant's friend fired from a job several weeks ago. Since that time, the defendant had been verbally harassing the man and calling him names. This particular night, the defendant went over to the man's table and flirted with his girlfriend. The man was infuriated after having taken the defendant's abuse for so long, so he jumped up and attacked the defendant with a knife. The defendant could have easily run away, but instead grabbed the man and slammed him backwards. The man went crashing through the front window and was severely cut by the broken glass. He died before he could be taken to the hospital. The defendant will most likely be found guilty of what crime? A Murder. B Voluntary manslaughter. C Involuntary manslaughter. D None of the above.
D None of the above. The defendant would most likely be guilty of none of the listed crimes because the defense of self-defense makes his homicide excusable. A person may use deadly force in self-defense if (i) he is without fault, (ii) he is confronted with unlawful force, and (iii) he reasonably believes that he is threatened with imminent death or great bodily harm. In a majority of states, a person may use deadly force in self-defense even if this could be avoided by retreating. Here, the defendant's use of force was privileged because it reasonably appeared necessary to defend him from the man's unlawful attack, and the defendant had no duty to retreat under the majority view. Furthermore, the defendant can claim the privilege of self-defense even though his words triggered the fight-calling someone names would not be considered adequate provocation that would make the defendant the aggressor. Hence, because the defendant's use of force was privileged, he cannot be convicted of any of the listed crimes, making (A), (B), and (C) incorrect.
113
While in a department store, a man picked up a sweater and slipped it under his shirt. The man then started for the door. A woman, who also was shopping in the store, saw the man take the sweater. The woman grabbed a baseball bat from the sporting goods aisle and chased the man into the parking lot. The woman began swinging the bat at the man's head, hoping to knock him out and thus prevent the theft. The man pulled a knife from his pocket and stabbed the woman, killing her. The man was arrested and charged with murder. At trial, will the man most likely be found guilty? A Yes, because the evidence shows that he provoked the assault on himself by his criminal misconduct. B Yes, because the evidence shows that the man intended to kill or cause serious bodily harm. C No, because the jury could find that the man acted recklessly and not with the intent to cause death or serious bodily harm. D No, because the man was acting in self-defense.
D No, because the man was acting in self-defense. The man will be found not guilty because he was acting in self-defense. A person is privileged to use deadly force to prevent a crime only if it is an inherently dangerous felony. Shoplifting is not an inherently dangerous felony; thus, the woman's use of deadly force was not privileged. A person may used deadly force in self-defense if he (i) is without fault, (ii) is confronted with unlawful force, and (iii) reasonably believes that he is threatened with imminent death or great bodily harm. The man was without fault because, although he shoplifted the sweater, he did not initiate any violence or provoke the woman. He was confronted with the woman's unlawful force, and it was reasonable for him to believe that a baseball bat swung at his head threatened him with death or great bodily harm. Therefore, the man was entitled to defend himself against the woman's improper use of deadly force by using deadly force himself. (A) is wrong because the man did not initiate an assault. (B) is wrong because even if the man intended to kill the woman, his action was justified. (C) is a misstatement of the law and of the facts.
114
You are a criminal defense lawyer representing a client who has been charged with burglary. After the client was given Miranda warnings at the police station, he telephoned his mother and asked her to come to the station to post bail. Instead, his mother immediately called you. In the meantime, the police had begun questioning the client. Although he never told the police to stop the questioning, his answers were at first vague or clearly unresponsive. During the course of the questioning, you phoned the station and told the police that you had been hired to represent the client and would be there in half an hour. The police did not inform the client of your call. Ten minutes later, the client admitted to committing the burglary, and signed a statement to that effect prepared by the police. You arrived a few minutes later and advised the client to remain silent, but he told you that he had already signed a confession. How is the court likely to rule if you file a pretrial motion to exclude the confession as evidence at trial? A Grant the motion, because the police had a duty to inform the client that an attorney was coming to represent him. B Grant the motion, because the client has been deprived of his Sixth Amendment right to counsel. C Deny the motion, because the client's statement admitting the crime was voluntary. D Deny the motion, because the client waived his Miranda rights.
D Deny the motion, because the client waived his Miranda rights. The client's confession should be admitted because he waived his Fifth Amendment privilege against compelled self-incrimination after receiving Miranda warnings. Miranda v. Arizona requires that a person in custody be informed of his right to remain silent and his right to the presence of an attorney during questioning. A suspect may subsequently waive his rights by making a confession, as long as the waiver was knowing and voluntary. In this case, the client received proper Miranda warnings, and there is no indication that he did not understand what his rights were. Although his answers during questioning were initially unresponsive, he never asked for an attorney or indicated that he wished to remain silent, and he voluntarily confessed after a relatively short period of interrogation. Hence, he validly waived his Miranda rights. (A) is incorrect because the police had no duty to inform the client that an attorney was attempting to see him. The client's ignorance of his attorney's efforts has no bearing on whether he made a knowing waiver of his Miranda rights. (B) is incorrect because the client's right to counsel was not violated. Although the client does have a separate Sixth Amendment right to counsel under Escobedo v. Illinois because he has already been arrested and charged with the crime, this right would only be violated if the client, after being informed of his right to counsel, had requested an attorney or had been prevented from seeing his attorney. Here, he made no request to see an attorney-even when he called his mother-and you were allowed to see him immediately upon your arrival. Thus, he has waived his Sixth Amendment right to counsel. (C) is incorrect even though it is true that the client made a voluntary statement. Due process requires that for confessions to be admissible, they must be "voluntary," based on the totality of the circumstances, and here all of the circumstances indicate that the client's confession was voluntary. However, even a voluntary confession will be inadmissible if it was obtained in violation of Miranda rights. (D) is therefore a better choice than (C).
115
A woman was arrested, given Miranda warnings, and questioned about an armed robbery. After she asked to speak with an attorney, the police stopped questioning her about the robbery. Several hours later, the police gave the woman a fresh set of Miranda warnings and began to question her about a different robbery. She did not repeat her request for an attorney and instead made several incriminating statements about the robbery. At the woman's trial for the robbery for which she made incriminating statements, the prosecution seeks to have her statements introduced into evidence. If the woman's attorney objects on appropriate grounds, how should the court rule? A Overrule the objection, because the police did not badger the woman into confessing. B Overrule the objection, because the woman did not renew her request for an attorney after receiving fresh Miranda warnings. C Sustain the objection, because the police did not honor the woman's request. D Sustain the objection, because a confession obtained in violation of a defendant's Miranda rights but otherwise voluntary may be used against the defendant.
C Sustain the objection, because the police did not honor the woman's request. The court should sustain the objection because the police did not honor the woman's request for an attorney. At any time prior to or during a custodial interrogation, the accused may invoke a Miranda (Fifth Amendment) right to counsel. If the accused invokes this right, all questioning must cease until the accused is provided with an attorney or initiates further questioning himself. Thus, the police questioning of the woman about the robbery was improper, and she can have her statements excluded. (A) is incorrect. After receiving Miranda warnings, if an accused invokes the right to remain silent, the police cannot badger the accused. However, courts have ruled that if the police scrupulously honor the request, they can rewarn the accused and later resume questioning, at least about a different crime. Here, however, the accused did not simply invoke the right to remain silent, but rather requested an attorney. After such a request, as indicated above, all questioning must cease. (B) is incorrect because the accused does not need to reassert the right to an attorney; all questioning must stop until the accused is provided an attorney or resumes the questioning herself. (D) is incorrect. It is stating the rule for impeachment-a confession obtained in violation of a defendant's Miranda rights but otherwise voluntary may be used against the defendant for purposes of impeachment, but there is no such rule for use of the confession for other purposes.
116
A defendant is charged with beating a victim to death with a set of brass knuckles during the course of a fight in a tavern. The victim was found to have a pistol on his person at the time of the fight. During the course of the trial, the defendant took the stand in his own defense and testified that the victim threatened him with a gun and the defendant had hit the victim with the brass knuckles in self-defense. To rebut the defendant's claim, the prosecution wishes to place the bartender on the stand, who will testify that two years prior to the attack on the victim, she had seen the defendant approach a customer in her tavern from behind, put on a pair of brass knuckles, and strike the customer a severe blow on the side of the face with a brass-knuckled fist. The prosecutor has complied with all notice requirements regarding the bartender's proposed testimony. As soon as the bartender is sworn in, the defense attorney raises an objection. How should the court rule on the admissibility of the bartender's testimony? A Admissible, as substantive evidence that the defendant did not act in self-defense in beating the victim. B Admissible, to attack the defendant's credibility. C Inadmissible, because prior bad acts cannot be admitted to prove the defendant's propensity to commit the specific crime with which he is charged. D Inadmissible, because the defendant has not put his character in issue in this case.
C Inadmissible, because prior bad acts cannot be admitted to prove the defendant's propensity to commit the specific crime with which he is charged. The bartender's testimony is inadmissible because the defendant's prior fight in the tavern cannot be admitted to prove his propensity to beat someone to death. The basic rule is that when a person is charged with one crime, extrinsic evidence of his other crimes or misconduct is inadmissible if such evidence is offered solely to establish a criminal disposition. [Fed. R. Evid. 404(b)] The danger is that the jury may convict because of past conduct rather than because of guilt of the offense charged. While evidence of other crimes is admissible if it is independently relevant to some other issue (e.g., motive, intent, or identity), the defendant's prior fight appears to have no relevance other than as evidence of his violent disposition. It is therefore inadmissible. (A) is incorrect because it suggests that the bartender's testimony should be admitted to show the defendant's propensity for violence. As stated above, extrinsic evidence of his prior misconduct is inadmissible if offered solely to establish a criminal disposition. Evidence of specific acts of the person in question as demonstrating that person's character is permitted only in the few instances when character is itself one of the ultimate issues in the case. [Fed. R. Evid. 405(b)] The defendant's propensity for violence is not an ultimate issue in this case. (B) is incorrect because extrinsic evidence of the defendant's previous bad acts cannot be used to impeach him. A specific act of misconduct must be probative of truthfulness (i.e., an act of deceit or lying) and can be elicited only on cross-examination of the witness. Extrinsic evidence is not permitted. Therefore, testimony concerning the defendant's prior incident is not admissible for impeachment. (D) is incorrect because it is irrelevant. It is true that the defendant has not put his character in issue in this case simply by pleading self-defense. Even if he had, however, the prosecutor could not rebut by having a witness testify as to prior instances of misconduct; only reputation or opinion evidence would be admissible.
117
A defendant is being tried for the murder of a victim, which occurred during the burglary of the victim's house. In its case-in-chief, the prosecution seeks to offer evidence that the defendant, who was arrested several days after the crime, had been caught with several grams of cocaine in his car. All notice requirements have been met. This evidence will most likely be: A Inadmissible, because the defendant has not offered evidence of good character. B Inadmissible, because it has limited probative value and is unduly prejudicial. C Admissible, because it tends to show what the defendant did with the money he stole. D Admissible, because it tends to show that the defendant is capable of committing serious crimes.
B Inadmissible, because it has limited probative value and is unduly prejudicial. The evidence will likely be inadmissible. Other crimes and wrongdoings of a defendant are sometimes admissible to prove motive, opportunity, intent, preparation, plans, knowledge, identity, or absence of mistake [Fed. R. Evid. 404(b)], provided, however, that the probative value of the evidence is not substantially outweighed by prejudice or other Rule 403 considerations. On these facts, the probative value of possession of cocaine seems very slight and is highly prejudicial. Therefore, the evidence will probably be inadmissible. (A) is wrong. Other crime evidence is sometimes admissible to show motive, opportunity, etc., even if the defendant does not place his character in issue. Also, if the defendant offers evidence of good character, the prosecutor cannot for that reason alone offer extrinsic evidence of specific crimes. (C) is wrong. The issue is whether the defendant murdered the victim during a burglary, not what he did with the money. It is true that the evidence tends to show that the defendant had money, but the probative value that he committed the crime charged would be very slight and clearly outweighed by prejudice. (D) is wrong. If the evidence were offered to prove that the defendant is capable of committing serious crimes, it would be inadmissible character evidence.
118
The plaintiff sued a local restaurant, claiming that she injured her teeth, gums, and mouth when she bit into a hamburger that contained a large, jagged piece of glass. The plaintiff called to the stand a waiter for the restaurant, who testified that, when he heard the plaintiff scream, he looked in her direction and saw her remove a piece of glass from her bleeding mouth. On cross-examination, the defense asked the waiter, "Isn't it a fact that three months ago you were fired by the restaurant for serving drinks to your friends and not charging for them?" The waiter responded, "Yes, but I wasn't trying to steal anything. I just forgot to charge them." The defense then asked, "Isn't it a fact that last month you threw a rock through the plate glass window at the restaurant?" The waiter replied, "That's not true; I was there but I didn't throw the rock." The defense then offered the testimony of a witness who was prepared to testify that she saw the waiter throw the rock through the restaurant's window. Assuming that there have been no criminal charges filed as a result of the broken window, is the witness's testimony admissible? A No, because specific acts of misconduct that did not result in a conviction cannot be used to impeach a witness, either on cross-examination or through extrinsic evidence. B No, because specific acts of misconduct that did not result in a conviction cannot be established through extrinsic evidence. C Yes, as evidence of bias. D Yes, to establish that the waiter lied under oath.
C Yes, as evidence of bias. The witness's testimony is admissible to show bias. A witness can be impeached, either on cross-examination or by extrinsic evidence, with evidence that suggests a bias on the part of the witness, because it tends to show that the witness has a motive to lie. Evidence that the witness disliked the party he is testifying against would qualify as evidence of bias. The witness could testify that she saw the waiter throw the rock through the restaurant's window, because such evidence would help establish the waiter's bias against the restaurant. (A) is incorrect for two reasons: Federal Rule 608 provides that, if offered to impeach, prior bad acts may not be proved through extrinsic evidence but may be inquired into during cross-examination. Furthermore, if the prior bad act also helps establish bias, the courts have held that extrinsic evidence also will be admissible. (B) is incorrect as well for this latter reason. (D) is too broad a statement. In a broad sense, the evidence is offered to impeach the credibility of the waiter and to suggest to the jury that he may be lying under oath. However, the defense does not need to establish that the waiter lied; the reason the evidence is relevant and does not constitute impeachment on a collateral matter is because it is offered to show bias, making (C) the better answer.
119
A pedestrian walking on the sidewalk was struck by a car backing out of a driveway. The driver did not see the pedestrian because her neighbor’s bushes obscured her view of the sidewalk. The pedestrian was seriously injured and brought suit against the driver and the neighbor. The pedestrian also included the city in his lawsuit, alleging that the city failed to enforce its ordinance requiring homeowners to provide a clear view of sidewalks where they intersect with driveways. The trier of fact determined that the driver was 60% at fault, the neighbor was 30% at fault, and the city was 10% at fault. The jurisdiction has adopted comparative contribution in cases applying joint and several liability. Which of the following is a correct statement regarding liability? A The city is liable to the pedestrian for the full amount of the damage award. B Both the driver and the neighbor are liable to the pedestrian for 90% of the damage award. C Each of the three defendants are liable to the pedestrian for one-third of the damage award. D The driver is liable to the pedestrian for 60% of the damage award, the neighbor is liable for 30% of the damage award, and the city is liable for 10% of the damage award.
A The city is liable to the pedestrian for the full amount of the damage award. The city is liable to the pedestrian for the full amount of the damage award. Under joint and several liability, each defendant found by the trier of fact to be at fault for an indivisible injury is liable to the plaintiff for the entire amount of damages incurred, not just a portion of it. (Of course, multiple recovery is not allowed.) Thus, because the city has been found to be at fault for the accident, the pedestrian could recover the full amount of the damage award from the city. (B) is incorrect because both the driver and the neighbor are liable for the full amount of the damage award rather than 90%. Again, each defendant found to be at fault by the trier of fact for an indivisible injury is liable to the plaintiff for the entire amount of damages incurred. (C) and (D) are incorrect because they are not applying joint and several liability. Under a joint and several liability system, contribution allows a defendant who pays more than his share of damages to recover the excess from the other jointly liable parties; responsibility for the total damages is thus apportioned among those at fault. Traditional contribution rules require all defendants to pay equal shares regardless of their respective degrees of fault (choice (C)), while states with a comparative contribution system impose contribution in proportion to the relative fault of the various defendants (choice (D)). Nevertheless, this simply means that the city (assuming it paid the judgment award to the plaintiff) has contribution rights against the other defendants (i.e., it can recover from the others for damages paid in excess of the amount proportionate to its relative fault). This does not, however, mean that the defendants’ liability to the plaintiff is based on their relative fault. In fact, if one defendant were judgment-proof, the others would still be responsible for the full amount despite the fact that the judgment-proof defendant was mostly at fault.
120
A woman sued her ex-husband for installments due under their divorce property settlement agreement. The ex-husband defended the suit on the ground that the ex-wife was in breach of the agreement. However, the court granted judgment for the ex-wife. Now, the ex-wife is suing her ex-husband for other installments due under the agreement. The ex-husband raises the defense that the agreement was void and illegal because of fraud perpetrated by his ex-wife at the time the agreement was signed. The ex-wife moves to strike her ex-husband’s defense, claiming issue preclusion. Should the court grant the motion? A No, because the issue was never litigated. B No, because judgment on one installment does not bar subsequent actions on other installments. C Yes, because the husband should have raised the defense in the first action. D Yes, because the issue of fraud is never waived.
A No, because the issue was never litigated. The court should not grant the motion. Generally, issues actually litigated between the parties are binding on them in subsequent actions concerning the same claim. If the second suit involves a different claim, the first judgment may be invoked as to all matters actually litigated and determined in the first action, provided that the findings were essential to the first judgment. Here, the fraud issue was not actually litigated in the first action, and thus it can be raised with respect to the later installments. (B) is incorrect. The general principle recited in this answer is true for claim preclusion (unless there is an acceleration clause). However, even if claim preclusion does not apply, if the defendant raises the same issues as in the first case, he may be prevented by issue preclusion (collateral estoppel) from re-litigating those issues in that subsequent case. (C) is incorrect. Claim preclusion (res judicata) would bar claims arising out of the same transaction or occurrence. Here, claim preclusion does not apply because a defense, not a claim, is involved, and the second installment is a separate transaction or occurrence. (D) is incorrect because there is no such rule.
121
A police officer spent several hours using binoculars to observe an older man loitering on a college campus. The man, who was shabbily dressed and carrying a backpack, would approach certain students as they walked by him, and after a brief conversation with them, discreetly pass the students a small envelope in exchange for cash. The officer stopped the man under suspicion that he was dealing drugs. The man was not dealing drugs, but instead had been soliciting donations for a radical political group. The man grew irate when the officer opened one of the envelopes in question and discovered that they only contained literature about the group. The officer then frisked the man and discovered an illegal weapon taped to his leg. The officer immediately arrested the man. Which of the following best describes the situation? A The officer's actions were unlawful because the officer initially failed to get an arrest warrant before approaching the man, even though he had ample time to do so because the surveillance had been going on for several hours. B The man's arrest was unlawful because the officer was mistaken about the man selling drugs and thus the weapon would be inadmissible as fruit of the poisonous tree. C The officer's actions were lawful in stopping the man because the officer had reasonable ground to believe that the man was dealing drugs, but the subsequent search was unlawful once the officer realized his mistake about the drug dealing. D The stop, search, and subsequent arrest were lawful.
C The officer's actions were lawful in stopping the man because the officer had reasonable ground to believe that the man was dealing drugs, but the subsequent search was unlawful once the officer realized his mistake about the drug dealing. The stop was lawful but the patdown search was unconstitutional. A police officer has the authority to briefly detain a person for investigative purposes if he has a reasonable suspicion supported by articulable facts of criminal activity. Here, the officer watched the man engage in what reasonably appeared to be drug transactions, and he was justified in stopping and detaining the man to investigate. However, a police officer may pat down a detained person only if the officer has a reasonable suspicion to believe that the detainee is armed and dangerous. Here, the man grew irate when he was stopped, and he belonged to a "radical" group, but these facts are not enough to give the officer any reason to believe that the man was armed. Therefore, the patdown was unconstitutional, and the evidence found as a result must be suppressed. (A) is incorrect. Police generally need not obtain a warrant before arresting a person in a public place, even if they have time to get a warrant. A police officer may arrest a person without a warrant when he has probable cause to believe that a felony has been committed and that the person before him committed it. (B) is incorrect. If during an investigatory detention, the officer develops probable cause for arrest, the officer can proceed on that basis. Although the officer was incorrect in his initial suspicions of drug dealing, the illegal weapon he discovered during the patdown was sufficient to establish probable cause for the man's arrest. (D) is incorrect because, as stated above, an officer may not pat down a detainee for weapons absent a reason to believe the detainee is armed and dangerous.
122
During the defendant's trial for armed robbery, evidence was introduced establishing that a rifle was found in the trunk of the defendant's car when he was arrested. On direct examination, the defendant testified that when he was arrested and the rifle was found, he stated, "I keep that with me for protection." Should the court allow the testimony? A Yes, because it is a statement of a party. B Yes, because it is an excited utterance. C No, because it is hearsay not within an exception. D No, because it is a self-serving statement.
C No, because it is hearsay not within an exception. The court should not allow the testimony. The defendant's testimony is being offered to prove the truth of the matter asserted (i.e., that he keeps the rifle for protection) and is hearsay. Hearsay is a statement, other than one made by the declarant while testifying at the trial or hearing, offered in evidence to prove the truth of the matter asserted. A hearsay problem can arise even if the out-of-court declarant and the in-court witness are the same person, as is the case here. Because the statement is not within any exception to the hearsay rule, it must be excluded. (A) is incorrect because a party's statement is considered nonhearsay when offered by a party-opponent. This hearsay exclusion does not apply here because the defendant's testimony was elicited on direct examination-i.e., the defendant was introducing his own statement into evidence. (B) is incorrect because an excited utterance applies only if made in response to a startling event. Although the conditions of the search might have been stressful to the defendant, it is not the type of situation that gives rise to an excited utterance. (D) is incorrect. The fact that a statement is self-serving is not grounds for its exclusion. All of a criminal defendant's evidence can be considered self-serving in the sense that it furthers his claim of innocence.
123
A defendant is on trial for robbing a liquor store. The store clerk testified that the defendant came into the store at about 11 p.m., pointed a black gun with a silver grip at him, and demanded that he give him all the money in the cash register. The clerk testified that the store was well lit and that the defendant was not wearing a mask. The defendant's attorney called the clerk's employer to testify that when the clerk gave her a report of the robbery, he told her that the defendant pointed a silver gun with a black grip at him. How should the trial judge rule on the admissibility of this testimony? A Admissible, because it tends to show that the clerk is an unreliable witness. B Admissible, because it has bearing on the clerk's truthfulness and veracity. C Inadmissible, because it is extrinsic evidence of a prior inconsistent statement on a collateral matter. D Inadmissible, because it is hearsay not subject to any exceptions.
C Inadmissible, because it is extrinsic evidence of a prior inconsistent statement on a collateral matter. The testimony should be found inadmissible. Extrinsic evidence of a prior inconsistent statement may not be used to impeach a witness upon a collateral matter. The clerk testified that he could recognize the defendant's face, and so the color of the gun is not material to any issue in the case under the facts given. Moreover, a witness must be given the opportunity to explain or deny a prior inconsistent statement before any extrinsic evidence can be introduced for impeachment purposes. (A) and (B) are incorrect because, even if the evidence does have some bearing on the clerk's credibility as a witness, it will be excluded because it will possibly confuse the issues or because it is a waste of time. (D) is not correct because the defendant is not seeking evidence to prove the truth of any material issue, but to impeach the clerk. The hearsay rule does not apply
124
A plaintiff and a defendant were involved in a two-car collision. The defendant was indicted for drunken driving, a crime that carries a maximum sentence of two years' imprisonment. A witness to the collision testified before the grand jury. The defendant pled guilty to the charge of drunken driving and was fined $500. After the criminal charge was disposed of, the plaintiff sued the defendant for negligence and sought personal injury damages. In the negligence action against the defendant, the witness testified for the plaintiff that the defendant was on the wrong side of the highway at the time of the collision. On cross-examination, the defendant questions the witness about his sworn grand jury statement that the defendant was driving normally at the time of the accident, and the witness denies having made the statement. The defendant now seeks to have the witness’s grand jury statement admitted into evidence. Upon proper objection, should the court rule that the witness's statement before the grand jury is admissible? A Yes, for impeachment only. B Yes, as substantive evidence only. C Yes, for impeachment and as substantive evidence. D No, because it is hearsay not within any exception.
C Yes, for impeachment and as substantive evidence. The grand jury statement is admissible both as impeachment evidence and as substantive evidence. A prior inconsistent statement made under penalty of perjury at a prior trial or proceeding, or in a deposition, is admissible nonhearsay and thus may be used as substantive evidence as well as for impeachment. The credibility of a witness may be impeached by showing that the witness has, on another occasion, made statements that are inconsistent with some material part of his present testimony. Because it is made by the declarant other than while testifying at the trial or hearing, a prior inconsistent statement will usually constitute hearsay if offered to prove the truth of the matter asserted therein. Under such circumstances, the statement would be admissible only to impeach the witness. However, where the statement was made under penalty of perjury at a prior proceeding, including a grand jury proceeding, it is admissible nonhearsay—it may be considered as substantive proof of the facts stated. [Fed. R. Evid. 801(d)(1)(A)] The witness's sworn statement before the grand jury that the defendant was driving normally at the time of the accident is inconsistent with his later in-court testimony that the defendant was on the wrong side of the highway at the time of the collision. Thus, this statement can be inquired into by the defendant to cast doubt on the witness's credibility. Because the statement was made at a prior proceeding, and was made under oath, it is nonhearsay, and is also admissible as substantive proof that the defendant was in fact driving normally at the time of the accident. (C) is the only answer that reflects the fact that the grand jury statement may be used both for impeachment and for substantive purposes. (A) reflects the view of prior law, which was that prior inconsistent statements were limited to impeachment regardless of the circumstances under which they were made. As noted above, Federal Rule 801(d)(1)(A) deems such statements made under penalty of perjury at a prior trial or other proceeding to be nonhearsay and, as such, admissible as substantive evidence. (B) is incorrect because it precludes use of the witness's grand jury testimony for impeachment purposes. A prior inconsistent statement may always be used to impeach the credibility of a witness. (D) is incorrect for two reasons. First, even if deemed to be hearsay, a prior inconsistent statement would be admissible to impeach the witness. Second, because the prior inconsistent statement of the witness was made under oath at a grand jury proceeding, it is admissible nonhearsay.
125
A defendant was tearing up a stretch of pavement with a jackhammer when a rock flew up and struck a plaintiff in the head, causing him to be hospitalized. Because the jackhammer manufacturer had been out of business for several years, the plaintiff filed a lawsuit for his medical costs, lost work time, and pain and suffering solely against the defendant. At trial, the plaintiff's attorney calls a witness who testifies that, at the time of the incident, the defendant stated, "It was my fault." The defense attorney objects, but the judge overrules the objection on the ground that this is a declaration against interest. Are the grounds for the judge's decision correct? A Yes, because the statement subjected the defendant to tort liability. B Yes, because the defendant is a party to the litigation. C No, because the statement is not against an important interest. D No, because the defendant is available to testify.
D No, because the defendant is available to testify. The ground for the judge's decision is incorrect because the defendant is available to testify. The statement against interest exception to the hearsay rule requires that the declarant be unavailable as a witness. A declarant is unavailable if: (i) she is exempted from testifying because the court rules that a privilege applies, (ii) she refuses to testify concerning the statement despite a court order to do so, (iii) she testifies to not remembering the subject matter of the statement, (iv) she cannot testify because she has died or is ill, or (v) she is absent and the statement's proponent is unable to procure her attendance or testimony by process or other reasonable means. [Fed. R. Evid. 804(a)(1) - (5)] None of the bases for a finding of unavailability is present here. The defendant, the declarant whose statement is at issue, is available as a witness; thus, the judge was incorrect in basing his decision on this exception. (A) is incorrect because the fact that the statement subjected the defendant to tort liability, and thus was against her interest, is not enough; she must also be unavailable. Also, this choice implies that this exception would be available only if she were subjected to tort liability, not criminal liability. Although some courts so limit the exception, the Federal Rules include statements against penal interest within the parameters of the statement against interest. (B) is incorrect because the defendant need not be a party to the litigation for her statement to qualify as a statement against interest. Thus, her status as a party would not be a basis for deciding that the statement against interest exception applies here. Of course, this choice is also incorrect because her availability to testify precludes application of this exception. (C) is incorrect because the defendant's statement, which effectively acknowledges liability for the plaintiff's injury, is most certainly against an important pecuniary interest; i.e., it subjects her to the possibility of being held financially liable for the plaintiff's damages. Note that the judge correctly overruled the objection by the defendant's attorney, but for the wrong reason. The defendant's statement constitutes a statement by a party-opponent (commonly called an admission), which is an act done or statement made by a party and offered against that party and is nonhearsay under the Federal Rules. [Fed. R. Evid. 801(d)(2)] The defendant is a party, and her statement is a prior acknowledgment of the highly relevant matter of fault. For a statement by a party-opponent, the declarant need not be unavailable. (Don't be confused by the fact that, although the judge was correct in allowing the testimony as to the defendant's statement, the call of the question pertains to the grounds for the ruling, which were incorrect.)
126
A wife is on trial for the murder of her husband. She is accused of pushing him from the window of their 13th floor apartment; she claims he committed suicide. The wife called an operator for a suicide-prevention clinic to testify that the deceased husband had called the clinic on more than one occasion, each time telling the operator that he wanted to "end it all." Is the testimony admissible? A Yes, because the statements were made in "contemplation" of death. B Yes, because it tends to show that the husband intended to commit suicide. C No, because it violates the psychiatrist-patient privilege. D No, because no phone calls were made to the clinic by the husband on the day he died.
C No, because it violates the psychiatrist-patient privilege. The court should rule that the testimony is admissible. Under the state of mind exception to the hearsay rule, a declaration of intent to do something in the future is admissible as circumstantial evidence tending to show that the intent was carried out. Here, the husband's statements to the operator tend to show that the husband intended to commit suicide, so they are admissible to prove that he did so. (A) is incorrect because a "dying declaration" must concern the cause or circumstances of what the declarant believed to be his "impending" death. Although the husband made threats to end his life, there is no indication he believed his death was impending, and he did not discuss the cause or circumstances of his impending death. (C) is incorrect because the operator is not a psychiatrist, and there is no evidence that the husband assumed her to be one. (D) is wrong because the state of mind need not be as of the time of the incident to be relevant.
127
A defendant was on trial for burglary, and he took the stand in his own defense. On direct examination, the defendant vigorously denied having committed the burglary. Also on direct examination, the defendant stated that his last regular employment was as a bookkeeper for a corporation. On cross-examination, the prosecutor asked the defendant if he had embezzled funds from the corporation. The defendant denied that he had embezzled from the corporation or from anyone else. The prosecutor then wanted to call a police officer to the stand to testify that when she arrested the defendant for embezzlement, the defendant admitted to the officer that he had embezzled money from the corporation. Assuming that the defendant has not yet been tried on the embezzlement charges, may the prosecutor call the officer to the stand? A Yes, but only for purposes of impeachment. B Yes, both for impeachment of the defendant and as substantive evidence. C No, because the defendant has not yet been convicted of embezzlement. D No, because the evidence would be extrinsic.
D No, because the evidence would be extrinsic. The officer may not testify about the embezzlement because it constitutes impeachment by extrinsic evidence of a specific instance of misconduct. A specific act of misconduct offered to attack the witness's character for truthfulness can be elicited only on cross-examination. If the witness denies the act, the cross-examiner cannot refute the answer by calling other witnesses or producing other evidence. Because the alleged embezzlement is admissible, if at all, only as impeachment evidence, when the defendant denied it the prosecutor could not call the officer to testify. (A) is wrong because extrinsic evidence, such as the officer's testimony, of an instance of misconduct is not admissible. (B) is wrong because when a person is charged with one crime, extrinsic evidence of other crimes or misconduct is inadmissible to establish criminal disposition. Because nothing in the facts indicates that such evidence is being offered to prove something other than disposition (e.g., motive, identity, common plan or scheme), the officer's testimony is not admissible as substantive evidence. As discussed above, for impeachment, the prosecutor is limited to inquiry on cross-examination regarding the embezzlement. (C) is wrong because it implies that a witness may not be impeached with a prior act of misconduct unless it resulted in a conviction, and this is not the case. A witness may be impeached on cross-examination with a prior act of misconduct that is probative of truthfulness even if the misconduct did not result in a conviction. Here the problem is that the prosecutor is attempting to prove the bad act with extrinsic evidence, which is not allowed.
128
A truck driver is suing a car driver for injuries he suffered when their vehicles collided at an intersection controlled by stoplights. The truck driver called a witness to the accident to testify that he saw the driver of the car drive through a red light. On cross-examination, the car driver's attorney asks the witness, "Isn't it true that the car driver's ex-wife is paying $500 for your testimony today?" The truck driver's attorney objects. Should the objection be overruled? A Yes, because the question gives the witness an opportunity to explain or deny the allegation. B Yes, because the question is a proper form of impeachment. C No, because the question addresses a collateral issue. D No, because it is a leading question.
B Yes, because the question is a proper form of impeachment. The objection should be overruled. It is proper to impeach a witness by showing that the witness has a possible bias. Evidence that the witness is being paid to testify would be proper impeachment through bias. (A) states the requirement for introduction of a prior inconsistent statement; this is obviously inapplicable here. The credibility of a witness is not collateral, and so (C) is incorrect. Leading questions are proper on cross-examination; thus (D) is incorrect.
129
During a heated argument, a man punched his female coworker in the stomach after learning that she had been awarded "Employee of the Month." Angered, the woman responded by stabbing the man with a letter opener, which severed his aorta and caused his death. The police arrested the woman. After receiving her Miranda rights, she confessed to killing her coworker, but stated that she had not previously held any grudge against him. When asked why she stabbed him, she stated, "He just made me so mad when he said that I was a terrible employee and that he deserved the reward instead of me, and I just lost it when he punched me." The district attorney charged the woman with homicide. If the jury believes the woman's statement, of what crime is she most likely to be found guilty? A Murder. B Voluntary manslaughter. C Involuntary manslaughter. D No homicide crime.
B Voluntary manslaughter. The woman would most likely be found guilty of voluntary manslaughter in light of the provoking event. Adequate provocation will reduce a killing to voluntary manslaughter if the defendant was both reasonably provoked and actually provoked. The woman also would have to offer evidence that the insults and taunting, followed by a sudden punch in the stomach, would cause an ordinary person to be provoked, which is likely to be the case. Hence, her statement in conjunction with the facts makes this the best answer. (A) is incorrect. At common law, murder was the unlawful killing of a human being with malice aforethought. Malice aforethought could be established with any one of the following states of mind: (i) intent to kill; (ii) intent to cause serious bodily harm; (iii) the depraved heart killing (a reckless indifference to an unjustifiably high risk to human life); or (iv) the commission of a felony. While at least an intent to cause serious harm may be present here, the provocation will reduce the homicide to voluntary manslaughter. (C) is incorrect because involuntary manslaughter is a killing that results from criminal negligence, which theory these facts do not support. (D) is incorrect because, as discussed above, the woman would likely be guilty of voluntary manslaughter.
130
A shopper at a flea market noticed a vase and asked a nearby person how much it cost. The person replied, "One hundred dollars." The shopper paid him the money and took the vase. The person, who was not the owner of the vase but merely a bystander, absconded with the $100. What crime at common law has the bystander committed with respect to the $100? A Embezzlement. B Larceny by trick. C Obtaining property by false pretenses. D No crime.
C Obtaining property by false pretenses. The bystander has obtained property by false pretenses. In the crime of false pretenses, the defendant obtains title to the property by means of a false representation of a material present or past fact that causes the victim to pass title to his property to the defendant, who knows his representations to be false and intends thereby to defraud the victim. Under the circumstances, by replying that the price of the vase was $100, the bystander implied that he was its owner or was authorized to sell it, which he was not. This false representation induced the victim to pass title to his money to the bystander, who knew the representation to be false. Therefore, he is guilty of obtaining money by false pretenses and (D) is incorrect. (A) is incorrect because the bystander took title, not mere possession. Therefore, the crime is false pretenses, not embezzlement. The physical transfer of that cash without any limitations on its use was a transfer of title to the money. Therefore, the crime is the obtaining of money by false pretenses. (B) is incorrect because larceny by trick occurs when the defendant obtains possession of another's property by lying or trickery. Here, the bystander obtained title to, rather than mere possession of, the money, and is therefore not guilty of larceny by trick.
131
A homeowner decided to destroy his home by fire in order to collect the insurance. A neighbor's house was located a short distance from the homeowner's home. The homeowner knew that there was a strong wind blowing towards the neighbor's home; while he did not want to burn the neighbor's home, he nevertheless set fire to his own home. The fire department was unable to save the homeowner's house. They did manage to put out the fire moments before it spread to the neighbor's home, which suffered damage from smoke and soot. The jurisdiction's arson statute covers burning one's own dwelling as well as the dwelling of another, but is otherwise unchanged from the common law. If the homeowner is charged with attempted arson of the neighbor's home, is he likely to be found guilty? A No, because he did not intend to burn the neighbor's house. B No, because the fire was put out before any part of the neighbor's home was burned. C Yes, because he intended to burn his own home and took a substantial step toward burning the neighbor's house. D Yes, because he acted with malice and took a substantial step toward burning the neighbor's house.
A No, because he did not intend to burn the neighbor's house. The homeowner will be found not guilty because he did not have the requisite mental state. To convict a person for an attempted crime, the prosecution must establish that the defendant had an actual specific intent to cause the harm prohibited by the statute and committed an act beyond mere preparation in furtherance of that intent. Those elements-specific intent and act-are required regardless of the mental state required by the target offense. A person who took a substantial step towards commission of the crime but was only reckless with respect to the target offense could not be found guilty of attempt. The homeowner did not intend to burn the neighbor's home. Therefore, he cannot be guilty of attempted arson of the neighbor's home. (B) is wrong. The fact that the fire was put out before it burned any of the neighbor's home would not preclude a conviction of attempted arson if the elements of attempted arson were otherwise established. (C) is wrong. To be guilty of attempted arson of the neighbor's home, the homeowner must have intended to burn the neighbor's home. The doctrine of transferred intent does not apply to attempt. (D) is wrong. A specific intent to burn the home is required for attempted arson. While malice satisfies the state of mind requirement for the completed crime of arson, it will not suffice for attempt.
132
A drug smuggler had just returned home after smuggling in a large quantity of cocaine in the false bottom of his suitcase. As he was about to leave his house again to deliver the cocaine to his contact in the city, a police officer arrived with a trained drug-sniffing dog and asked him if he could come in and ask him some questions. The smuggler declined but the officer stepped into the doorway, and the dog immediately caught the scent of the cocaine and pulled the officer toward the suitcase in the hallway. Based on the dog's clear indication that the suitcase contained narcotics, the police officer opened the suitcase and found the cocaine. The smuggler was then arrested and the cocaine and suitcase seized. At a pretrial hearing, should the judge grant the smuggler's motion to suppress evidence of the cocaine in the suitcase? A Yes, because a warrantless search and seizure of items within the defendant's home is not permissible absent exigent circumstances. B Yes, because the search and seizure required a warrant. C No, because the cocaine was seized as a search incident to a lawful arrest. D No, because there is no reasonable expectation of privacy in the smell of one's suitcase.
B Yes, because the search and seizure required a warrant. The motion should be granted because the search and seizure required a warrant. To have a Fourth Amendment right, a person must have a reasonable expectation of privacy with respect to the place searched or the item seized or must prove that there was a physical intrusion into a constitutionally protected area. In the instant case, the place searched was the defendant's home, which clearly is a place in which a person has a reasonable expectation of privacy. Even the entry to a home is within the curtilage and protected against unreasonable searches. A police officer nevertheless has an implied license to approach the door of a home and knock, just like anyone else. However, the Supreme Court has held that bringing a drug-sniffing dog along violates the license. Since the presence of the dog at the doorway was a physical intrusion into a constitutionally protected area, the whole episode was unreasonable under the Fourth Amendment. Moreover, consent to enter was not granted and no other exceptions apply, so evidence of the cocaine should be suppressed. (A) is incorrect because it is too broad of a statement. A warrantless search and seizure of items in one's home may be based on other grounds, such as consent or plain view. (C) is incorrect because the search occurred prior to the arrest and thus was not a search incident to a lawful arrest (even assuming the arrest was lawful). (D) is incorrect. Although it has been held that one does not have a privacy interest in the smell of one's luggage, here the luggage was located in the defendant's home. The fact that the cocaine was still in the defendant's luggage, rather than in some other location within the house, is irrelevant. The search occurred within the defendant's home, which was not permissible under the facts.
133
A man beat his live-in girlfriend and fled. The girlfriend called the police and told them about the beating. She also told them that the man likely fled to his best friend's house. The police quickly obtained a valid arrest warrant for the man and went to the friend's house a few hours after the beating. On arriving, the police noticed that a car registered to the man was parked nearby. They knocked and the friend answered the door. The friend told the police that the man was not there. The police pushed past the friend and began searching for the man. They found the man hiding in a closet and arrested him. On searching the man after his arrest, police found cocaine in a small metal box in the man's pants pocket. The man was charged with assault and possession of cocaine. In a pretrial motion, the man moved to suppress the cocaine, claiming that it was the fruit of an unconstitutional arrest. Should the court grant the motion? A No, because the police found the cocaine after executing a valid arrest warrant. B No, because the search of the friend's house was valid under the community caretaker exception. C Yes, because the police entered the house without a search warrant or consent. D Yes, because the small metal box could not have felt like a weapon or contraband.
A No, because the police found the cocaine after executing a valid arrest warrant. The court should not grant the motion. The police may search a person after making a valid arrest. The arrest here was valid because the police had a valid arrest warrant. While the police should have obtained a search warrant to search for the man in the friend's home, the man does not have standing to complain of the illegal search. One may not raise a violation of another's constitutional rights at a Fourth Amendment suppression hearing. A person generally does not have standing to complain about a warrantless search of another's home unless the home was also his home or he was at least an overnight guest in the home. Here, the facts say that the man lived with his girlfriend, fled to the friend's house, and had been there a few hours. Thus, he lacks standing to complain about the warrantless search. (B) is incorrect. The community caretaker exception to the warrant requirement is not applicable here. The exception applies when police are acting to protect a person from imminent physical harm. Here, the beating had ended several hours before and the police had no reason to believe that anyone was in danger in the house. Therefore, the community caretaker exception does not apply. (C) is incorrect for the reasons stated above. While the police should have obtained a warrant to search the friend's home, the man lacks standing to complain about the warrantless search. (D) is incorrect because it is irrelevant. Incident to arrest, police may perform a full search. Only a pat down conducted during an investigatory detention is limited to revealing weapons or items immediately recognizable as contraband.
134
Two men were arrested while riding in a stolen automobile. They were taken to the police station, booked, and fingerprinted. They were then taken to an interrogation room. After the detective gave them their Miranda warnings, one of the men said, "Forget it. As soon as you check for outstanding warrants, you'll find out that I escaped from prison. Since I am going back anyway, it's a farce to deny that we stole that car." The other man said nothing, and the first man proceeded to write and sign a full confession. The man who remained silent pled not guilty to the charge of grand theft auto. At his trial the prosecutor seeks to introduce evidence to show that he did not deny that he stole the automobile when the other man told the police in front of him that he was a party to the theft. Should the court hold that this evidence is proper? A Yes, because silence in this situation is indicative of guilt and is an implied admission. B Yes, because the man who confessed had voluntarily waived his right to remain silent. C No, because an accomplice's evidence is inherently unreliable and unduly prejudicial. D No, because under this circumstance, the man who remained silent had no duty or responsibility to deny the allegation.
D No, because under this circumstance, the man who remained silent had no duty or responsibility to deny the allegation. The court should hold that this evidence is improper because the man who remained silent had no duty to deny the allegation here. A defendant in custody has no duty to speak at all, and the exercise of this constitutional right cannot be used against a defendant to show probable guilt. The Miranda warnings carry an implicit assurance that silence will carry no penalty. Thus, (A) is incorrect. (B) is immaterial, because the question only refers to the silent defendant's conduct and not the confessing defendant's statement. (C) suffers from the same problem, in addition to being a misstatement of the law.
135
A woman whose children attended a charter school learned that the children of the woman's neighbor who attended a parochial school received a hot lunch paid for, in part, through federal expenditures enacted under Congress's spending power. The charter school received no funding from the federal government and was not allowed to participate in the hot lunch program. The woman challenged this federal expenditure as a violation of the Establishment Clause. For her to bring the suit, at the very least what must the woman allege? A She pays federal income taxes and has children who are not eligible for the lunch program because they attend the charter school. B She pays federal income taxes and the use of federal funds in this manner is an improper taxing and spending method. C She pays federal income taxes and has children who are of school age. D She pays federal income taxes and has children who are of school age, and the use of federal funds in this manner is an improper taxing and spending method.
B She pays federal income taxes and the use of federal funds in this manner is an improper taxing and spending method. The woman must allege that she pays federal income taxes and that the use of federal funds in this manner is improper under Congress's taxing and spending power. In general, a taxpayer has no standing to challenge the expenditure of federal funds. The major exception to this rule is where the taxpayer alleges that the expenditure was enacted under Congress's taxing and spending power and exceeds some specific limitation on that power, in particular the Establishment Clause. Typically, neutral benefit programs that provide aid to private schools do not violate the Establishment Clause if they treat religious schools the same as other eligible private schools. This program, however, may be problematic because it provides a benefit to religious schools that isn't available to other schools. Thus, the woman would have standing to contest this federal expenditure. The woman does not have to have children to bring this challenge, let alone children in the charter school. However, she would have to be a taxpayer. Thus, (A), (C), and (D) are wrong.
136
Which of the following is not an exception to the general rule under the Commerce Clause prohibiting states from discriminating against out-of-state competition? A When the regulation is necessary to further an important, noneconomic state interest such as health or safety. B When the regulation furthers an important, noneconomic state interest such as health or safety and there are no reasonable alternatives available. C When the regulation limits access to privately owned in-state products. D When the state acts as a market participant.
C When the regulation limits access to privately owned in-state products. A regulation limiting access to privately owned in-state products would violate the negative implications of the Commerce Clause; there is no exception for such regulations. When a state acts as a market participant, it generally is not restricted by the Commerce Clause; it may favor its own citizens, such as by distributing state-owned resources only to residents or paying residents more for something than it would pay a nonresident (although such conduct might violate the Article IV Privileges and Immunities Clause). When a regulation is necessary to further an important, noneconomic state interest such as health or safety, it is excepted from the general rule of the negative implications of the Commerce Clause prohibiting discrimination against out-of-state competition. There also is an exception for when the regulation furthers an important, noneconomic state interest such as health or safety and there are no reasonable alternatives available. Actually, this is the same answer as above-if there are no reasonable alternatives, the method chosen is considered a necessary method.
137
The Federal Communications Commission ("FCC") issued a lengthy set of regulations regarding personal radar detectors. The regulations deal with the safety of such detectors and the frequencies on which they may operate, so as not to interfere with FCC-licensed radio and television stations or with radar used by commercial airliners and private aircraft. May a state constitutionally ban the use of radar detectors on its roads? A No, because the regulation of radio transmissions is within the purview of the FCC rather than the states, and state laws that attempt to regulate devices such as radar detectors are preempted. B No, because such a ban would burden interstate commerce. C Yes, because such a ban would operate only within the state. D Yes, because the state has a legitimate interest in regulating the use of radar detectors in order to promote safe driving.
D Yes, because the state has a legitimate interest in regulating the use of radar detectors in order to promote safe driving. The state may ban the radar detectors. States may regulate local aspects of interstate commerce as long as the local regulation does not conflict with, or is not preempted by, federal regulation and the regulation meets the following tests: (i) the regulation does not discriminate against out-of-state competition in order to benefit local economic interests, and (ii) the incidental burden on interstate commerce does not outweigh the local benefits of the regulation. In this case, the federal regulations do not conflict with the state ban and are not so comprehensive as to preempt nonconflicting state regulation. With regard to the two-part test, the first standard is met because the regulation is not discriminatory against out-of-state products (because it bans all radar detectors regardless of origin). The second part is a balancing test, in which the court will consider whether the regulation promotes legitimate state interests and whether less restrictive alternatives are available. Here, the ban clearly promotes the state's legitimate interest in highway safety by making it harder for speeding motorists to evade detection. Anything less than a ban would not be effective in preventing the use of the detectors, and their use makes radar, the state's best means of preventing speeding, much less effective. On balance, the ban's local safety benefits outweigh its burden on interstate commerce and transportation. (A) is incorrect because a field will be held to be preempted only where the federal statute is so comprehensive that it appears that Congress intended to occupy the whole field, and here the federal regulations concern only safety of the devices and use of frequencies. (B) is incorrect because it is too broad-not every law that burdens interstate commerce is unconstitutional. Rather, a balancing test will be applied. (C) is not as good an answer as (D). The fact that a state ban affecting interstate commerce operates only within the state does clear one hurdle to validity, because a state does not have the power to regulate interstate commerce. However, the mere fact that the ban would not operate outside the state does not go as far as (D) in addressing other Dormant Commerce Clause issues (e.g., is the law preempted, are its benefits to the state outweighed by its burden on interstate commerce).
138
What does the Fourteenth Amendment Privileges or Immunities Clause protect? A The rights of national citizenship B The rights of corporations C The rights of aliens D The rights of legal residents
A The rights of national citizenship The Fourteenth Amendment Privileges or Immunities Clause prohibits states from denying their citizens the rights of national citizenship, such as the right to petition Congress for redress of grievances, the right to vote for federal officers, the right to enter public lands, the right to interstate travel, and any other right flowing from the distinct relation of a citizen to the United States Government. Corporations, aliens, and legal residents are not citizens of the United States and are not protected by the Fourteenth Amendment Privileges or Immunities Clause.
139
Commercial fishing has long been one of the major industries of a coastal state. To protect the fishing industry and to promote the general welfare of the state's citizens, the legislature of the state enacted statutes requiring licenses for commercial fishing. An applicant for the license must pay a $300 fee and establish that he has been engaged in commercial fishing in the waters of the state for 10 years. A commercial fisherman residing in a neighboring state frequently takes his fishing boat up the coast. His favorite spot is approximately two miles off the coast of the legislating state. If the commercial fisherman challenges the constitutionality of the legislating state's statutes, should the court find the statutes constitutional? A Yes, because Congress has not enacted legislation regarding the subject matter of the statutes. B Yes, because economic and social regulations are presumed valid. C No, because less restrictive means are available. D No, because Congress has exclusive power to regulate foreign commerce, which includes commercial ocean fishing.
C No, because less restrictive means are available. The court should not find the statutes constitutional, because less restrictive means are available. The statutes violate the Privileges and Immunities Clause of Article IV, which prohibits discrimination against nonresidents with respect to essential activities (e.g., pursuing a livelihood) unless (i) the discrimination is closely related to a substantial state purpose, and (ii) less restrictive means are not available. Here, other controls could be placed on fishing without discriminating against out-of-state fishermen. (A) is wrong because, even though Congress has not acted in this area, the statutes would still be unconstitutional in light of the negative implications of the "dormant" Commerce Clause. Congressional silence is, therefore, irrelevant. (B) states a due process test which, even if applicable, would not preclude a finding of unconstitutionality on other grounds. (D) is wrong because the activity here does not involve foreign commerce-this is a dispute between one state and a citizen of another state.
140
A state law prohibits physicians from practicing medicine within the state without a state license. Among other things, the grant of a state license requires a physician to have been a resident of the state for at least one year. A physician moved to the state from a nearby state and immediately applied for a license to practice medicine. Although otherwise qualified, the physician's request for a license was denied based on the residency requirement. The physician brought suit, alleging that the residency requirement violated the United States Constitution. Will the physician likely succeed? A Yes, because the requirement violates the Privileges and Immunities Clause of Article IV. B Yes, because the requirement violates the Privileges or Immunities Clause of the Fourteenth Amendment. C Yes, because the requirement violates the Due Process Clause of the Fourteenth Amendment. D No, because the state has a compelling interest in furthering the welfare of its residents.
B Yes, because the requirement violates the Privileges or Immunities Clause of the Fourteenth Amendment. The physician will succeed. The Privileges or Immunities Clause of the Fourteenth Amendment prohibits states from denying their citizens the privileges and immunities of national citizenship. This includes the right to travel, and the Court has held that the right to travel includes the right of newly arrived citizens to enjoy the same privileges and immunities as are enjoyed by other citizens of the state. A state law that distinguishes between new residents solely on the length of their residency will serve no legitimate state interest. Thus, a law limiting medical licenses to persons who have resided in the state for a year runs afoul of the clause. (A) is incorrect. The Privileges and Immunities Clause of Article IV prohibits discrimination by a state against nonresidents. Here, the physician is a resident of the state discriminating against him. Hence, the Article IV privileges and immunities protection does not apply. (C) is not as good an answer as (B) because the law here involves discrimination based on duration of residency-making the Privileges or Immunities Clause more direct. (D) is incorrect because duration of residency does not impact the welfare of residents.
141
A state located in the southern half of the United States experienced a strong influx of retirees, due in part to its mild winters and in part to the generous health benefits that the state historically provided to its elderly residents who fell below the federal poverty line. The state's Office of Budget Management determined that the influx of retirees would bankrupt the state's health care benefit fund within five years. To preserve the fund and ensure the health of its citizens, the state revised its health care statute to make persons ineligible for coverage until they have lived in the state for at least one year. If a retiree who was denied benefits because she just moved to the state challenges the constitutionality of the statute in federal court, is she likely to prevail? A No, because the state has a compelling interest in maintaining the fiscal integrity of its health care fund. B No, because the states do not have a constitutional duty to provide health care benefits to retirees even if they fall below the federal poverty line. C Yes, because the requirement improperly burdens the right of interstate travel in violation of the Equal Protection Clause of the Fourteenth Amendment. D Yes, because the requirement deprives some retirees of certain privileges and immunities in violation of the Privileges and Immunities Clause of Article IV.
C Yes, because the requirement improperly burdens the right of interstate travel in violation of the Equal Protection Clause of the Fourteenth Amendment. The court will likely find that the one-year residency requirement is unconstitutional because it burdens the right to travel. An individual has a fundamental right to travel from state to state, and a state law that is designed to deter persons from moving into the state is likely to violate the Equal Protection Clause (as well as the Fourteenth Amendment Privileges or Immunities Clause). When a state uses a durational residency requirement (a waiting period) for dispensing benefits, that requirement normally should be subject to the strict scrutiny test, and usually will be found not to have satisfied the test. One such requirement that has been invalidated on this basis is a one-year waiting period for state-subsidized medical care, such as the one here. [See Memorial Hospital v. Maricopa County (1974)] (A) is incorrect. The Supreme Court has specifically held that a state's interest in fiscal integrity is not sufficient to justify a one-year waiting period for welfare or health benefits. (B) contains a true statement-the states have no constitutional duty to provide health care benefits for those below the poverty line. However, once a state chooses to provide such benefits, it may not do so in a manner that violates the Constitution, and, as explained above, the restriction here violates the right to travel. (D) is incorrect because the privileges and immunities protection of Article IV prohibits discrimination by a state against nonresidents when fundamental national rights are involved. Here, the restriction differentiates between residents. While that could violate the Fourteenth Amendment Privileges or Immunities Clause, Article IV is not implicated.
142
A woman was arrested, given Miranda warnings, and questioned about an armed robbery. After she asked to speak with an attorney, the police stopped questioning her about the robbery. Several hours later, the police gave the woman a fresh set of Miranda warnings and began to question her about a different robbery. She did not repeat her request for an attorney and instead made several incriminating statements about the robbery. At the woman's trial for the robbery for which she made incriminating statements, the prosecution seeks to have her statements introduced into evidence. If the woman's attorney objects on appropriate grounds, how should the court rule? A Overrule the objection, because the police did not badger the woman into confessing. B Overrule the objection, because the woman did not renew her request for an attorney after receiving fresh Miranda warnings. C Sustain the objection, because the police did not honor the woman's request. D Sustain the objection, because a confession obtained in violation of a defendant's Miranda rights but otherwise voluntary may be used against the defendant.
C Sustain the objection, because the police did not honor the woman's request. The court should sustain the objection because the police did not honor the woman's request for an attorney. At any time prior to or during a custodial interrogation, the accused may invoke a Miranda (Fifth Amendment) right to counsel. If the accused invokes this right, all questioning must cease until the accused is provided with an attorney or initiates further questioning himself. Thus, the police questioning of the woman about the robbery was improper, and she can have her statements excluded. (A) is incorrect. After receiving Miranda warnings, if an accused invokes the right to remain silent, the police cannot badger the accused. However, courts have ruled that if the police scrupulously honor the request, they can rewarn the accused and later resume questioning, at least about a different crime. Here, however, the accused did not simply invoke the right to remain silent, but rather requested an attorney. After such a request, as indicated above, all questioning must cease. (B) is incorrect because the accused does not need to reassert the right to an attorney; all questioning must stop until the accused is provided an attorney or resumes the questioning herself. (D) is incorrect. It is stating the rule for impeachment-a confession obtained in violation of a defendant's Miranda rights but otherwise voluntary may be used against the defendant for purposes of impeachment, but there is no such rule for use of the confession for other purposes.
143
A pedestrian was injured in a car accident involving two cars. The pedestrian filed a negligence action in federal district court against the first driver, seeking $100,000 in damages. The pedestrian is a citizen of State A and the first driver is a citizen of State B. The first driver then filed a third-party claim against the second driver, claiming that the second driver is responsible for half of the harm caused to the pedestrian and seeking to recover half of any liability the first driver is found to have to the pedestrian. The second driver is a citizen of State A. Does the federal court have subject matter jurisdiction over the third-party claim asserted by the first driver against the second driver? A Yes, because the court has supplemental jurisdiction over the third-party claim. B Yes, because the court has diversity of citizenship jurisdiction over the third-party claim. C No, because the amount in controversy in the third-party claim is too small. D No, because complete diversity of citizenship is lacking.
A Yes, because the court has supplemental jurisdiction over the third-party claim. The court has supplemental jurisdiction over the third-party claim. Diversity of citizenship jurisdiction is available when (i) there is complete diversity of citizenship, meaning that each plaintiff must be a citizen of a different state from every defendant; and (ii) the amount in controversy exceeds $75,000. In the instant case, the case is properly in federal court because diversity jurisdiction exists for the underlying claim (i.e., the claim by the pedestrian against the first driver), given that the pedestrian is from State A, the first driver is from State B, and the amount in controversy is $100,000. The third-party indemnity claim, however, cannot invoke diversity jurisdiction, even though the first driver is from State B and the second driver is from State A, because the amount claimed is $50,000. This makes (B) an incorrect answer choice. When the federal court has subject matter jurisdiction over one claim, it has discretion to exercise supplemental jurisdiction over related claims that derive from the same common nucleus of fact and are such that a plaintiff would ordinarily be expected to try them in a single judicial proceeding. (Essentially, this means that the supplemental claim must arise from the same common nucleus of operative fact as the claim invoking federal subject matter jurisdiction.) In the instant case, the claims of pedestrian (the underlying claim) vs. first driver and of first driver vs. second driver (the indemnity claim) are derived from the same accident, and thus both are derived from the same common nucleus of operative fact. There are restrictions on the use of supplemental jurisdiction when the use of supplemental jurisdiction would be contrary to diversity jurisdiction. In terms of third-party practice, claims by a plaintiff against an impleaded party may not use supplemental jurisdiction to circumvent the diversity statute. However, claims by a defendant are not listed among the restrictions; thus, supplemental jurisdiction is available to a defendant (third-party plaintiff) against a third-party defendant. Therefore, the first driver may use supplemental jurisdiction to have his claim against the second driver heard in federal court. (This would be true even if they were from the same state, so long as the claim was a true indemnity claim.) As a result, (A) is correct, and (C) and (D) incorrect.
144
A statute requires that any pilot who flies passengers for hire must have a commercial pilot’s license. An experienced pilot who had only a private pilot’s license and not the commercial license required by statute was hired by an attorney to fly her to another city to close a deal. The attorney knew that the pilot did not have a commercial license but the only commercial flight to the city was at an inconvenient time. The pilot flew the attorney through bad weather and landed safely, but because of a minor navigational error he landed at an airport a few miles away from the airport he was heading for. As he was going to start taxiing toward the hangar, another plane struck the aircraft. The student pilot of that plane had ignored the control tower’s instructions and gone onto the landing runway instead of the takeoff runway. The attorney was injured in the collision. If the attorney sues the pilot for her injuries, who will prevail? A The pilot, because the attorney knew he lacked a commercial license and voluntarily assumed the risk of flying with him. B The pilot, because the injuries to the attorney were caused by the negligence of the student pilot of the other plane. C The attorney, because the pilot violated a statute designed to prevent persons without commercial licenses from flying passengers for a fee, and such violation imposes liability per se. D The attorney, because the pilot landed at the wrong airport, and but for this mistake the attorney could not have been injured by the other aircraft.
B The pilot, because the injuries to the attorney were caused by the negligence of the student pilot of the other plane. The pilot will prevail because the conduct of the other plane’s student pilot constituted a superseding intervening force that relieves the pilot from liability. To establish a prima facie case for negligence, the attorney must show that the pilot’s breach of his duty to her was the actual and proximate cause of her injury. The attorney can establish actual cause because but for the pilot’s error, she would not have been injured. However, not all injuries “actually” caused by a defendant will be deemed to have been proximately caused by his acts. The general rule of proximate cause is that the defendant is liable for all harmful results that are the normal incidents of and within the increased risk caused by his acts. This rule applies to cases such as this, where an intervening force comes into motion after the defendant’s negligent act and combines with it to cause plaintiff’s injury (indirect cause cases). Here, the pilot’s navigational error did create a greater risk of collision with other planes in the process of landing, but it did not increase the risk of a plane using the landing runway to take off in disregard of the control tower’s instructions once the pilot was safely on the ground. Hence, the student pilot’s unforeseeable conduct was not within the increased risk created by the pilot’s negligence and constitutes a superseding force that breaks the causal connection between the pilot’s conduct and the attorney’s injury, enabling the pilot to avoid liability to the attorney. (A) is incorrect because assumption of the risk requires knowledge of the specific risk and the voluntary assumption of that risk. Although the attorney knew that the pilot lacked a commercial license, she also was under the impression that he was a very good pilot. There is no indication that she knew of or voluntarily assumed any risk. Certainly, she did not assume the risk of the type of harm she suffered. (C) is incorrect because even though the pilot may be negligent per se, he would not be liable per se. A clearly stated specific duty imposed by a statute may replace the more general common law duty of due care when: (i) the plaintiff is within the class to be protected by the statute, and (ii) the statute was designed to prevent the type of harm suffered. The statutory duty is clearly stated and arguably applies here because the attorney, as the pilot’s paying passenger, is within the protected class, and runway collisions and other pilot errors are what the license requirement is officially designed to prevent. There are no grounds for excusing the pilot’s violation of the statute, so the pilot’s conduct could be seen as “negligence per se.” This means that plaintiff will have established a conclusive presumption of duty and breach of duty. However, for the attorney to prevail, she must also establish actual and proximate causation. As explained above, the attorney will not be able to show that the pilot’s negligence was the proximate cause of her injuries. Thus, while she may be able to establish “negligence per se,” she has not made a case for “liability per se.” (D) is true as far as it goes. An act or omission to act is the cause in fact of an injury when the injury would not have occurred but for the act, and this injury would not have occurred but for the pilot’s landing at the wrong airport. However, the attorney must also be able to establish that the pilot’s conduct was a proximate cause of her injury. As noted above, the pilot’s conduct was not a proximate cause of her injury because the student pilot’s actions acted as a superseding intervening force.
145
A dog whistle manufacturer’s factory was located near a residential area. The manufacturer used the most effective methods for testing its whistles, but it was impossible to completely soundproof the testing area. A breeder of champion show dogs bought some property near the factory and raised and trained her dogs there. Although the whistles were too high-pitched to be perceived by human ears, they could be heard by the breeder’s dogs. Consequently, the dogs often were in a constant state of agitation. In a suit by the breeder against the manufacturer, what is the likely outcome? A The breeder will prevail on a trespass theory, because the sound waves are entering onto the breeder’s property. B The breeder will prevail on a nuisance theory, because the sound of the whistles is a substantial interference with the breeder’s use of her land. C The breeder will not prevail, because the sound of the whistles is not a substantial interference with the breeder’s use of her land. D The breeder will not prevail, because the manufacturer has acted reasonably in testing its whistles.
C The breeder will not prevail, because the sound of the whistles is not a substantial interference with the breeder’s use of her land. The breeder will not recover because there has been no substantial interference with her use or enjoyment of her land, nor has there been a trespass. A private nuisance is a substantial, unreasonable interference with another person’s use or enjoyment of her property. The interference must be offensive, inconvenient, or annoying to the average person in the community. It is not a substantial interference if it merely interferes with a specialized use of the land. Here, the testing of the dog whistles did not bother humans, and so it did not disturb the average person in the community. It is disturbing to the breeder’s dogs, but this affects only her specialized use of her land. Thus, the manufacturer’s actions do not constitute a private nuisance. (Nor do they constitute a public nuisance—an act that unreasonably interferes with the health, safety, or property rights of the community.) Therefore, (C) is correct, and (B) is incorrect. (A) is incorrect because the sounds reaching the breeder’s property do not constitute a trespass. A trespass is an intentional physical invasion of another’s land. Sound waves do not produce a physical invasion. Thus, the facts here do not support a basis for trespass. (D) is incorrect because the manufacturer could be found liable to the breeder even if it acted reasonably. In determining whether there is a nuisance, a court would consider the manufacturer’s care in testing its whistles, but that factor alone would not be determinative. If the activities were offensive to the average person, the court might still find there is a nuisance—even if it is impossible to do a better job of soundproofing. The court would have to consider the “reasonableness” of the interference, i.e., balance the injury against the utility of the manufacturer’s conduct.
146
An airline passenger nearly killed in a crash is suing the airline for personal injuries. To prove the extent of his injuries, the passenger offers a videotape taken by a local news station immediately after the crash that shows serious burns covering much of the passenger's face. The airline moves to exclude the videotape on grounds that its probative value is substantially outweighed by the danger of unfair prejudice. In making his ruling, which of the following is NOT appropriate for the judge to consider? A The videotape will make it more likely that the passenger will win the suit. B There are other methods of proving the passenger's damages. C The videotape can be restricted to its proper purpose by instructing the jury to disregard any possible emotional appeal. D The videotape will encourage the jury to decide the suit on an emotional basis.
A The videotape will make it more likely that the passenger will win the suit. The judge may not consider whether the videotape will make it more likely that the passenger will win. This question involves the use of the probative value/prejudicial impact balancing test contained in Federal Rule of Evidence 403. Under this test, relevant evidence is admissible unless its prejudicial impact substantially outweighs its probative value. Since the question asks which argument is least appropriate for the trial judge to consider, turn the question around and ask which of the choices are appropriate to consider when balancing the probative value and prejudicial impact of an item of evidence. The key to answering this question is to recognize that the only kind of prejudice that can properly be balanced under this test is unfair prejudice. The fact that evidence, if admitted, will help one party and hurt another party could be considered prejudicial in the most general sense, but it does not make the evidence unfairly prejudicial within the meaning of the balancing test. Thus, (A) is the correct answer. In determining whether the videotape, which shows serious burns over the passenger's face, should be excluded, it would be inappropriate for the trial judge to consider the argument that the videotape will make it more likely that the passenger will win the suit. That would not render the evidence unfairly prejudicial. Unlike (A), all the other choices contain arguments that would be appropriate to consider when applying the probative value/prejudicial impact balancing test. It is appropriate to consider whether the videotape would cause jurors to decide the suit on an emotional basis; that is the essence of unfair prejudice. Thus, (D) is incorrect. (C) is incorrect because it is likewise appropriate to consider whether an instruction to the jury to disregard any possible emotional appeal would be effective. (B) is incorrect because it is also appropriate to consider whether there are other means available to the passenger to prove damages that would bear on how necessary (i.e., how probative) the videotape would be to prove damages. If other evidence is available to prove damages, especially evidence less emotionally charged than the videotape, a judge should consider that fact in determining whether the prejudicial impact of the videotape substantially outweighs its probative value.
147
In a property dispute, a granddaughter claims that her grandfather gave her a deed to his home just before he died. The grandfather's son claims that the property is rightfully his by a previously executed will. At issue is the authenticity and content of the deed. The granddaughter begins to testify as to the content of the deed, but the son's attorney objects. Should the court sustain the objection? A Yes, because only the original deed itself is admissible to prove its contents. B Yes, unless the court is satisfied that the granddaughter is accurately testifying as to its contents. C No, if the judge is satisfied that the deed could not be found after a reasonable search. D No, if the deed is recorded, because the court can take judicial notice of the recorded deed.
C No, if the judge is satisfied that the deed could not be found after a reasonable search. The court should rule that the testimony is admissible if the deed could not be found after a reasonable search. When a proponent is attempting to prove the contents of a document, the best evidence rule applies and the original must be accounted for in order to introduce secondary evidence as to its contents. A proper foundation for the admissibility of secondary evidence may be laid by a showing that the original has been lost and cannot be found despite diligent search. Hence, (A) is incorrect. (D) is incorrect because the court may take judicial notice of court records, but the proponent of a recorded document must produce the document or record she desires introduced into evidence. A court has no power to take judicial notice of public records. (B) makes no sense because the court has no way of making such a determination.
148
The defendant, a used car seller, is on trial for criminal fraud, charged with selling used cars with major mechanical problems while representing to buyers that the cars were mechanically sound. The defendant claims that she had no knowledge the cars were not fit for sale. At trial, the prosecution offers evidence to show that, eight months prior, the defendant was fired from a different used car lot for knowingly selling defective automobiles with major mechanical problems. What is the best basis for admitting this evidence? A As evidence tending to show the defendant's criminal character. B As evidence of the defendant's criminal intent. C To impeach the defendant's credibility. D As evidence that the defendant is not a competent used car seller.
B As evidence of the defendant's criminal intent. The best basis for admitting this evidence is as evidence of intent. Evidence of past crimes or misconduct may not be admitted to show the accused's criminal character or her disposition to commit the present crime. However, such evidence is admissible to show the accused's knowledge or to show lack of mistake. This evidence, then, may be admitted to show the defendant's intent to commit criminal fraud. (A) is wrong because, as stated, the evidence cannot be admitted to show her criminal character. (C) is wrong because, even if the defendant had testified as to her lack of knowledge, she cannot be impeached by extrinsic evidence of prior bad acts. (D) is wrong because the prosecution is not alleging that the defendant is an incompetent used car seller but rather a dishonest one. Hence, the prosecution is offering the evidence to show criminal intent.
149
A defendant on trial for robbery took the stand in his own defense and testified that the robber was his neighbor. The jury acquitted the defendant based on this testimony. The neighbor was then indicted and brought to trial for the robbery. At that trial the prosecution called the defendant from the first trial to the stand, expecting him to incriminate the neighbor. Surprisingly, the defendant testified: "My neighbor didn't have anything to do with that robbery, but I know who did! I committed the robbery myself." When asked about the testimony he gave at his own trial, the defendant insisted he didn't remember anything about it. Finding her case in shambles, the prosecutor calls a juror from the first trial to the stand as a witness, who is prepared to testify that the defendant said at the first trial that the neighbor committed the robbery. On objection by the defense, should the court admit the juror's testimony? A Yes, to impeach the first defendant's credibility as a witness, but not as substantive evidence of the neighbor's guilt. B Yes, to impeach the first defendant's credibility as a witness, and as substantive evidence of the neighbor's guilt. C No, because former jurors are not competent to testify concerning cases upon which they sat as jurors. D No, because a transcript of the first defendant's testimony at his robbery trial is the best evidence.
B Yes, to impeach the first defendant's credibility as a witness, and as substantive evidence of the neighbor's guilt. The juror's testimony is admissible to impeach the first defendant's credibility and as substantive evidence of the neighbor's guilt. As long as the witness first is given an opportunity to explain or deny the statement, extrinsic proof of a prior inconsistent statement is admissible to impeach the witness's testimony. If the witness is subject to cross-examination and the prior inconsistent statement was made under penalty of perjury at a prior trial or proceeding, or in a deposition, it is admissible nonhearsay; i.e., it is admissible as substantive evidence. A witness is generally "subject to cross-examination" if he responds willingly to questions (even if he claims a lack of memory). In this case, the prior inconsistent statement was made under oath at the first defendant's trial and thus is admissible for its substance as well as for impeachment. (A) is incorrect because, as discussed above, the juror's testimony is admissible as substantive evidence of the neighbor's guilt. (C) is incorrect because jurors are incompetent to testify only (i) before the jury on which they are sitting, and (ii) in post-verdict proceedings as to certain matters occurring during jury deliberations. Since the juror is not testifying before the jury on which she was sitting and is not testifying about jury deliberations, she is a competent witness. (D) is incorrect because the best evidence rule does not apply to this situation. The juror is not being called to prove the terms of a writing or to testify about knowledge she gained from reading a writing. The facts she is testifying to exist independently of any writing; thus, the best evidence rule does not apply.
150
A defendant is on trial for a murder that occurred during a robbery at the victim's home. A witness helped the police artist compose an accurate depiction of the defendant. The witness was unavailable at the time of trial and the prosecutor offers the sketch into evidence. Is the sketch admissible? A No, under the best evidence rule. B No, as hearsay not within any exception. C Yes, as a record by a public employee. D Yes, as prior identification.
B No, as hearsay not within any exception. The sketch is inadmissible on hearsay grounds. Under Rule 801 of the Federal Rules, a prior identification can be admissible nonhearsay, and the sketch could be deemed a prior identification by the witness. However, the hearsay exclusion for prior identifications only applies when the declarant is testifying at trial and subject to cross-examination. Here the witness is unavailable; hence, this exclusion does not apply. (D) is therefore incorrect. (A) applies to documentary evidence and has no relevance to this question. (C) is likewise not applicable, because this exception applies only to information within the personal knowledge of the public employee. In this case, the public employee gained the knowledge from the hearsay statements of an absent witness.
151
A pedestrian is suing a driver after being run over by the driver. At trial, a copy of a hospital record is offered into evidence. The record indicates that hospital personnel took the pedestrian's statement 30 minutes after the accident, and includes the following statement: "The pedestrian's leg was run over by a car." The driver's counsel objects to the admission of the pedestrian's statement in the hospital record. Should the court admit the pedestrian's statement in the record? A Yes, even though hearsay within hearsay, because there is an applicable exception to each level of hearsay. B Yes, as a statement by a party. C No, as hearsay not within any exception. D No, because of the physician-patient privilege.
A Yes, even though hearsay within hearsay, because there is an applicable exception to each level of hearsay. The statement in the record is admissible. Hearsay within hearsay is admissible if each level of the hearsay-here (i) the pedestrian's statement to the admitting officer, and (ii) the admitting officer's statement in the record-comes within a hearsay exception. The business records exception to the hearsay rule, under Federal Rule 803(6), admits into evidence those records kept in the course of the regular conduct of any business, organization, occupation, or calling. Therefore, the statement about the pedestrian's medical condition would be admissible. The record itself (level (ii)) is kept in the ordinary course of the hospital's business of treating patients, and the pedestrian's statement about his medical condition (level (i)), while not made by someone with a duty to keep a record, is admissible both as a statement of present physical condition under Federal Rule 803(3), and as a statement made for purposes of medical treatment under Federal Rule 803(4). Thus, (C) is incorrect. (B) is incorrect. Statements by an opposing party are not hearsay under the Federal Rules; this hearsay exclusion does not apply when a party is introducing his own statement. (D) is incorrect because the physician-patient privilege is waived by the patient when he introduces his medical record at trial.
152
A wife is on trial for murdering her husband. At trial, the prosecution entered into evidence the fact that the wife fired the gun which killed her husband. The wife testified in her defense that her husband was threatening her with a knife when she picked up the gun and shot him. In rebuttal, the prosecution calls one of the officers who responded to the wife's 911 call right after the shooting. The officer will testify that the wife said, "I accidentally dropped my gun on the floor and it went off, killing my husband." Is the officer's testimony admissible? A Yes, as an excited utterance. B Yes, to impeach the wife and as evidence that she did not act in self-defense. C No, because of the wife's privilege against self-incrimination. D No, for the purpose of impeaching the wife, because the prosecutor did not call her attention to her statement to the officer on cross-examination.
B Yes, to impeach the wife and as evidence that she did not act in self-defense. The officer's testimony is admissible. Because the wife is a party, any statement she makes can be offered against her as a statement by an opposing party (commonly called an admission). Statements by an opposing party are not hearsay and therefore need not qualify for an exception to the hearsay rule. The statement is admissible substantively because it contradicts, and therefore tends to disprove, the self-defense theory. It is likewise a prior inconsistent statement which can be used to impeach the wife's credibility as a witness. (A) is incorrect because the statement made in this case was made to a police officer when he arrived at the house some time after the shooting. Moreover, the form of the statement shows it to be the product of reflection, rather than an unthinking response to an exciting event. (C) is incorrect because as a general rule the privilege against self-incrimination only permits an individual to refuse to give an answer to a question because it might tend to incriminate her; it does not require that an answer already given be held inadmissible later when offered in evidence. The exception is when an individual is held in custody and interrogated by the police without being given Miranda warnings and without waiving Miranda rights. However, Miranda warnings are not required when a statement is volunteered and not the product of interrogation, as here. (D) is incorrect. Extrinsic evidence of a prior inconsistent statement ordinarily may not be introduced unless the witness first is given the opportunity to explain or deny the inconsistent statement. However, this foundational requirement does not apply to statements by an opposing party.
153
For many years, civil service rules have provided that any member of the city's police department must serve a one-year probationary period before he or she will be considered a permanent employee. However, because the rules were enacted before the city's police academy was established, a prospective police officer now spends six months in the academy before being hired by the city. A graduate of the police academy was with the city police department for eight months after graduation when she was terminated. There were no city ordinances or state laws that required that she be given either a reason for the termination or a hearing, and she was given neither. The graduate brought suit against the city in state court because of the termination of her employment, alleging a violation of her due process rights. Which of the following would most likely give her a constitutional basis to require the city to give her a statement of reasons for the termination and an opportunity for a hearing? A No police officer had ever been terminated during probation except where there was actual cause. B The six months she spent in the academy must be considered as part of her probationary period. C The budget of the police department was recently increased to allow for the hiring of additional officers. D She was the only female police officer on probation and the only officer not given permanent employment.
A No police officer had ever been terminated during probation except where there was actual cause. The fact that no police officer has been terminated during probation except for cause may be enough for the graduate to show that she has a right to a hearing. Continued public employment may be a protected property interest if there is a clear practice or mutual understanding that an employee can be terminated only for "cause." If the graduate can establish this, she will be able to force the city to give her a reason for her termination and a hearing. (C) is incorrect because a general increase in police officer positions does not establish a specific right to employment for the graduate. (B) is wrong because there is nothing in the Constitution that requires that a city follow any particular method of employment practice, as long as the method chosen by a city does not violate some constitutional prohibition. The fact that the city's civil service law was not changed to reflect the additional period of time a police officer spends in the police academy does not create a right protected by the Fourteenth Amendment. (D) states facts that could give rise to an equal protection, rather than due process, claim.
154
The state passed a law stating that "only persons living with their parents or guardians who are bona fide residents of the state shall be entitled to free public education; all others who wish to attend public schools within the state may do so, but they must pay tuition of $3,000 per semester." A 15-year-old girl moved in with her friend so that she could attend the public schools in the state, and the state legislature passed the tuition statute just as she completed her junior year. The girl wants to complete her senior year in the state high school, but cannot afford to pay tuition. If the girl sues in federal court to strike down the tuition statute, is the court likely to rule that the statute is constitutional? A Yes, provided that the state can show that the statute is necessary to promote a compelling state interest. B Yes, unless the girl can show that the statute is not rationally related to a proper state interest. C No, because it infringes on the girl's fundamental right to an education. D No, because it interferes with the girl's fundamental right to interstate travel.
B Yes, unless the girl can show that the statute is not rationally related to a proper state interest. The court is likely to rule that the statute is constitutional. A bona fide residence requirement, such as this statute, that is not based on a suspect classification and does not limit the exercise of a fundamental right, is judged by the rational basis test. Thus, (A) is incorrect. The statute provides free education for all children who are bona fide residents of the state. Thus, it uniformly furthers the state interest in assuring that services provided for its residents are enjoyed only by residents. (C) is incorrect because education is not a fundamental right. (D) is incorrect because this statute does not impair the right of interstate travel. Any person is free to move to the state and establish residence there. This statute does not deter people from moving into the state.
155
The police department of a small city has jurisdiction within the city limits and over a defined portion of the surrounding rural communities within the county. A farmer lives in one of the rural communities receiving police protection from the city. The farmer does not pay any tax to the city directly, but a portion of the farmer's county property tax is turned over by the county to the city in order to support the city's police department. The farmer's property was vandalized several times over the past several months, and the farmer became unhappy with the police protection that the city was providing. After his complaints to the police department and city hall did not improve the situation, the farmer wanted to vote against the mayor in the next election, but a city ordinance provides that only residents of the city may vote in city elections. If the farmer brings a suit to compel the city to allow him to vote in the city's mayoral election, is he likely to prevail? A No, because the resident voting limitation appears to be rationally related to a legitimate government interest. B No, but only if the city can prove that the resident voting limitation, which affects a fundamental right, is necessary to a compelling interest. C Yes, because the resident voting limitation violates the Privileges or Immunities Clause of the Fourteenth Amendment. D Yes, because the resident voting limitation constitutes an instance of taxation without representation.
A No, because the resident voting limitation appears to be rationally related to a legitimate government interest. The farmer is not likely to prevail because the rational relationship test applies. Although the right to vote is a fundamental right, laws prohibiting nonresidents from voting are generally valid, provided that they meet the minimal scrutiny, or "rational basis," standard. Under this standard, a law will be upheld as long as it is rationally related to a legitimate government interest. Limiting the voters in a city's mayoral election to residents of the city serves the interests of efficiency and prevents persons with little personal interest in the city from voting. Thus, a court would likely uphold the ordinance. Answer (B) is incorrect because it sets out the strict scrutiny standard. While voting is a fundamental right and governmental action affecting fundamental rights generally is reviewed under strict scrutiny, that is not the case with laws limiting voting to residents. Answer (C) is incorrect. The Privileges or Immunities Clause of the Fourteenth Amendment prohibits a state from denying its citizens certain privileges or immunities of national citizenship. The voting restriction here does not affect any national right, except, perhaps, the right to travel. Even if that right is impacted here, as discussed above, the Supreme Court has approved limiting the right to vote to residents. Answer (D) is incorrect. While "no taxation without representation" was a rallying cry for the war of independence, it is broader than the constitutional doctrines applicable here.
156
Intending to encourage long-time resident aliens to become American citizens, a state passed a law denying numerous state and municipal jobs to persons who had been resident aliens for longer than 10 years. Those already in the state had to apply for American citizenship within a year after the law took effect. Persons who had acquired resident alien status prior to achieving the age of majority had until age 30 to acquire such status or be automatically disqualified from obtaining such a job. A 40-year-old man who has been a resident alien in the state for 15 years applied for a job as a police emergency response telecommunications expert. He had not filed for citizenship within the one-year grace period. May the state constitutionally rely on the statute to refuse to hire the man? A Yes, because a police department performs an integral governmental function and the state law does not discriminatorily classify resident aliens by race or ethnicity. B Yes, because aliens are not entitled to the privileges and immunities of state citizenship. C No, because the law does not apply equally to all aliens. D No, because the reasons for application of the law to the man do not appear compelling.
D No, because the reasons for application of the law to the man do not appear compelling. The law probably is unconstitutional as applied to the man in question. An equal protection issue is involved. Under the Equal Protection Clause, state classifications based on alienage are subject to strict scrutiny and so must serve a compelling interest to be constitutional. No compelling purpose seems to be present here. Thus, (D) is correct. (A) is incorrect because, although there is an exception from the strict scrutiny standard where a state or local government discriminates against aliens when hiring persons for jobs involving "self-government" processes, the job here (emergency communications for a police department) is a technical position and probably would not be found to be related to the self-government process; in any event, the statute applies to all positions and not just to jobs involving only the self-government process. (B) is incorrect because, although it is true that aliens are not entitled to the privileges and immunities of state citizenship, the law here is still unconstitutional under the Equal Protection Clause, as discussed above, which applies to aliens. (C) is incorrect because the discrimination would be unconstitutional even if it did apply to all aliens equally, as discussed above.
157
Which of the following is not considered a type of "fighting words" that may be forbidden by statute? A Statements likely to incite physical retaliation. B Conduct undertaken with the intent to cause fear of bodily harm (e.g., cross burning). C Statements meant to place a person in fear of bodily harm. D "Abusive language."
B Conduct undertaken with the intent to cause fear of bodily harm (e.g., cross burning). A statute that prohibits "abusive language" is overbroad and thus not an enforceable fighting words statute. Lots of language that is "abusive" is protected speech. Statements meant to place a person in fear of bodily harm and statements likely to incite physical retaliation are both classic examples of fighting words subject to government regulation. Conduct undertaken with the intent to cause fear of bodily harm (e.g., cross burning) is considered to be a form of fighting words, even though there is no actual spoken threat. This is because the conduct is intended to convey a message that is unprotected by the First Amendment.
158
After a state supreme court overturned the conviction in a murder case for failure to give proper Miranda warnings, a reporter asked the murder victim's father to comment on the case as he exited the supreme court building. The father made the following statement: "Each one of the so-called supreme court justices is worse than a murderer, because they make it possible for more sons and daughters to be murdered. I'd like to see every one of them strung up, like they should have done to the creep who was set free, and if someone will give me a rope I'll go in there and do it myself." A state statute proscribes, with criminal penalties, "the making of any threat to the life or safety of a public official for any act the official performed as part of the official's duties in office." Which of the following is correct regarding the statute? A The victim's father could constitutionally be punished under the statute, but only if the state supreme court justices heard the threats he made. B The victim's father could constitutionally be punished under the statute. C The victim's father could not be constitutionally punished under these circumstances, but the statute is constitutional on its face. D The statute is unconstitutional on its face.
C The victim's father could not be constitutionally punished under these circumstances, but the statute is constitutional on its face. The statute is not unconstitutional. True threats are not protected by the First Amendment. While the First Amendment requires proof that the defendant had some subjective understanding that their threats were of a threatening nature, a mental state of recklessness is sufficient, meaning that the speaker is aware that others could regard the statements as threatening violence and delivers them anyway. As long as the state interprets the statute in accordance with these principles, it will likely be deemed constitutional. Moreover, content-based restrictions on speech are permitted in cases where the speech creates a clear and present danger of imminent lawless action. A state can forbid advocating the use of force or of law violation if such advocacy (i) is directed to producing or inciting imminent lawless action, and (ii) is likely to produce or incite such action. Thus, a statute proscribing threats to the life or safety of a public official, such as the statute here, is valid. However, it cannot constitutionally be applied to the victim's father. It is doubtful that the father's words will be interpreted as a true threat of immediate harm. In context, the speech seems to be more a political commentary, which would be protected by the First Amendment. The father appeared to be merely venting his outrage. There was no indication that the father's words were inciting imminent lawless action or were likely to produce such action. Neither does it appear that the father was recklessly threatening the justices with harm or inciting anyone to storm into the court building. Thus, his speech was protected and (B) is incorrect. (A) is incorrect because the father cannot constitutionally be punished for the reasons stated above. Moreover, whether the justices actually heard the threats would be irrelevant if the threats were otherwise punishable. (D) is incorrect because the statute is valid on its face, as discussed above.
159
After the failure of a state bill granting gay and lesbian couples the right to marry, 30 students from a local university marched on the state capitol to protest, carrying signs with slogans such as "let gays marry" and "no religious tyranny." As they marched, about 15 people began following them, shouting anti-gay remarks. At the capitol building, they were met by 50 officers in full riot gear. The leader of the students addressed his followers in the park across the street, vigorously denouncing the legislature's actions, which caused the counter-protestors to become more vocal. The officer in charge told the leader that he must end his speech because a riot was about to start. The leader refused and was arrested and convicted of disorderly conduct. If the leader appeals his conviction on constitutional grounds, will the conviction be reversed? A Yes, because the leader's arrest constituted an interference with his First Amendment right to free speech. B Yes, because the leader's arrest constituted undue interference with the students' right to peaceably assemble under the First Amendment. C No, because the leader's speech caused an immediate and substantial threat to public order. D No, because the leader had a duty to obey the police officer since other avenues of communication likely were available
A Yes, because the leader's arrest constituted an interference with his First Amendment right to free speech. The speaker's conviction will be reversed. A park is a public forum. The government can limit rights of speech in such a forum only when there is a serious and imminent threat to the public order. It can restrict the speech of a speaker because of an unruly audience only in the rare case when the police are absolutely unable to control the crowd. [See Feiner v. New York (1951)] In this question, the conditions under which the police can prevent a speaker from continuing because of an unruly crowd have not been met. There were 50 police officers who would have been able to restrain or subdue anyone who appeared to be intent on committing violence. Hence, (C) is incorrect. (B) is incorrect. No one in the audience has raised any constitutional argument. The speaker probably cannot raise the audience members' First Amendment rights in this situation. (D) is incorrect. The state may not limit access to a public forum on the sole basis that there are other times and places where the right of free speech can be exercised. The state must show a more substantial reason.
160
A group of students held a demonstration against the proliferation of weapons in space in one of the city's parks. The defendant spoke at the demonstration, and to make a point during her speech, she walked over to one of the trash barrels and dumped the contents out on the ground. As she did so, she told her listeners, "This is what outer space is starting to look like, cluttered with the trash of nuclear weapons." The meeting broke up 15 minutes later. The students left the park peacefully, but no one bothered to pick up the trash or right the overturned barrel. The defendant was arrested pursuant to the city's littering ordinance, which carried fines of up to $500 and was strictly enforced. She was convicted and fined $500. The defendant brings an appropriate appeal to have her conviction set aside on constitutional grounds. Is the defendant likely to succeed? A No, because the anti-littering ordinance furthers an important governmental interest and is not aimed at communication. B No, because the defendant was convicted for her conduct rather than for what she said. C Yes, because the defendant's conduct constituted symbolic speech. D Yes, unless the city can prove a compelling interest in its anti-litter laws.
A No, because the anti-littering ordinance furthers an important governmental interest and is not aimed at communication. The anti-littering ordinance will be upheld because it furthers an important government interest unrelated to the content of the communication and is narrowly tailored to the furtherance of that interest. As a general rule, conduct that is intended to communicate is not immune from reasonable government regulation, even though it takes place in a public forum such as a park. The noncommunicative impact of speech-related conduct in a public forum can be regulated to further an important government interest independent of the speech aspects of the conduct as long as the incidental restriction on the ability to communicate that message is narrowly tailored to further the interest in question, so that alternative channels for communicating the message are available. The prevention of litter, as a means of maintaining public facilities in usable condition and protecting property values, is an important enough government interest to allow some type of regulation. The ban on littering is narrowly tailored to accomplish its purpose, unlike, for example, a ban on distributing leaflets that may end up on the ground. The regulation probably would not have precluded the defendant even from dumping the barrel if she had picked up the trash after her speech was over. (B) is incorrect because it is too broad; some speech-related conduct cannot be punished (e.g., burning a flag). The critical distinction is whether the offense relates to the communicative content of the conduct or to state interests independent of its communicative aspects. (C) is incorrect because the conduct aspect of symbolic speech can be regulated under the test indicated above. (D) is incorrect because the compelling interest standard only applies where the restrictions are based on the content of the message being communicated. Where the regulation is not based on content, the government need show only an important interest.
161
Which of the following is not a factor in the test for the validity of government action under the Establishment Clause when no sect preference is involved? A The action comports with historical practices B The government action is narrowly tailored to promote a compelling interest C The action faithfully reflects the understanding of the Founding Fathers The action faithfully reflects the understanding of the Founding Fathers - no response given D The action pursues a course of neutrality toward religion
B The government action is narrowly tailored to promote a compelling interest "The government action is narrowly tailored to promote a compelling interest" is not part of the test for determining the validity of government action under the Establishment Clause. This strict scrutiny test is applied to determine the validity of government action that includes a sect preference under the Establishment Clause. In determining whether government action is valid under the Establishment Clause, courts will consider whether the action is neutral with regard to religion. Next, courts will consider whether the government action accords with history and faithfully reflects the understanding of the Founding Fathers. If so, the action is unlikely to violate the Establishment Clause.
162
A state that had experienced a great deal of labor unrest in recent years enacted a law restricting striking workers from obtaining benefits from the state's child care assistance program while on strike. In contrast, there were no restrictions on receiving child care benefits for those who voluntarily left employment. A worker whose child care assistance benefits were suspended when she went on strike sued the appropriate state officials in federal district court, challenging the constitutionality of the law. Which of the following is the standard for the court to apply in reviewing the constitutionality of the law? A The state must show that the law is necessary to serve a compelling government interest because it interferes with the fundamental right to strike. B The state must show that the law is substantially related to an important government interest because it places an undue burden on women, who are statistically more in need of child care assistance than men. C The worker must show that the state did not have a rational basis for enacting the law, which is an example of economic or social welfare legislation. D The worker must show that the state did not have an important government interest in discriminating between those on strike and those who voluntarily left employment.
C The worker must show that the state did not have a rational basis for enacting the law, which is an example of economic or social welfare legislation. The worker must show that the state did not have a rational basis for enacting the law. The state statute here does not unduly burden any fundamental right or discriminate against a suspect class; rather, it simply withholds a state-supplied economic benefit to those engaging in strikes. (A) is incorrect. The right to strike has not been determined to be a fundamental right by the Supreme Court. (B) is incorrect. While gender classifications that intentionally discriminate against women will generally be invalid under the intermediate standard, any gender-related impact here is incidental and not a classification in the statute. (D) is incorrect because it applies the wrong burden.
163
To stabilize state corn prices, a state purchased large quantities of corn from resident farmers and converted the corn into biodegradable plastics. The state then sold the plastics to state residents at cost and to out-of-state residents at cost plus 25%. An out-of-state corporation purchased biodegradable plastics from the state at a cost substantially below the price other companies charge. Nevertheless, the corporation believes that it is unconstitutional for the state to charge out-of-state purchasers more than resident purchasers. The out-of-state corporation, therefore, brings suit in federal court challenging the state pricing scheme. Assuming that the court has jurisdiction, should it uphold the constitutionality of the pricing scheme? A Yes, because as a market participant the state is free to charge nonresidents more than residents. B Yes, because the state is selling plastics to nonresidents at prices substantially below that of other companies. C No, because the scheme discriminates against nonresidents in violation of the Commerce Clause. D No, because charging nonresidents more for plastics than residents pay violates the Privileges and Immunities Clause guaranteeing benefits of state citizenship.
A Yes, because as a market participant the state is free to charge nonresidents more than residents. The court will likely reject the company's challenge to the state pricing scheme. Although the Commerce Clause generally prohibits states from discriminating against out-of-state businesses to benefit local economic interests, the market participant exception applies here. The Commerce Clause does not prevent a state from preferring its own citizens when the state is acting as a market participant (e.g., buying or selling products, hiring labor, giving subsidies, etc.). Because the pricing scheme here involves the sale of goods, the state can constitutionally charge whatever prices it desires to whomever it desires. Therefore, (A) is correct and (C) is incorrect. (B) reaches the right result but for the wrong reason. But for the market participant exception, the state pricing scheme would violate the Commerce Clause for discriminating against nonresidents. It would not matter that the state's prices to nonresidents are lower than anyone else's price, because the state is charging its own residents an even lower price. (D) is incorrect. Although the Privileges and Immunities Clause of Article IV entitles citizens of each state to the privileges and immunities of citizens of the several states, and thus prohibits discrimination by a state against nonresidents, the Clause does not apply to corporations, such as the purchaser here. Therefore, the Clause cannot be the basis for the court's ruling.
164
To help alleviate discrimination in private contracts, Congress passed a bill providing: "It shall be unlawful to discriminate against minority race members in the making and enforcement of any public or private contract, of every kind whatsoever. Any person whose rights under this statute are violated may bring a cause of action against the party that has so violated the person's rights in the federal district court for the district in which he resides, seeking treble damages or $1,000, whichever is greater." Several large banks that have been accused of discriminatory loan practices challenge the federal statute. If the court finds that Congress had the power to enact the statute, the court most likely will find that the power arose from which of the following? A The Thirteenth Amendment. B The Contract Clause. C The Fourteenth Amendment. D The Commerce Clause.
A The Thirteenth Amendment. The court most likely will find that Congress had the power to enact the legislation under the Thirteenth Amendment. The Thirteenth Amendment simply provides that neither slavery nor involuntary servitude shall exist within the United States and gives Congress the power to adopt appropriate legislation to enforce the proscription. Since the amendment is not limited to proscribing state action, Congress may adopt legislation regulating private parties. Under the amendment, the Supreme Court has allowed Congress to prohibit any private conduct that Congress deems to be a "badge" or "incident" of slavery, and has upheld statutes regulating private contracts. [See, e.g., Runyon v. McCrary (1967)] (B) is not a good basis for the statute because the Contract Clause is a limitation on states' rights to modify contracts retroactively; it is unrelated to Congress's power to regulate private contracts. (C)-the Fourteenth Amendment-is incorrect. The Fourteenth Amendment prohibits states from discriminating on the basis of race; it does not extend to private conduct. [See United States v. Morrison (2000)] (D)-the Commerce Clause-might also be a basis for the legislation here, but it is not as good an answer as (A) because the commerce power is limited to transactions that either in themselves or in combination with other activities have a substantial economic effect on interstate commerce, and by its terms the legislation here can reach wholly intrastate transactions. The interstate commerce requirement is a limit on congressional legislation and no such limit is present under the Thirteenth Amendment. Therefore, the Thirteenth Amendment is a better basis for the legislation here.
165
An accountant employed by a federal agency was offended by jokes and cartoons that employees would post in the office cafeteria. Although none of the material violated any agency policy, the accountant lodged a number of complaints with his supervisor that went unheeded. Finally, the accountant posted his own notice chastising the agency for allowing cartoons the employee found offensive. The notice prompted a great deal of disruption in the office, particularly after it was posted on another employee's blog and received some media attention. The accountant did not have an employment contract with the agency and was not covered by any policy entitling him to dismissal only for certain grounds. Which of the following statements is most accurate regarding the agency's right to dismiss the accountant? A The accountant has a liberty interest in the exercise of his First Amendment rights that entitles him to a hearing to contest the grounds of his dismissal. B The accountant has a property interest as a public employee that precludes him from being fired without notice and an opportunity to respond. C The accountant has no right to a hearing because his statements were not an expression of views on public issues. D The accountant has both a liberty interest and a property interest that entitles him to a pretermination evidentiary hearing.
A The accountant has a liberty interest in the exercise of his First Amendment rights that entitles him to a hearing to contest the grounds of his dismissal. The accountant is entitled to a hearing because he has a liberty interest in the exercise of his First Amendment rights. If the accountant is fired, he has a right to a hearing to determine whether his First Amendment rights were violated by his dismissal. Under the Due Process Clause of the Fifth Amendment, a person has a liberty interest in the exercise of specific rights provided by the Constitution, including freedom of speech. A government employee may not be fired for expressing his views regarding public issues, but can be fired for speech that disrupts the employer's policies or undermines the employer's authority. Under the Court's expansive interpretation of what a public issue is in this context [see Rankin v. McPherson (1987)], the accountant's statement would probably qualify. At the very least, he can make enough of a showing that his termination violates his free speech rights to be entitled to a hearing on the issue under procedural due process principles. [See Givhan v. Western Line Consolidated School District (1979)] (B) is wrong because the accountant does not appear to have a property interest in his job. A public employee who is subject to removal only for "cause" has a property interest in his job and must be given notice of the charges against him that are to be the basis for his job termination, and a pretermination opportunity to respond to those charges. Here, however, the accountant did not have a property interest in his job. He could have been dismissed for no reason at all. (C) is wrong because the accountant is entitled to a hearing as long as he can raise a prima facie claim that his speech, which was regarding an issue important to the perception of his agency, was on a public issue and therefore protected by the First Amendment. (D) is wrong for two reasons: As discussed above, the accountant does not have a property interest in his job. Also, due process does not necessarily entitle him to a pretermination evidentiary hearing; a post-termination evidentiary hearing is probably sufficient. [See Cleveland Board of Education v. Loudermill (1985)]
166
Congress enacted a statute requiring state-supported institutions of higher education that provide federal student loan funds to their students to fund women's sports according to a complex formula intended to fairly support women's athletics and remedy past funding discrimination. Under the formula, a particular state military school will be required to allocate 25% of its athletic budget to its female athletics programs even though only 10% of the school population is female. A male student whose athletic program will be discontinued because of the budget allocation filed suit in federal court challenging the federal statute on various constitutional grounds. Is the court likely to find that the statute is constitutional? A No, because the government will be unable to prove that the discriminatory funding requirements of the statute are necessary to achieve a compelling government interest. B No, because the federal government does not have the power to dictate the budget allocations of state-supported educational institutions. C Yes, because remedying past discrimination is a legitimate government interest, and the student will be unable to prove that the statute's funding requirements are not rationally related to that interest. D Yes, because the government will be able to prove that the statute's funding requirements are substantially related to an important government interest.
D Yes, because the government will be able to prove that the statute's funding requirements are substantially related to an important government interest. The statute will likely be held constitutional because the attempt to compensate for past discrimination against women is substantially related to an important government objective. When examining federal government action involving classifications of persons, the Supreme Court, using the Due Process Clause of the Fifth Amendment, applies the same standards that it applies to state actions under the Fourteenth Amendment Equal Protection Clause. When analyzing government action based on gender classifications, the Court will apply an intermediate standard and strike the action unless the government proves, by an exceedingly persuasive justification, that the action is substantially related to an important government interest. Applying this standard, the Court has generally upheld classifications benefiting women that are designed to remedy past discrimination against women, because remedying past gender discrimination is an important government interest. Here, the federal statute establishes a formula designed not only to ensure current "gender equity" in funding of intercollegiate athletic programs but also to correct specific past inequities, and the school's required funding allocation in favor of women is designed to correct inequitable allocations by the school in prior years. Hence, even though the statute's allocation requirement may discriminate against the student and other males at the school, the government can satisfy its burden of showing a substantial relationship to an important government interest. (A) is incorrect because classifications based on gender are subject to an intermediate standard rather than a strict scrutiny standard; in other words, the government need not show that the classification is necessary to achieve a compelling interest, only that it is substantially related to an important interest. Furthermore, if the classification were one subject to strict scrutiny, remedying past discrimination based on the classification would probably be considered a compelling government interest. (B) is incorrect because Congress may "regulate" states through the spending power by imposing conditions on the grant of money to state governments. Even if Congress lacked the power to directly regulate the activity that is the subject of the spending program, attaching conditions on the spending does not violate the states' Tenth Amendment rights as long as the conditions are (i) clearly stated, (ii) related to the purpose of the program, and (iii) not unduly coercive. (C) is incorrect because it imposes the burden of proof on the wrong party and relies on the wrong standard. Because the statute results in gender discrimination, the government has the burden of proof, and that burden is to prove that the statute is substantially related to an important government interest.
167
To help reduce a rising crime rate among teenage boys in a city, a scout leader decided to organize an overnight jamboree to get teens interested in scouting. The scout leader met with the city's parks commissioner and requested a permit to camp at a large city park located on the oceanfront. The parks commissioner told the scout leader that a city ordinance prohibited large organized use of the park during the evening and all overnight camping. The commissioner explained that the city wished to keep the park open for general use during the evening, when most people were off work, and the park was cleaned overnight. The scout leader brought an action in federal district court, seeking to compel the city to allow overnight camping for this one special occasion. If the court determines that the ordinance is valid, what will be the basis for its decision? A The ordinance is rationally related to a legitimate government interest and burdens the First Amendment rights involved no more than is reasonable under the circumstances. B The ordinance is narrowly tailored to serve an important government interest and does not unreasonably limit alternative channels of communication. C The ordinance is substantially related to a legitimate government interest and burdens the First Amendment rights involved no more than is reasonable under the circumstances. D The ordinance is rationally related to a legitimate government interest and does not unreasonably limit alternative channels of communication.
B The ordinance is narrowly tailored to serve an important government interest and does not unreasonably limit alternative channels of communication. The court will base its decision on its determination that the ordinance is narrowly tailored to serve an important government interest and does not unreasonably limit alternative channels of communication. While the First Amendment protects the freedoms of speech and assembly, the government may reasonably regulate speech-related conduct in public forums through content-neutral time, place, and manner regulation. To avoid strict scrutiny and be upheld, government regulations on speech and assembly in public forums must be content neutral and narrowly tailored to serve an important government interest, and must leave open alternative channels of communication. Here, the ban on camping overnight in the park, a content-neutral regulation of a public forum, would be evaluated by the court using the standard in choice (B). (A) and (D) are incorrect because the rational relationship test is used for restrictions on free speech rights in nonpublic forums. Here, because the park is a public forum, the more restrictive test stated in (B) is used. (C) is incorrect because it misstates both parts of the standard
168
A state law provides that all persons who have been residents of the state for more than three years shall be entitled to free tuition at the state's main university. It further provides that persons who have resided in the state for three years or less shall pay the nonresident tuition rate, which is significantly higher. A student at the state's university who had been a state resident for less than three years filed a class action in federal court on behalf of himself and other similarly situated university students, seeking a declaration that the state statute is unconstitutional. When the case came to trial, the student had been a resident of the state for more than three years and was no longer required to pay tuition. By that time, a number of amicus curiae briefs had been filed in the case, some supporting and some opposing the student's position. Nevertheless, the state moved to dismiss the case as moot. Should the state's motion to dismiss be granted? A Yes, because the student is now a three-year resident. B Yes, because the student lacks standing. C No, because amicus curiae briefs have been filed. D No, because there is a live controversy.
D No, because there is a live controversy. There is a live controversy and the case is not moot. A federal court will not hear a case unless there is a real, live controversy at all stages of the proceeding, not merely when the case is filed. Because the student is no longer required to pay nonresident tuition, there is arguably no controversy and the case may seem moot. However, a class action is not moot, and the class representative may continue to pursue it-even if the representative's own controversy has become moot-because the claims of others in the class are still viable. Here, the student filed his suit as a class action for university students with less than three years' residency; undoubtedly some of those students will still have a real controversy at this time. Thus, the case is not moot. (A) is wrong although it states a true fact. (A) implies that the case should be dismissed because the student's claim is moot. As explained above, this is a class action and other members of the class have a viable case. Thus, even though the named student's case by itself would be moot, he may continue the case as a representative of the class action. (B) is wrong because standing (the requirement that a plaintiff have a concrete stake in the outcome of the case) is determined at the beginning of a lawsuit. At the beginning of this case, the student had standing because he had suffered an injury (i.e., had to pay nonresident tuition), caused by the government, that was remediable by the court. Thus, he had a concrete stake in the outcome of the case and had standing. (C) is wrong because it is irrelevant; the fact that amicus curiae briefs have been filed has no effect on mootness. A moot case will not be heard simply because amicus briefs have been filed.
169
A state adopted a statute making the ritual slaughter of chickens illegal. The legislative debates made clear that the purpose of the statute was to prevent unnecessary cruelty to animals. The religious leader of a church located within the state, whose core religious beliefs require ritual slaughter of chickens during worship services, brought suit to have the statute declared unconstitutional for violating her right to practice her religion. How will the court likely rule? A Uphold the statute, because of the compelling state interest involved. B Uphold the statute, because it is a neutral law of general application. C Invalidate the statute, because ritual slaughter is a core tenet of the plaintiff's religious beliefs. D Invalidate the statute, because it targets only ritual slaughter.
D Invalidate the statute, because it targets only ritual slaughter. The court will most likely strike the statute because it targets ritual slaughter. The First Amendment provides that the free exercise of religion shall not be abridged; however, the prohibition is far from absolute. The Supreme Court has stated that the amendment prohibits the government from outlawing religious beliefs and it has struck down a statute similar to the one here that outlaws conduct merely because it is religious (i.e., ritual slaughter of chickens is prohibited but not other instances of chicken slaughter), at least when the law is not necessary to achieve a compelling interest. (A) is incorrect because even if prevention of cruelty to animals is a compelling interest, a statute prohibiting all ritual slaughter of chickens probably is not necessary to achieve the goal of preventing unnecessary cruelty to animals; a statute prohibiting cruel methods of slaughter would serve such a purpose. (B) is incorrect because it states the rule that the Court would use for government action that does not target religious practice. A religiously neutral law of general application may validly proscribe general conduct; i.e., a law of general application will not be held invalid under the First Amendment merely because it happens to proscribe conduct that is required by one's religious beliefs. Neither will the state be required to provide religious exemptions from the statute. [Employment Division v. Smith (1990)] However, the statute here is clearly aimed at religious practices only, as it prohibits only ritual slaughter. Thus, (B) reflects the wrong standard to be applied. (C) is incorrect because it is irrelevant whether the religious practice interfered with is a core religious belief or merely a minor belief-the Court will not assess the centrality of religious belief, but will only inquire into whether a person's belief is sincere and whether the government action targets that belief.
170
Which of the following is required for a merchant's firm offer under Article 2? A Consideration paid by the offeree to keep the offer open B A written assurance signed by the offeror C Both the offeree and offeror are merchants D A specific time frame that the offer will remain open
B A written assurance signed by the offeror Under Article 2, a merchant's firm offer arises when a merchant offers to buy or sell goods in a signed writing and the writing gives assurances that the offer will be held open. If no specific time frame is stated in the offer, a merchant's firm offer will remain open for a reasonable time (but in no event may such period exceed three months). For a merchant's firm offer, it is not necessary that both parties be merchants; only the offeror must be a merchant. A merchant's firm offer is enforceable even if no consideration has been paid by the offeree to keep the offer open
171
A small business owner decided to retire, so she offered her long-time employee a chance to buy the business for $1 million. She promised in writing to keep the offer open to him for 90 days and to give him enough time to secure financing once he accepted the offer. Over the next few days, the employee cashed out all his retirement accounts and took a second mortgage on his home to raise the funds to purchase the business. When he approached the business owner to discuss the details of the sale, she said that she changed her mind and was revoking her offer because she did not want to retire after all. Was the owner's revocation of her offer proper? A Yes, because it was an offer that could be revoked at will. B No, because the owner created an option contract by promising to keep the offer open for 90 days. C No, because the employee detrimentally relied on the offer. D No, because the offer constitutes a merchant's firm offer.
A Yes, because it was an offer that could be revoked at will. The owner's revocation of her offer was proper because the offer could be revoked at will. Generally, offers can be revoked at will by the offeror, even if she has promised not to revoke for a certain period of time. There are limitations on the offeror's power to revoke, but none of those exceptions apply in this case. (B) is incorrect because an option contract requires that the offeree give consideration for the promise by the offeror to keep the offer open, and no consideration is indicated by the facts. (C) is also incorrect. Detrimental reliance can limit an offeror's power to revoke where the offeror could reasonably expect that the offeree would rely to his detriment on the offer, and the offeree does so rely. However, this usually is limited to those situations in which the offeror would reasonably contemplate reliance by the offeree in using the offer before it is accepted; e.g., when a general contractor uses a subcontractor's bid in making its own offer. Here, the offer itself included a promise by the owner to give the employee time to secure financing after the offer was accepted. Therefore, the owner had no reason to anticipate that the employee would take immediate steps to raise the purchase money before he even accepted the offer. (D) is incorrect because these facts are not an example of a merchant's firm offer. A merchant's firm offer does not apply to any offer by a merchant; it applies only to an offer under the UCC for the sale of goods where a signed writing gives assurances that the offer will be held open.
172
An art collector was interested in buying a painting from his neighbor. The neighbor told the collector that he could have the painting for $30,000. The collector wanted to think the purchase over. Therefore, the two agreed in writing that the neighbor would keep the offer open for 30 days in exchange for $500, which the collector paid. The terms of the written agreement provided that the offer would expire at 11:59 p.m. on September 30 if the collector failed to accept by that time. On September 20, the collector telephoned his neighbor and told him, "The more I think about it, the less I think that I want your painting." The neighbor responded, "That's your decision to make." On September 26, one of the neighbor's friends was visiting him, saw the painting, and offered his friend (the neighbor) $35,000 for it. On September 27, the neighbor mailed a $50 check to the collector with a letter stating that he was terminating his offer to the collector regarding the painting and refunding 10% of the money that the collector paid him to keep the offer open. He mailed the letter at 11:59 p.m. on September 27. The collector received the letter at 11:30 a.m. on September 29. On September 28, at 9:30 a.m., the collector mailed a letter to his neighbor stating that he had decided to purchase the painting and a certified check in the amount of $30,000 was enclosed. Two hours later, the neighbor sold the painting to his friend for $35,000. The neighbor received the collector's letter on October 1 and immediately mailed the check back to the collector. Can the collector maintain a successful legal action against his neighbor? A Yes, because the neighbor sold the painting after the collector's effective acceptance, and before the neighbor's revocation became effective. B Yes, because in his revocation the neighbor did not refund the full $500 to the collector. C No, because the neighbor effectively revoked his offer before the collector accepted. D No, because the collector's power to accept lapsed before he effectively accepted.
D No, because the collector's power to accept lapsed before he effectively accepted. The collector's power to accept lapsed because the option contract specified that the offer would expire at 11:59 p.m. on September 30. Hence, the power had to be exercised prior to that time and it was not. The mailbox rule does not apply to the exercise of options. In such cases, acceptance is effective when received by the offeror, here on October 1. Thus, (D) is correct. (A) is wrong because, for the reasons discussed above, the collector did not effectively accept before his option expired. (C) is wrong for two reasons: (i) a revocation is not effective until received; and (ii) because the contract is an option, the offeror's power to terminate the offer through revocation is limited. Even if the revocation had arrived earlier, the neighbor lacked the power to revoke. (B) is irrelevant. Returning the consideration, in and of itself, would not give the offeror the power to revoke in an option situation.
173
A farmer who supplies several local bakeries with grains wanted to sell his rye before the growing season was over. The farmer sent the following e-mail to a local baker: "Will sell my unprocessed rye, 20 bushels maximum, best price $100 per bushel, firm for 48 hours. /s/ Farmer." Unsure how the baker would respond, and anxious to find a buyer for the rye, the farmer made the same offer to the baker's chief competitor by e-mail later that same day. The baker was delighted to receive the offer, but needed a day or so to figure out how much rye she needed. When she accepted the farmer's offer the next day, e-mailing to him an order for 20 bushels, she was aware of the farmer's offer to her competitor, and that her competitor had also e-mailed an order to the farmer for 20 bushels. Unbeknownst to the baker, the farmer has only 30 bushels of rye left in his fields. Assuming the farmer is a merchant with respect to rye, which of the following states the probable legal consequences of the correspondence between the parties? A The farmer has a contract with the baker and her competitor for 15 bushels each. B The farmer has a contract with the baker's competitor for 20 bushels and a contract with the baker for the remaining 10 bushels. C The farmer has a contract with the baker for 20 bushels and a contract with her competitor for 20 bushels. D The farmer has a contract with neither the baker nor her competitor.
C The farmer has a contract with the baker for 20 bushels and a contract with her competitor for 20 bushels. The farmer has two contracts, one with the baker and one with the competitor, for 20 bushels each. Because his e-mail provided a firm price for 48 hours and the farmer is a merchant, the offer was an irrevocable firm merchant's offer during the 48 hours. Under the UCC, which governs here because goods are involved, a written offer signed by a merchant giving assurances that it will stay open will be irrevocable for the time stated. The farmer qualifies as a merchant of rye (one who deals in goods of that kind sold) and his offer was written and signed and contained words of firmness ("firm for 48 hours"), so it was irrevocable for 48 hours. The baker accepted the offer within the stated time. Thus, a contract was formed between the baker and the farmer. A contract was also formed between the baker's competitor and the farmer because the competitor accepted the farmer's offer. Therefore, the farmer is obligated to both the baker and her competitor for 20 bushels. If the farmer does not have the appropriate quantity in his field, he will have to procure it from somewhere else or be in breach. (A) and (B) are incorrect because they do not reflect the terms of the contracts agreed to by the parties. If a seller is unable to fully perform because of an unforeseen circumstance (i.e., impracticability), he must allocate deliveries between customers. First, this is not an unforeseen circumstance. Second, allocating between customers does not change those contracts. It is still a breach, and the customers may cancel the contract. (D) is incorrect because, as explained above, the farmer has a contract with both the baker and her competitor.
174
On July 1, a cattle breeder, who was planning to retire soon, sent a note to his neighbor offering to sell his prize bull for $15,000. On July 10, the neighbor, who was also a cattle breeder, wrote the following note to the retiring breeder: "I have decided to take the bull. I will give you a cashier's check on delivery on Saturday, July 28." The retiring breeder did not respond. The retiring breeder did not want to deliver the bull on July 28 and did not think that the delivery day was agreed to. Instead, he delivered the bull on Monday, July 30. The neighbor refused the delivery and stated that he had found another bull he likes better. The retiring breeder sues the neighbor for breach of contract. Is the retiring breeder likely to prevail? A Yes, because his breach, if any, was minor. B Yes, because the parties had not agreed on July 28 as the delivery date. C No, because there was no contract. D No, because he did not deliver the bull on July 28.
D No, because he did not deliver the bull on July 28. The retiring breeder will not prevail because he did not deliver the bull on July 28. This is a contract for a sale of goods and thus is governed by the UCC. Under the UCC, an acceptance with additional terms does not constitute a rejection and counteroffer, but rather is an effective acceptance unless made expressly conditional on the assent to the additional terms. Here, the neighbor accepted the offer and added the additional term of a delivery date. Thus, there was a contract. Whether additional terms become part of the agreement depends on whether both parties are merchants. If both parties to the contract are merchants, additional terms in the acceptance will be included in the contract unless they materially alter the terms of the offer, the offer expressly limited the acceptance to its terms, or they are objected to within a reasonable time. Here, both parties are breeders in the cattle business and, thus, are merchants. The change in the delivery date does not materially change the offer (i.e., it does not change a party's risk or remedies), the offer did not limit the acceptance to its terms, and the retiring breeder did not object. Therefore, the July 28 delivery date became part of the contract. By delivering the bull on July 30th, the retiring breeder breached the contract. (A) is incorrect because this is a contract for the sale of goods, which requires perfect tender. Whether the breach was material or minor has no effect. (B) is incorrect because under the UCC, the July 28 term became part of the contract when the breeder failed to object to it. (C) is incorrect because, under the UCC, an acceptance is effective even if it includes additional terms. Thus, the neighbor's letter on July 10 was sufficient to create a contract.
175
On July 1, a cattle rancher offered to sell his ranch to a dairy farmer for $150,000. The dairy farmer paid the cattle rancher $1,000 to hold the offer open for a period of 30 days. On July 10, the dairy farmer wrote to the cattle rancher, telling him that he could not pay more than $100,000 for the ranch, and that if he would not agree to accept that amount, he would not go through with the deal. The dairy farmer received no reply from the cattle rancher. On July 29, the dairy farmer mailed a letter to the cattle rancher telling him that he accepted his offer to sell the ranch and enclosed a check for $150,000. The cattle rancher received this letter on August 1. Has a contract been formed between the parties for the sale of the ranch? A No, because the dairy farmer's letter of July 10 terminated the cattle rancher's offer. B No, because the cattle rancher did not accept the dairy farmer's counteroffer of $100,000. C No, because the cattle rancher did not receive the dairy farmer's acceptance within 30 days. D Yes, because the dairy farmer dispatched his acceptance of the cattle rancher's offer prior to the expiration of 30 days.
C No, because the cattle rancher did not receive the dairy farmer's acceptance within 30 days. No contract was formed because the cattle rancher did not receive the dairy farmer's acceptance within 30 days. Under the mailbox rule, acceptance by mail or similar means creates a contract at the moment of dispatch. However, the mailbox rule does not apply to option contracts. An acceptance under an option contract is effective only upon receipt. [Restatement (Second) of Contracts §63] Here, an option contract existed because the dairy farmer paid the cattle rancher $1,000 to hold the offer open for 30 days. The dairy farmer mailed his acceptance within 30 days but it was not received by the cattle rancher within the 30-day period, so the acceptance was not effective. The option specified the period of time during which the offer would remain open, after which the offer terminated. Thus, (C) is correct, and (D) is wrong. (A) and (B) are wrong because an option contract is irrevocable for the time period stated. Thus, not even the dairy farmer himself could revoke the offer within the 30-day period.
176
How can one avoid the preexisting legal duty rule? A By full performance of the duty B By modifying the original consideration slightly C By making a brand-new identical promise D By beginning performance
B By modifying the original consideration slightly Courts are anxious to avoid the preexisting duty rule, which states that the promise to perform, or the performance of, an existing legal duty is not consideration. Thus modifying the original consideration, even slightly, is generally enough to avoid the rule. Making a brand-new identical promise is not sufficient because there is no consideration for the new promise. There must be new consideration or the consideration that is different in some way, such as by accelerating performance, to avoid the preexisting duty rule. Beginning performance does not avoid the preexisting legal duty rule. Even full performance of a preexisting legal duty is not sufficient consideration. There must be some new or different obligation.
177
A professional baseball player visited a sick boy in the hospital. The player told the boy that in consideration of the boy's courage, he would hit a home run for him in his next game. As the player was leaving the hospital, the boy's father stopped the player and told him how important the home run could be in improving his son's spirits and health. The father told the player he would pay him $5,000 if he did hit a home run in his next game. The player agreed and took extra batting practice before his next game to improve his chances. In his next game, the player hit two home runs. The player's contract with his ball club does not forbid him from accepting money from fans for good performance. The player has now asked the father for the $5,000. If the father refuses to pay and the baseball player brings an action against him for damages, which of the following is correct under the prevailing modern rule in contract law? A The player can recover the $5,000 because the preexisting duty rule does not apply where the duty is owed to a third person. B The player can recover the $5,000 if he can prove that the value of the home run to the boy is at least $5,000. C The player cannot recover from the father because the player had a preexisting duty to use his best efforts to hit home runs. D The player cannot recover from the father because, even under the modern trend, moral consideration is not valid.
A The player can recover the $5,000 because the preexisting duty rule does not apply where the duty is owed to a third person. The player can recover because, under the prevailing modern rule, the preexisting duty rule does not apply if the duty is owed to a third person. Generally, contracts must be supported by consideration. A promise to perform is valid consideration, but if a person already owes a duty to perform, traditionally that performance cannot be used as consideration for another promise. Thus, under the traditional rule, the player could not enforce the father's promise to pay the player $5,000 if he hit a home run because the player gave no valid consideration in exchange for the father's promise, since the player owed a preexisting duty to his ball club to exert his best efforts to hit home runs. However, under the modern view as formulated in Restatement (Second) of Contracts, section 73, and followed by a majority of courts, a duty is a preexisting duty only if it is owed to the promisee. Thus, a promise to perform a duty is valid consideration as long as the duty of performance is not already owed to the promisee. In other words, if the duty is owed to a third party, a promise to perform given to another is valid consideration as long as it was bargained for. (B) is incorrect because there is no exception to the preexisting duty rule-modern or otherwise-that allows the promisor to recover merely because his performance benefited a third party. The player can recover under the modern approach because his promise to the father was bargained for. Conversely, the player does not have to prove that the value of his home run to the boy was at least $5,000, because courts generally will not inquire into the adequacy of consideration. (C) would be correct under the traditional rule, but, under the modern trend, the promise here is valid consideration because the duty to hit home runs was owed to a third party (the ball club) rather than to the promisee (the father). (D) is incorrect because while it is true that moral consideration is not good consideration, the father did not rely on moral consideration, but rather exchanged a promise to pay $5,000 for the player's performance.
178
The owner of a one-acre parcel of land with a small house on it rented the property to a professor of a nearby college at a monthly rental of $500. Several years later, after the professor got tenure, the parties orally agreed that the professor would purchase the property from the owner for the sum of $60,000, payable at the rate of $500 a month for 10 years. They agreed that the owner would give the professor a deed to the property after five years had passed and $30,000 had been paid toward the purchase price, and that the professor would execute a note secured by a mortgage for the balance. The professor continued in possession of the property and made all monthly payments in a timely fashion. When he had paid $30,000, he tendered a proper note and mortgage to the property owner and demanded that she deliver the deed as agreed. The owner refused because valuable minerals had been discovered on adjacent parcels in recent months, causing the value of this parcel of land to increase to 10 times its former value. The professor brought suit against the property owner for specific performance. If the court rules in favor of the property owner, what is the likely reason? A The transaction had not proceeded far enough to amount to an estoppel against enforcement of the Statute of Frauds. B The purchase price, given the present value of the land, made the contract unconscionable, providing the property owner with a valid defense to enforcement. C Oral agreements are generally revocable unless expressly made irrevocable. D The professor's payments are as consistent with there being a landlord-tenant relationship between them as with there being an oral contract.
D The professor's payments are as consistent with there being a landlord-tenant relationship between them as with there being an oral contract. If the property owner wins, it will be because the payments by the professor may be based on a valid landlord-tenant relationship. A promise creating an interest in land must be in writing to be enforceable. This includes not only agreements for the sale of real property or an interest therein, but also leases for more than one year. However, under the part performance doctrine, conduct that unequivocally indicates that the parties have contracted for the sale of land will take the contract out of the Statute of Frauds. Here, the parties had originally created a landlord-tenant relationship, and the lease would be enforceable even without a writing as a month-to-month tenancy. The continuation of the monthly payments can as readily be explained by a continuation of the lease relationship as by an oral agreement for an installment land sale contract. Thus, because the conduct does not unequivocally indicate a contract for the sale of land, the Statute of Frauds requirements will not be excepted. (A) is wrong because while part performance may create an estoppel, the professor will have a hard time proving it because the parties' conduct is consistent with a lease relationship as well. (B) is wrong because unconscionability is measured at the time the contract is formed, and there is nothing in the facts to indicate that the price was not fair at that time. Moreover, the property owner was not in a weaker bargaining position vis-??-vis the professor that would have forced her to accept an unfair price for the property; the parties were of roughly equal bargaining position and, as discussed above, the price was not unfair when the deal was struck. (C) is wrong because it states an incorrect position of the law; other than Statute of Frauds requirements, oral agreements are no more revocable than written agreements.
179
A massive earthquake struck Mexico, causing widespread death and destruction. The President of Mexico made an urgent plea to the President of the United States asking for assistance. Congress was not in session and the President, without calling Congress into special session, ordered extensive military aid, in the form of personnel and equipment, to assist Mexico. Which of the following best describes the constitutionality of the President's orders? A The action was a valid exercise of the President's position as Commander in Chief of the Armed Forces of the United States. B The action was valid under the plenary powers of the President, inasmuch as the Army units were used for humanitarian, rather than warlike, purposes. C The action was unconstitutional, because the President may not perform acts when Congress is not in session that he would have to ask Congress to approve if it were in session. D The action was unconstitutional, because the President does not have the power to unilaterally authorize this type of foreign aid.
A The action was a valid exercise of the President's position as Commander in Chief of the Armed Forces of the United States. The President as Commander in Chief of the Armed Forces has broad discretion to use American troops in foreign countries. In fact, never in all of American history has a presidential use of troops in a foreign nation been declared unconstitutional. Thus, the President's power as Commander in Chief would justify this action. (B) is incorrect because the Constitution does not limit the President's use of troops to humanitarian, as opposed to war, purposes. The President may send troops into actual hostilities without a congressional declaration of war. (B) draws a distinction that has no support in the text of the Constitution or in Supreme Court precedents. (C) is incorrect because the President may use troops in foreign countries whether or not Congress is in session. Moreover, (C) is based on a false premise-that the President would need prior congressional approval to send the troops to Mexico. (D) is incorrect because the President does have the unilateral authority to send troops to a foreign country. Calling the troops "foreign aid" does not change the legality of the President's power as Commander in Chief to use troops in foreign nations.
180
A lunarian society believed that intelligent life existed on the moon, but that the government was not interested in searching for it. A wealthy political contributor who was a member of this society successfully lobbied Congress to pass a $100 million bill to investigate this hypothesis, even though virtually all reputable scientists rejected it. The President vetoed the bill, calling it a waste of money, but Congress overrode the veto, and the appropriation was authorized. If the constitutional validity of the expenditure is challenged in federal court, is the court likely to find it valid? A Yes, because the spending power of Congress is limited only by the political process. B Yes, because Congress could believe that the expenditure is a reasonable measure to advance the general welfare. C No, because the expenditure is inconsistent with the exclusive authority of the President over foreign affairs. D No, because the spending does not directly affect interstate or foreign commerce.
B Yes, because Congress could believe that the expenditure is a reasonable measure to advance the general welfare. The court is likely to find the expenditure constitutional. Congress can spend federal monies for the general welfare [art. I, §8] provided the spending is reasonably related to a legitimate federal interest. [South Dakota v. Dole (1987)] Here, Congress could believe that it is in the national interest to determine conclusively whether life exists on the moon. (A) is incorrect because Congress cannot spend for unconstitutional purposes, e.g., to promote racial discrimination. (C) is too broad a statement. While the President has broad authority over foreign affairs, the authority is not "exclusive," as Congress also plays a major role in this area. (D) is incorrect because the Spending Clause is not limited by its relationship to commerce.
181
The President issued an executive order prohibiting exportation to certain countries of specific computer software that, although not usable directly to develop nuclear weapons, would facilitate nuclear weapons technology. Congress had previously passed a law authorizing the issuance of such orders. Prior to the issuance of the executive order, a computer software company contracted with one of those countries for software that is now banned for sale and distribution. What effect does the executive order have on this contract? A The executive order unconstitutionally impairs the obligation of the company's preexisting contract, which was lawful when made. B The executive order unconstitutionally denies the company a valuable property interest without due process, because it is not limited to computer software used directly to produce nuclear weapons and, therefore, is not necessary to vindicate a compelling national need. C The executive order is constitutional because Congress has plenary powers to regulate commerce with foreign nations and has used that power to authorize such orders. D The executive order is constitutional, because the inherent power of the President to conduct foreign affairs is plenary.
C The executive order is constitutional because Congress has plenary powers to regulate commerce with foreign nations and has used that power to authorize such orders. The executive order is constitutional because it was authorized by Congress. For all practical purposes, the power to regulate foreign commerce lies exclusively with Congress, and Congress may delegate this power to the President. By authorizing the issuance of the Presidential executive order, Congress has constitutionally exercised this power. (D) is incorrect. The President's power is limited by the Constitution, which grants Congress the power to regulate foreign commerce. (A) is incorrect because the impairment of the Contract Clause applies to state governments only, not to the federal. (B) is incorrect because the company's contract right would not qualify as a property interest under the Fifth Amendment.
182
Due to violence erupting against picketers advocating automatic deportation of foreign persons accused of a crime, a state enacted a law prohibiting all picketing "carried out for the purpose of deterring others from exercising their constitutional rights." The strongest constitutional defense that can be asserted by those charged with violating this statute is which of the following? A The Fifth Amendment right to due process of law, because the statute is so vague that reasonable persons cannot ascertain its scope. B Equal protection of the laws, because the statute does not prohibit picketing for purposes other than those specified. C First and Fourteenth Amendment rights of free expression and assembly, because the statute excessively restricts the marketplace of ideas. D Article IV privileges and immunities of state citizenship, because picketing is a fundamental right.
C First and Fourteenth Amendment rights of free expression and assembly, because the statute excessively restricts the marketplace of ideas. The strongest defense is based on free expression and assembly grounds. It is presumptively unconstitutional for the government to place burdens on speech because of its content. Because the statute regulates content of the speech rather than conduct, the state would need to establish a compelling interest behind the legislation for the law to be valid. This puts the burden on the state to prove the need for the legislation. (A) is incorrect because the Due Process Clause of the Fifth Amendment applies to the federal government, not to the states. (B) is incorrect because any discrimination that may arise from the statute is merely an effect of facially neutral legislation and not subject to the Equal Protection Clause. (D) is incorrect because Article IV prohibits discrimination by a state in favor of its own citizens, which is not the case here.
183
In an effort to keep all protest demonstrations peaceful, legislation was enacted that imposed fines and jail time on leaders of unruly demonstrations. The law made it a crime to "aid, abet, incite, urge, or encourage behavior that amounts to a breach of the peace." A protester led a large group of picketers to a plant allegedly involved in chemical warfare research. He shouted to his followers, "Let's show these murderers what we think of them! Let's stop the poison gas machine now!" A student at the local university, carried away with emotion, picked up a brick and heaved it through one of the plant's windows. The student was arrested for, charged with, and convicted of disturbing the peace, and is now challenging his conviction on constitutional grounds. The protester leading the event also was arrested, charged with, and convicted of violating the statute. If the protester challenges his conviction in federal court, what is his best argument? A The protester is not vicariously liable for the student's activities. B The student has raised a constitutional challenge to his own conviction for disturbing the peace. C The statute is unconstitutionally vague. D The First and Fourteenth Amendments preclude a state from restricting political protest speech.
C The statute is unconstitutionally vague. The protester's best argument is that the statute is unconstitutionally vague. If a criminal law or regulation fails to give persons reasonable notice of what is prohibited, it may violate the Due Process Clause. This principle is applied somewhat strictly when First Amendment activity is involved in order to avoid the chilling effect a vague law might have on speech. Here, it is difficult to know what behavior is proscribed by a statute making it a crime to "aid, abet, incite, urge, or encourage behavior that amounts to a breach of the peace." This offers the protester his best argument because the other choices are clearly wrong. (A) is incorrect because vicarious liability is a tort law concept that makes one person liable for another person's tort. Aside from this concept being inapplicable in this setting, the protester is being charged with a separate crime rather than the breach of peace crime that the student was charged with. (B) is incorrect because it is irrelevant. The protester has a valid constitutional challenge to the statute he was convicted of violating, regardless of the student's challenge to his breach of peace conviction. (D) is incorrect because it is too broad. The state may limit political protest speech as long as the restriction is a reasonable time, place, and manner limitation that is not content based.
184
At common law, a conveyance of property from O "to O and A as joint tenants with right of survivorship" creates a __________. Responses A Tenancy for years B Tenancy by the entirety C Tenancy in common D Joint tenancy
C Tenancy in common At common law, a conveyance of property from O "to O and A as joint tenants with right of survivorship" creates a tenancy in common. There are three forms of concurrent ownership in land: joint tenancy with right of survivorship, tenancy in common, and tenancy by the entirety. In a joint tenancy, each co-tenant owns an undivided share of the property, and the surviving co-tenant has the right to the whole estate (right of survivorship). At common law, four unities are required to create a joint tenancy: (i) time (interests vested at the same time), (ii) title (interests acquired by the same instrument), (iii) interest (interests of the same type and duration), and (iv) possession (interests give identical rights to enjoyment). If these four unities are not present, a joint tenancy cannot be created at common law. Instead, a tenancy in common results. A tenancy in common is a concurrent estate with no right of survivorship. A tenancy by the entirety is a marital estate akin to a joint tenancy in that four unities (plus a fifth-marriage) are required for its creation, and the surviving spouse has the right of survivorship. A conveyance from O "to O and A" does not satisfy the unities of time and title because O acquired his interest first by another instrument. Thus, the conveyance creates a tenancy in common rather than a joint tenancy. The conveyance does not create a tenancy by the entirety at common law because, as explained above, it does not satisfy the unities of time and title, and the facts do not indicate that O and A are husband and wife. The conveyance does not create a tenancy for years at common law. A tenancy for years is a leasehold estate in land wherein the tenant has a present possessory interest in the leased premises and the landlord has a reversion. Here, O and A are not in a landlord-tenant relationship.
185
A landowner conveyed her parcel of land to "my brother and my sister jointly, with right of survivorship." Shortly thereafter, the brother was in an automobile accident. The driver of the other vehicle sued the brother on a theory of negligence, and obtained a judgment in the amount of $250,000. Because the brother did not have insurance or enough cash to satisfy the judgment, the driver levied on the brother's interest in the land. What interest will the driver most likely take? A None, because the brother's interest in the land cannot be partitioned. B An undivided one-half interest, regardless of whether the brother and the sister's title to the land is construed as a joint tenancy or a tenancy in common. C An undivided one-half interest, assuming the brother and the sister's interest is construed as a tenancy in common and not a joint tenancy. D A contingent right of survivorship that will vest if the brother survives the sister.
B An undivided one-half interest, regardless of whether the brother and the sister's title to the land is construed as a joint tenancy or a tenancy in common. The driver will get an undivided one-half interest in the land regardless of the status of the brother and the sister's title. A joint tenancy is a concurrent estate with a right of survivorship, while a tenancy in common does not have a right of survivorship. At common law, the conveyance here would qualify as a joint tenancy because the unities of time, title, interest, and possession are present in the conveyance. Although under modern law a joint tenancy must be created with specific language or else it will be presumed to be a tenancy in common, the conveyance here still would probably qualify as a joint tenancy, even though it did not use the words "joint tenancy," because it contained the "right of survivorship" language. However, regardless of whether the estate is characterized as a joint tenancy or a tenancy in common, one tenant's interest may be transferred without the consent of the other tenant, and a creditor may levy on the interest. In most jurisdictions, a lien against one joint tenant's interest does not sever the joint tenancy until the lien holder proceeds to enforce it by foreclosure. At that point, the purchaser at the foreclosure sale will hold the property as a tenant in common with the other tenant, but will still have an undivided one-half interest in the property unless and until he brings an action to partition the estate. (A) is incorrect because both joint tenancies and tenancies in common may be subject to partition. (In contrast, tenancies by the entirety cannot be terminated by involuntary partition.) (C) is incorrect because, as discussed above, a joint tenant may validly convey or encumber his interest in the property. (D) is incorrect because the driver does not have a contingent interest; she has a present lien on the brother's interest that can be enforced immediately by foreclosure, which would sever the joint tenancy.
186
A tenant rented an apartment in a large multi-unit building. One day vandals broke into several of the building's apartments, including the tenant's, and smeared excrement into the carpets and on the walls, and broke out all of the windows. The jurisdiction provides by statute that if a tenant notifies her landlord in writing of a repair that is needed to keep the premises in a habitable condition and the landlord does not repair it within 15 days, the tenant may, at her option, either repair it herself and withhold the expenses from rent, or consider herself constructively evicted and terminate her tenancy. The tenant wrote a letter to the landlord informing him that her walls, carpets, and windows had been damaged and needed repair, and after eight days she received a letter in reply stating that such damages to her apartment were her responsibility to repair. After waiting another week, the tenant paid to have her carpets and walls cleaned and to have her windows replaced. She then withheld the entire next month's rent of $400, because the cleaning and repair bills had totaled $750. After sending her the required statutory notices, the landlord commenced unlawful detainer litigation, seeking to have the tenant evicted for nonpayment of rent. How should the court rule? A For the landlord, because the damage was the result of the criminal acts of a third party. B For the landlord, because the damage was to a private apartment and not to the common areas of the apartment complex. C For the tenant, if she can show that the landlord was negligent in connection with the vandalism. D For the tenant, because she satisfied the requirements of the statute.
D For the tenant, because she satisfied the requirements of the statute. The tenant will win because she had a right under the statute to withhold the rent. The general rule at common law was that the landlord was not liable to the tenant for damages caused by the landlord's failure to maintain the premises during the period of the leasehold. Today, however, a majority of jurisdictions, usually by statute, provide for an implied warranty of habitability for residential tenancies. The statute in this question allows the tenant to make the repairs and withhold the cost of the repairs 15 days after notifying the landlord in writing. The statute is applicable because the damage done by the vandals makes the apartment unfit for habitation under whatever standard the court would apply. The tenant has complied with the terms of the statute; she therefore cannot be evicted for nonpayment of rent. (A) is incorrect. The fact that the damage was caused by a third party would be relevant only if the tenant were relying on the judicially developed remedy of constructive eviction, which requires that the damage making the premises uninhabitable have been caused by the landlord. The warranty of habitability is not limited in this way. (B) is incorrect because the statute has extended the common law duty of the landlord, which applied only to the common areas of a multiunit building. (C) is incorrect because the tenant does not need to show that the landlord was negligent.
187
Which of the following transfers creates an assignment of the lease from T to T2? A Four years into a six-year tenancy for years, T orally transfers his entire interest to T2 for two years. B Two years into a four-year tenancy for years, T "assigns my entire interest to T2 for one year". C One year into a five-year tenancy for years, T transfers his interest "to T2 for four years; however, if T2 breaches the original lease terms, T may reenter and retake the premises". D Six months into a tenancy at will, T transfers "my entire interest to T2".
C One year into a five-year tenancy for years, T transfers his interest "to T2 for four years; however, if T2 breaches the original lease terms, T may reenter and retake the premises" If one year into a five-year tenancy for years, T transfers his interest "to T2 for four years; however, if T2 breaches the original lease terms, T may reenter and retake the premises," the effect of the transfer is an assignment of the lease from T to T2. A complete transfer of the entire remaining lease term constitutes an assignment of the lease. If the tenant retains any part of the remaining lease term, other than a right of reentry for breach of the original lease terms, the transfer is a sublease. Here, T transferred the remaining four years of the lease to T2. By the slight majority view, T's reservation of a right of reentry does not result in a sublease, but rather is still an assignment. If six months into a tenancy at will, T transfers "my entire interest to T2," the attempted assignment is void and terminates the tenancy at will by operation of law. A tenancy at will is a leasehold estate that is terminable at the will of either the landlord or the tenant. Such a tenancy terminates by operation of law if: 1. Either party dies; 2. The tenant commits waste; 3. The tenant attempts to assign his tenancy; 4. The landlord transfers her interest in the property; or 5. The landlord executes a term lease to a third person. If two years into a four-year tenancy for years, T "assigns my entire interest to T2 for one year," the effect of the transfer is to create a sublease between T and T2. The label given to the transfer by the parties does not determine whether the transfer is an assignment or a sublease. The nature of the transaction is determined by what interest, if any, the tenant retains. Here, although T "assigned" his interest to T2, he transferred only one of the remaining two years of the lease. Thus, the transfer is a sublease rather than an assignment. If four years into a six-year tenancy for years, T orally transfers his entire interest to T2 for two years, the attempted assignment is ineffective under the Statute of Frauds. Most states require that a lease creating a tenancy for more than one year, including an assignment of an interest in a lease for more than one year, be in writing to satisfy the Statute of Frauds.
188
A landlord leased a building to a baker for 10 years, commencing January 1, at a monthly rental of $1,700. The lease stated in part, "The tenant may not sublet or assign this lease without first receiving written permission from the landlord to do so. Any attempt to sublet or assign the lease without first receiving written permission shall constitute a breach entitling the landlord to terminate this lease." Five years later, an investor approached the baker and offered to purchase the bakery if the baker would agree to sublet the premises to him. The baker agreed and executed a sublease on July 1 of that year. The investor took possession the same day. On July 3, the baker approached the landlord and asked for written permission to sublet the premises to the investor. The landlord said he had no real objection to the sublease and would execute the document requested by the baker, but only if the investor would sign a five-year extension of the existing lease. The investor refused to extend the lease, but remained in possession of the building. At no time did the landlord accept rent from the investor. After notice was given to all parties and the applicable grace period in the lease had elapsed, the landlord brought an appropriate action against the baker and the investor to evict them from the premises and to declare the lease terminated because it had been breached. How should the court rule in this action? A Against the landlord, because his withholding consent is an invalid restraint on alienation. B Against the landlord, because his conditional consent operated as a waiver of the term of the lease requiring the landlord to give written permission for subletting. C For the landlord, because the baker has breached the lease. D For the landlord, because his oral consent to sublet is not enforceable under the Statute of Frauds.
C For the landlord, because the baker has breached the lease. The landlord should prevail because the baker has breached the lease. Generally, if a tenant transfers (assigns or sublets) in violation of a prohibition in the lease against transfers, the transfer is not void. However, the landlord usually may terminate the lease under either the lease terms or a statute. Here, because the baker has breached the provision of the lease prohibiting assignment or sublease, and the lease contains a forfeiture clause, the landlord was within his rights to terminate the lease. (A) is incorrect because clauses restricting assignment or sublease are not considered to be restraints on alienation. (B) is incorrect because a conditional consent is not a waiver where the condition is not agreed to. (D) is incorrect because an oral consent, if made, is sufficient to waive the provision. Here, the landlord's consent was conditioned on the investor signing an extension of the lease, which he did not do.
189
A man had rented a woman's home from her for seven years. When the time came to sign a new lease, the woman decided that because the man had always been a quiet tenant, she would continue to charge him only $350 per month rent instead of the $500 to $550 she could probably get otherwise. The new lease was for a period of five years, and by its terms, the man was specifically prohibited from assigning the lease without the woman's specific written consent. About a year later, the man got married and moved into his new wife's home. Instead of giving up his lease, the man sublet the property to a friend for $500 a month. The man did not get the woman's permission to sublease the property. If the woman brings an action to either eject the friend from the premises or to recover damages from the man for subletting the premises without her consent, what is the most likely result? A The woman will be able to recover damages and to eject the new tenant. B The woman will be able to eject the new tenant only, because she has suffered no money damages. C The woman will not be able to eject the new tenant because, although the man did not have the right to sublet, he had the power, but she will be entitled to recover the full rent paid by the new tenant because it would be unfair to let the man profit from his wrongful act. D The woman will have no cause of action for either ejectment or damages.
D The woman will have no cause of action for either ejectment or damages. The woman will most likely have no cause of action for either ejectment or damages. There are two ways for a tenant to transfer the right to possession under a lease: assignment (transferring the entire period of time remaining under the lease) and sublease (transferring only a portion of the time remaining under the lease). Restraints on alienation are traditionally strictly construed. Thus, a covenant prohibiting assignment does not prohibit subleasing and vice versa. Hence, this prohibition against assignment would not be read to include a prohibition against subleasing. Therefore, the woman would have no cause of action against the man, and (A) and (B) are incorrect. (C) is incorrect. If a tenant transfers (assigns or sublets) in violation of a prohibition in the lease against transfers, the transfer is not void, but the landlord usually may terminate the lease under either the lease terms or a statute. Here, however, there is no cause of action because subleasing was not prohibited.
190
If an easement is said to be surcharged, this means: A The easement holder paid valuable consideration to the servient landowner B The easement was terminated by operation of law C The easement's legal scope was exceeded D The easement's legal scope was interpreted to accommodate the holder's future reasonable needs
C The easement's legal scope was exceeded If an easement is said to be surcharged, this means the easement's legal scope was exceeded. The holder of an easement has the right to use another's land (i.e., the servient tenement), but has no right to possess the land. The scope of an easement is determined by the reasonable intent of the original parties, and when the scope has been specified, these specifics will govern. However, when an easement's scope has been set out only in general language, courts will interpret it to accommodate the holder's present and future reasonable needs. In either event, if the easement holder uses the easement in a way that exceeds its legal scope, the easement is surcharged. The servient landowner may enjoin the excess use and possibly sue for damages if the land has been harmed. However, the easement does NOT terminate by operation of law, nor does such use give the servient owner a power of termination. If an easement is said to be surcharged, this does NOT mean the easement holder paid valuable consideration to the servient landowner, but rather that he exceeded its legal scope.
191
A landowner owned a large parcel of land in a rural area. He built his home on the northern half of the property, and developed a large orchard of fruit trees on the southern portion. A county road ran in front of the northern portion. To service his orchard, the landowner built a driveway directly from the county road across the northern portion of the property to the orchard. To provide electricity to his house, the landowner ran an overhead power line across the orchard property to hook up to the only available electric power pole located on the far southern side of the property. Subsequently, the landowner conveyed the northern parcel to his brother and the southern parcel to his daughter, who said that she did not mind having the power line on the property. Recently, the brother has begun parking his car on the driveway, thus blocking the daughter's access to the southern parcel. Finding no recorded document granting an easement for the power line, the daughter has decided to remove it. If the brother is successful in preventing the daughter from removing the power line, what is the likely reason? A The daughter knew that the power line ran across the land when she accepted the deed from the landowner. B The brother's alternative access to power is much less convenient and would cost 100 times as much. C The daughter told the landowner that she did not mind having the power line on the property. D The daughter is acting in retaliation against the brother for blocking the driveway, and not in any good faith belief that she has the right to remove the power line.
B The brother's alternative access to power is much less convenient and would cost 100 times as much. If the brother is successful in preventing the daughter from removing the power lines, it will be because the brother's alternative access to power is much less convenient and would cost 100 times as much as the current arrangement. This helps to prove that there was an easement implied by operation of law ("quasi-easement"). An easement may be implied if, prior to the time the tract is divided, a use exists on the "servient part" that is reasonably necessary for the enjoyment of the "dominant part," and a court determines that the parties intended the use to continue after division of the property. To give rise to an easement, a use must be apparent and continuous at the time the tract is divided. In this case, the landowner used the servient part of his property (the southern parcel) to run an overhead power line to the dominant part of his property (the northern parcel). Overhead wires are clearly visible and would be readily discoverable on reasonable inspection. The lines are, therefore, apparent. The use must also be reasonably necessary. Whether a use is reasonably necessary depends on many factors, including the cost and difficulty of the alternatives. This use was reasonably necessary to the enjoyment of the dominant parcel because electricity is important to the enjoyment of the property, and the cost (100 times as much) and difficulty of the alternatives are excessive. Thus, the fact that the use of the southern parcel is reasonably necessary would bolster the brother's case. (A) is wrong because the daughter's actual knowledge is irrelevant. The daughter need not be aware of the use; it need only be shown that the use was apparent (see above). (C) is similarly wrong. Oral statements made to the grantor after the northern parcel had been conveyed have little effect. They show the daughter's knowledge, but as discussed above, that has little relevance with respect to an implied easement. (D) is wrong because the daughter's motive for removing the power line is also irrelevant. If no easement is established, the daughter may remove the lines for whatever reason she likes. If, however, the requirements for an implied easement are satisfied, the daughter may not remove the lines regardless of how good her reasons are.
192
A landowner and her neighbor owned adjoining parcels of land. The landowner's property was situated to the west of the neighbor's property. A highway ran along the east of the neighbor's property. Twelve years ago, the landowner asked the neighbor if it would be all right for the landowner to use an eight-foot strip along the northern part of the neighbor's land to access the highway. The only other way for the landowner to get to the highway was to use a one-lane unpaved road that meandered through the woods for two miles. The neighbor agreed, and the landowner used the strip of land regularly to access the highway. The statutory period for adverse possession in this jurisdiction is 10 years. What is the landowner's interest in the neighbor's eight-foot strip of land? A An easement appurtenant. B An easement by necessity. C An easement by prescription. D Not an easement.
D Not an easement. The landowner's interest in the neighbor's eight-foot strip of land is not an easement. In effect, the landowner only has a "license" (i.e., a revocable privilege) to use the land. The answer is best reached by the process of elimination. Because an easement is an interest in land, the Statute of Frauds applies. Here, the agreement between the landowner and the neighbor was not in writing; thus, the Statute of Frauds requirements for the creation of an express easement were not met. Therefore, (A) is incorrect. (B) is incorrect because an easement by necessity is created when the owner of land sells a part of it and deprives the part sold of access to the public road. Here, the facts do not indicate that the landowner's and the neighbor's parcels were once part of a common tract, and the landowner has an alternate, albeit inconvenient, way to access the highway-the one-lane road. Thus, the landowner does not have an easement by necessity. (C) is incorrect because the landowner's use of the land was permissive. To acquire a prescriptive easement, the use must be open and notorious, adverse, and continuous and uninterrupted for the statutory period. Although the landowner used the strip for the requisite 10-year period, she does not meet the adverse requirement necessary to obtain a prescriptive easement.
193
A developer owned a 240-acre parcel of land zoned for commercial and residential use. He prepared and recorded, after obtaining approval from all appropriate agencies, a subdivision plan that included a commercial center and a number of lots for single- and multi-family residences. The list of covenants, conditions, and restrictions recorded with the plan included provisions that required every building constructed in the subdivision to be of "simulated adobe style" architecture approved in advance by an association. A year later, the developer sold many of the lots in the commercial center, including several to a real estate firm. Each deed prepared by the developer contained a reference to the design restriction in the recorded plan. The developer also sold almost all of the residential lots, the deeds of which contained the same reference to the restriction. The following year, the real estate firm sold one of its lots to a burger franchise. The deed contained no reference to the design restriction. The franchise's prefabricated restaurant, complete with a giant burger logo mounted on the roof, was constructed over the weekend. A merchant, an original purchaser of one of the commercial lots, owned the lot next to the burger franchise. She did not learn of construction of the restaurant until she came in to work on Monday, and saw the giant burger logo. The merchant brings an action seeking a mandatory injunction compelling the burger franchise to demolish the restaurant. At trial, the merchant proves that the burger franchise did not seek or obtain approval of the association for its building. Should the court issue the injunction? A No, because destruction of the restaurant would be a tremendous waste of resources. B No, because the burger franchise's deed contained no restriction on the type of building that could be constructed on the lot. C Yes, because the restrictive covenant runs with the land. D Yes, unless the burger franchise can establish to the court's satisfaction that its restaurant design has at least as much aesthetic merit as any "simulated adobe style" design.
C Yes, because the restrictive covenant runs with the land. The court should issue the injunction because the covenant runs with the land. A covenant will be enforceable as an equitable servitude-allowing a covenantee, covenantor, or successor to enforce the covenant in equity by way of injunction-when there is (i) a covenant in a writing satisfying the Statute of Frauds, that (ii) touches and concerns the land (i.e., the effect of the covenant makes the land more useful or valuable to the benefited party) and that (iii) indicates an intention that the servitude exists, and (iv) notice is given to future owners of the burdened land. Here, the covenant was in writing in the subdivision plan and presumably it satisfied the Statute of Frauds. It touches and concerns the land-benefiting all of the lots and burdening all of the lots. The intention to create the servitude is established by the writing and can also be implied from the common scheme for development. There was sufficient record notice of the covenant because the plan was recorded and was noted in all of the original deeds prepared by the developer, including the one in the burger franchise's chain of title. Thus, the covenant is enforceable and (C) is the best answer. (A) is incorrect because although an injunction is equitable in nature-so equitable principles govern-it is not a defense in equity merely to claim that granting an injunction will result in a waste of assets. (B) is incorrect because the burger franchise had record notice of the restriction. The deed from the developer to the real estate firm, which contained a reference to the restriction in the recorded plan, was in the burger franchise's direct chain of title and could have been discovered by it. (D) is incorrect because a court will not modify the covenant-it will enforce it or not enforce it, but will not substitute its judgment of what is aesthetically pleasing for the requirements of the covenant.
194
O conveys a life estate to A, with a remainder to B. If during A's lifetime, X enters into actual, exclusive possession that is open and notorious and hostile for the statutory period, will X obtain title to the land? A Yes, if X had color of title B No, because land subject to a future interest cannot be acquired by adverse possession C No, but X will acquire A's life estate D Yes, because X satisfied the elements of adverse possession
C No, but X will acquire A's life estate No, X will not obtain title to the land. If during A's lifetime, X enters into actual, exclusive possession that is open and notorious and hostile for the statutory period, X will not obtain title to the land, but X will acquire A's life estate. If a landowner does not commence an action to eject a would-be adverse possessor before the statute of limitations expires, she is barred from suing for ejectment, and title vests in the possessor. However, the statute of limitations does not run against the holder of a future interest (e.g., remainder, reversion) until her interest becomes possessory. The future interest holder has no right to possession until the prior present estate terminates, and thus no cause of action for ejectment accrues until that time. Here, X will acquire A's life estate by adverse possession (i.e., a life estate pur autre vie, measured by A's life), but not B's remainder, which remains nonpossessory while A is living. Thus, upon A's death, X's interest will terminate. This is true even though X satisfied the elements of adverse possession. To establish title by adverse possession, the occupier must show: (i) An actual entry giving exclusive possession that is (ii) Open and notorious, (iii) Adverse (hostile), and (iv) Continuous throughout the statutory period. Land subject to a future interest CAN be acquired by adverse possession; however, the statute will not begin to run as against the future interest holder until her interest becomes possessory, as explained above. This is true regardless of whether X had color of title (i.e., a document purporting to convey title).
195
Fifty-one years ago, an owner conveyed land to a taker for "so long as the land is used solely for residential purposes; otherwise, the interest in land shall revert to the owner and his heirs." The taker used the land as her personal residence for 20 years, but 31 years ago, she began operating a children's day camp on the land. The owner knew of this operation, but he took no action. Two years ago, the aged taker decided to get out of the camp business. She closed her business and once again began to use the land solely as her personal residence. Also two years ago, the owner died, survived by his son and only heir. Now the son is laying claim to the conveyed land. The jurisdiction in which the land is located has a seven-year adverse possession statute and another statute that bars enforcement of possibilities of reverter 55 years after their creation. May the son validly claim title to the land? A Yes, because less than 55 years have elapsed since the creation of the possibility of reverter. B Yes, because the adverse possession period began to run when the taker returned the property to residential status, and the taker has not held for the requisite seven years. C No, because the adverse possession period began 31 years ago, and the taker has held the property for more than the requisite seven years. D No, because the owner did not assert his possibility of reverter; thus, no cause of action arose in the owner or his heirs.
C No, because the adverse possession period began 31 years ago, and the taker has held the property for more than the requisite seven years. On the happening of the prohibited event (using the land for other than residential purposes), the taker's fee simple determinable automatically came to an end, and the owner was entitled to present possession. Not having claimed possession within the applicable seven-year period, and with the taker's possession being open, notorious, continuous, and adverse, any action by the owner or his heirs is now barred by adverse possession. Thus, (A) and (B) are incorrect. (D) is incorrect because a possibility of reverter becomes possessory automatically upon termination of the prior determinable estate. Unlike a right of entry, a grantor does not have to assert a possibility of reverter in order for a cause of action to arise.
196
A buyer entered into a contract with a seller to purchase the seller's farm. The contract of sale referred to the farm as containing 250 acres. The agreed-on price was $1 million. Before the date on which escrow was to close, the buyer learned from a surveyor he had hired that the farm actually contained 248 acres. On the date the sale was to close, the buyer instructed the escrow agent to release all but $8,000 of the purchase money because he was not getting what he bargained for. The seller refused to proceed with the sale. The buyer brings an action for specific performance and also seeks an $8,000 reduction of the agreed-upon contract price. What will be the probable outcome of the litigation? A The seller will win, because the buyer refused to tender the contract price when the seller tendered substantially what the contract called for her to perform. B The seller will win, because both parties had seen the farm before the contract was formed. C The buyer will win, because he is not receiving what he bargained for under the contract. D The buyer will win, if the court finds that the $8,000 reduction in price is a fair reflection of the title defect.
D The buyer will win, if the court finds that the $8,000 reduction in price is a fair reflection of the title defect. This answer states the traditional rule where the amount of land in a land sale contract is less than as agreed. When a buyer has a remedy of specific performance in a land sale contract, a court of equity will order a seller to convey the title if the buyer tenders the purchase price. If the seller cannot provide marketable title under the terms of the contract, but the buyer wishes to proceed with the transaction, the buyer can usually get specific performance with an abatement of the purchase price in an amount reflecting the title defect. A defect as to the quantity of land conveyed is usually corrected by a pro rata abatement of the price. (D) states the factors that a court of equity will look for when deciding whether to grant specific performance with abatement. (A) is incorrect because the parties' contract did not merely refer to the farm as a named parcel of land; it recited that it contained 250 acres. Based on this recital, a court could readily conclude that the difference of two acres is a material change in the terms of the contract and that the seller's tender of 248 acres was not substantial performance. (B) is incorrect because viewing the property did not put the buyer on notice as to the discrepancy; the buyer is not required to visually calculate the amount of acreage a parcel of land contains. (C) is not as good an answer as (D) even though it is probably a true statement. Not only must the defect as to quantity be material, so that the buyer is not receiving what he bargained for, but the abatement amount must be appropriate and not an excessive reduction of the purchase price, as choice (D) states.
197
A landowner gratuitously conveyed his interest in land to a friend by quitclaim deed. The friend promptly and properly recorded her deed. Six months later, the landowner conveyed his interest in the same land to an investor for $50,000 by warranty deed, which was promptly and properly recorded. As between the friend and the investor, who has the superior right to title to the land? A The friend, regardless of the type of recording statute. B The friend, because she recorded prior to the investor's recording. C The investor, regardless of the type of recording statute. D The investor, because it took by warranty deed rather than quitclaim deed.
A The friend, regardless of the type of recording statute. Because the friend recorded prior to the subsequent conveyance, she has the superior right to title regardless of the type of recording statute. A conveyance that is recorded can never be divested by a subsequent conveyance through operation of the recording statutes. By recording, the grantee gives constructive (or "record") notice to everyone. Hence, proper recording prevents anyone from becoming a subsequent bona fide purchaser ("BFP"). Because the landowner's conveyance to the friend was recorded at the time of the landowner's conveyance to the investor, the investor cannot prevail. The investor will clearly lose under a pure race statute because the friend recorded first. The investor will also lose under notice and race-notice statutes because the conveyance to the friend was recorded at the time of the conveyance to the investor. The investor, therefore, had record notice and cannot claim the protection that these types of statutes provide for subsequent purchasers for value who take without notice. Thus, (A) is correct and (C) is incorrect. The fact that the friend is merely a donee rather than a BFP does not mean that her recording has no effect. It is only the subsequent taker who has to be a BFP rather than a donee to utilize the recording statute. The prior grantee, regardless of her status, protects her interest by recording because it prevents anyone from becoming a subsequent BFP. (B) is incorrect because, as noted above, the friend will prevail under any type of recording act, but not necessarily because she recorded prior to the investor's recording. If the jurisdiction has a notice statute, whether the friend recorded prior to the investor's recording is irrelevant. Rather, it is the fact that the friend recorded prior to the investor's purchase that gives the friend superior title in a notice jurisdiction, because the investor would have record notice of the conveyance and thus would not qualify as a BFP. (D) is incorrect because the quitclaim/warranty deed distinction does not affect who has title to the land; that status merely affects the parties' respective causes of action and ability to recover against the landowner.
198
To satisfy a debt owed to a creditor, a son executed and delivered to the creditor a warranty deed to a large tract of undeveloped land. The creditor promptly recorded the deed. Shortly thereafter, she built a house on the property and has lived there ever since. The son never actually owned the land. It belonged to his father, but the father had promised to leave the property to the son. Later, the father died and his will devised the property to the son. Pressed for money, the son then sold the land to an investor by warranty deed, which the investor promptly recorded. Although the investor paid full value for the property, he purchased it strictly for investment and never visited the site. He therefore did not realize that the creditor was living there, and knew nothing of the son's earlier deed to the creditor. The jurisdiction in which the land is located has the following statute: "A conveyance of an estate in land (other than a lease for less than one year) shall not be valid against any subsequent purchaser for value without notice thereof unless the conveyance is recorded." Which of the following is the most likely outcome of a quiet title action brought by the creditor against the investor? A The creditor prevails, because the son had no title to convey to the investor. B The creditor prevails, because the investor was not a purchaser for value without notice of the creditor's interest. C The investor prevails, because under the doctrine of estoppel by deed, title inures to the benefit of the original grantee only as against the grantor. D The investor prevails, because under the recording acts, the deed from the son to the creditor was not in the chain of title and hence did not constitute notice to the investor.
B The creditor prevails, because the investor was not a purchaser for value without notice of the creditor's interest. The creditor will prevail in a suit to quiet title because the investor had notice of the creditor's interest in the property and, thus, is not a bona fide purchaser for value. When a grantor purports to convey property that he does not own, his subsequent acquisition of title to that property vests in the grantee under the doctrine of estoppel by deed. Most courts, however, hold that this is personal estoppel, which means that title inures to the grantee's benefit only as against the grantor, not a subsequent bona fide purchaser. If the grantor transfers his after-acquired title to an innocent purchaser for value, the bona fide purchaser gets good title. There is a split of authority as to whether the original grantee's recordation of the deed imparts sufficient notice to prevent a subsequent purchaser from being a bona fide purchaser, but the majority view is that it does not because it is not in his chain of title. Thus, it is not the fact that the creditor recorded that prevents the investor from being a bona fide purchaser. The fact that the creditor built a home and was living on the property gave the investor constructive notice of her interest. A title search is not complete without an examination of possession. If the possession is unexplained by the record, the subsequent purchaser is charged with knowledge of whatever an inspection of the property would have disclosed and anything that would have been disclosed by inquiring of the possessor. Therefore, the investor is charged with knowledge of the creditor's possession and with what the creditor would have told him about her possession; i.e., that the property was conveyed to her by the son prior to his conveyance to the investor. Consequently, the investor does not qualify as a bona fide purchaser, and (C) is an incorrect choice. (A) is incorrect because, although the son is estopped to deny that he acquired title for the benefit of the creditor, he could have conveyed valid title to a subsequent purchaser for value who had no notice of the creditor's interest. Therefore, it is not exactly correct to say that the son had no title to convey. (D) is incorrect because the investor will not prevail. It is true that under the recording acts the creditor's deed was not in the chain of title, but the investor still does not qualify as a bona fide purchaser. The investor is on inquiry notice arising from the creditor's possession of the property.
199
On February 10, an owner took out a $10,000 mortgage on her land with a bank. On February 15, the owner conveyed the land for $50,000 to a buyer who was not aware of the mortgage. On February 17, the bank recorded its mortgage interest in the land. On February 21, the buyer recorded his deed to the land. Does the buyer hold the land subject to the bank's mortgage? A Yes, in a race-notice jurisdiction. B Yes, regardless of the type of recording statute. C No, in a race-notice jurisdiction. D No, because the buyer was a bona fide purchaser for value who bought the land before the bank recorded its mortgage.
A Yes, in a race-notice jurisdiction. The buyer takes subject to the bank's mortgage in a race-notice jurisdiction because it was recorded first. All recording acts apply to mortgages as well as deeds. Thus, a subsequent purchaser of the property will take subject to a prior mortgage unless the recording act changes the result. A race-notice recording act would change this result only where a subsequent purchaser did not have notice of the mortgage at the time of purchase and recorded his deed before the mortgage was recorded. Here, the buyer did not have notice of the mortgage but he recorded after the bank; thus, he takes subject to the bank's interest. (C) is wrong because the buyer did not win the race to record, which is one of the two requirements for a subsequent purchaser to prevail in a race-notice jurisdiction. (B) is wrong because the buyer would not take the land subject to the bank's mortgage in a pure notice jurisdiction. Under a notice recording act, a subsequent bona fide purchaser with no actual or constructive notice prevails over a prior grantee or mortgagee who has not recorded at the time of the conveyance to the subsequent purchaser. (D) is not the best answer because it would only be true in a notice jurisdiction. The buyer would take subject to the mortgage in a pure race or race-notice jurisdiction because the mortgage was recorded before the buyer's deed (even though the buyer did not have notice of the mortgage when he bought the land).
200
An advertising agency specializing in aerial banners and skywriting signed a contract with a film production company that was premiering a new blockbuster film. The contract provided that the agency would advertise the film by flying over the city towing a giant streamer belonging to the film company heralding the film's catch phrase and title in large letters. This contract specified that the flight was to be conducted on the first Saturday in June at noon (the day of the local premier), and the film company was to pay the advertising agency $500 for the flight. On the designated Saturday, the advertising agency was unable to fly because of a defective fuel pump. The defective condition was entirely unforeseeable and did not occur through any negligence or fault of the agency. The film company did not pay the agency, and each of the parties has sued the other for damages. Which of the following best states the rights and liabilities of the parties? A The film company is entitled to recover damages from the advertising agency on account of the agency's failure to fly. B The advertising agency is entitled to recover from the film company the $500 contract price, as the incapacity of the airplane was not the agency's fault. C Neither party is entitled to recover against the other, because the advertising agency's duty to fly was discharged by impossibility, and the film company's duty to pay was contingent on the agency's flight. D Neither party is entitled to recover against the other, because the film company's offer to pay $500 for the flight was in effect an offer for an act, and because the act was not performed, there was no valid acceptance.
A The film company is entitled to recover damages from the advertising agency on account of the agency's failure to fly. The film company will be able to recover damages from the advertising agency because the agency's failure to fly constituted a breach of contract. The parties entered into a bilateral contract-the agency promised to fly with the streamer and the film company promised to pay for the flight. The agency breached the contract by failing to fly on the designated Saturday. Its duty to fly was not discharged by impossibility. A contractual duty to perform may be discharged by objective impossibility (i.e., no one could have performed), but subjective impossibility (defendant could not perform) is insufficient. Here, the defect in the plane constituted only subjective impossibility (if it amounted to impossibility at all) because the agency could have obtained another plane to pull the streamer. If the agency had been unable to fly the plane because of weather (e.g., a severe ice storm), its performance would have been objectively impossible, and the agency would have been discharged. However, under these facts, the film company is entitled to damages for the agency's breach. (B) is incorrect because the film company's duty to perform (pay $500) was subject to the condition precedent of the agency's performance (flying), and, as discussed above, the agency breached the contract by failing to fly. Therefore, the film company's duty to pay never arose. The fact that the engine problem was not the agency's fault does not change things. The agency's inability to perform, even if it were due to impossibility, would merely discharge the contract, and each party would be excused from performance; the film company would not have to pay the $500. (C) is incorrect because, as determined above, the agency's duty was not discharged because performance was still possible. (If there had been objective impossibility, (C) would have been the correct choice.) (D) is incorrect because it suggests that the contract was a unilateral one (the offer to pay could be accepted only by completion of performance). This interpretation is clearly contrary to the facts. Although the film company offered to pay $500 for the flight, the agency accepted that offer by signing the contract. A promise to pay was given in exchange for a promise to fly. Thus, there was a contract to which both parties were bound.
201
A distributor of electric toy trains and a hobby shop owner entered into a written contract providing that the distributor will tender to the shop owner four dozen of a popular electric train set at a price of $100 apiece, to be delivered no later than October 31, to take advantage of the holiday shopping season. The shop owner chose to order from this distributor because its price for the train set was lower than that of other distributors. Shortly after the shop owner placed his order, the distributor raised its prices due to a sudden surge in popularity of that train set. Because the distributor did not have enough train sets to accommodate everyone due to the surge of orders, it decided to deliver train sets only to those buyers who had ordered them at the increased price. The distributor notified the shop owner that it would not deliver the train sets it ordered. The shop owner filed an action to force the distributor to deliver the train sets at the agreed-upon price. Will the court compel the distributor to deliver the train sets to the shop owner? Responses A No, because a contract for the sale of goods is not subject to specific performance. B No, because the shop owner can buy them from another distributor. C Yes, because the shop owner will not be able to buy them from another source at the contract price. D Yes, because time is of the essence.